You are on page 1of 67

Psychiatric Nursing Exam He is actively hallucinating, agitated, with irritated nasal

1. Situation: The nurse assigned in the detoxification septum.


unit attends to various patients with substance-related
a) Heroin
disorders. A 45 years old male revealed that he
b) cocaine
experienced a marked increase in his intake of alcohol
c) LSD
to achieve the desired effect This indicates:
d) Marijuana
a) withdrawal
The manifestations indicate intoxication with cocaine, a CNS
b) tolerance
stimulant.
c) intoxication
d) psychological dependence A. Intoxication with heroine is manifested by euphoria then
impairment in judgment, attention and the presence of
tolerance refers to the increase in the amount of the papillary constriction.
substance to achieve the same effects.
C. Intoxication with hallucinogen like LSD is manifested by
A. Withdrawal refers to the physical signs and symptoms that grandiosity, hallucinations, synesthesia and increase in vital
occur when the addictive substance is reduced or withheld. signs
B. Intoxication refers to the behavioral changes that occur D. Intoxication with Marijuana, a cannabinoid is manifested
upon recent ingestion of a substance. by sensation of slowed time, conjunctival redness, social
D. Psychological dependence refers to the intake of the withdrawal, impaired judgment and hallucinations
substance to prevent the onset of withdrawal symptoms. 5. A client is admitted with needle tracts on his arm,
2. The client admitted for alcohol detoxification stuporous and with pin point pupil will likely be
develops increased tremors, irritability, hypertension managed with:
and fever. The nurse should be alert for impending: a) Naltrexone (Revia)
a) delirium tremens b) Narcan (Naloxone)
b) Korsakoff’s syndrome c) Disulfiram (Antabuse)
c) esophageal varices d) Methadone (Dolophine)
d) Wernicke’s syndrome Narcan is a narcotic antagonist used to manage the CNS
Delirium Tremens is the most extreme central nervous system depression due to overdose with heroin.
irritability due to withdrawal from alcohol A. This is an opiate receptor blocker used to relieve the
B. This refers to an amnestic syndrome associated with craving for heroine
chronic alcoholism due to a deficiency in Vit. B C. Disulfiram is used as a deterrent in the use of alcohol.
C. This is a complication of liver cirrhosis which may be D. Methadone is used as a substitute in the withdrawal from
secondary to alcoholism. heroine
D. This is a complication of alcoholism characterized by 6. Situation: An old woman was brought for evaluation
irregularities of eye movements and lack of coordination.
due to the hospital for evaluation due to increasing
3. The care for the client places priority to which of the forgetfulness and limitations in daily function. The
following: daughter revealed that the client used her toothbrush
to comb her hair. She is manifesting:
a) Monitoring his vital signs every hour
b) Providing a quiet, dim room a) apraxia
c) Encouraging adequate fluids and nutritious b) aphasia
foods c) agnosia
d) Administering Librium as ordered d) amnesia

Pulse and blood pressure are usually elevated during This is the inability to recognize objects.
withdrawal, Elevation may indicate impending delirium
A. Apraxia is the inability to execute motor activities despite
tremens
intact comprehension.
B. Client needs quiet, well lighted, consistent and secure
B. Aphasia is the loss of ability to use or understand words.
environment. Excessive stimulation can aggravate anxiety
and cause illusions and hallucinations. D. Amnesia is loss of memory.

C. Adequate nutrition with supplement of Vit. B should be 7. She tearfully tells the nurse “I can’t take it when she
ensured. accuses me of stealing her things.” Which response by
D. Sedatives are used to relieve anxiety. the nurse will be most therapeutic?

4. Another client is brought to the emergency room by


friends who state that he took something an hour ago.
a) ” Don’t take it personally. Your mother does not Fluid volume deficit is the priority over altered nutrition since
mean it.” the situation indicates that the client is dehydrated.
b) “Have you tried discussing this with your A and D are psychosocial needs of a client with anorexia
mother?” nervosa but they are not the priority.
c) “This must be difficult for you and your
mother.” 12. What is the best intervention to teach the client
d) “Next time ask your mother where her things when she feels the need to starve?
were last seen.” a) Allow her to starve to relieve her anxiety
This reflecting the feeling of the daughter that shows b) Do a short-term exercise until the urge passes
empathy. c) Approach the nurse and talk out her feelings
d) Call her mother on the phone and tell her how
A and D. Giving advise does not encourage verbalization.
she feels
B. This response does not encourage verbalization of feelings.
The client with anorexia nervosa uses starvation as a way of
8. The primary nursing intervention in working with a managing anxiety. Talking out feelings with the nurse is an
client with moderate stage dementia is ensuring that adaptive coping.
the client: A. Starvation should not be encouraged. Physical safety is a
priority. Without adequate nutrition, a life threatening
a) receives adequate nutrition and hydration
situation exists.
b) will reminisce to decrease isolation
c) remains in a safe and secure environment B. The client with anorexia nervosa is preoccupied with losing
d) independently performs self-care weight due to disturbed body image. Limits should be set on
attempts to lose more weight.
9. She says to the nurse who offers her breakfast, “Oh
D. The client may have a domineering mother which causes
no, I will wait for my husband. We will eat together”
the client to feel ambivalent. The client will not discuss her
The therapeutic response by the nurse is:
feelings with her mother.
a) “Your husband is dead. Let me serve you your
13. The client with anorexia nervosa is improving if:
breakfast.”
b) “I’ve told you several times that he is dead. It’s a) She eats meals in the dining room.
time to eat.” b) Weight gain
c) “You’re going to have to wait a long time.” c) She attends ward activities.
d) “What made you say that your husband is alive? d) She has a more realistic self-concept.
The client should be reoriented to reality and be focused on Weight gain is the best indication of the client’s improvement.
the here and now. The goal is for the client to gain 1-2 pounds per week.

B. This is not a helpful approach because of the short-term (A)The client may purge after eating.
memory of the client.
(C) Attending an activity does not indicate improvement in
C. This indicates a pompous response. D. The cognitive nutritional state.
limitation of the client makes the client incapable of giving
(D) Body image is a factor in anorexia nervosa but it is not an
explanation.
indicator for improvement.
10. Dementia unlike delirium is characterized by:
14. The characteristic manifestation that will
a) slurred speech differentiate bulimia nervosa from anorexia nervosa is
b) insidious onset that bulimic individuals
c) clouding of consciousness
a) have episodic binge eating and purging
d) sensory perceptual change
b) have repeated attempts to stabilize their weight
Dementia has a gradual onset and progressive deterioration. c) have peculiar food handling patterns
It causes pronounced memory and cognitive disturbances. d) have threatened self-esteem
A, C and D are all characteristics of delirium. Bulimia is characterized by binge eating which is
characterized by taking in a large amount of food over a short
11. Situation: A 17-year-old gymnast is admitted to the
period of time.
hospital due to weight loss and dehydration secondary
to starvation. Which of the following nursing diagnoses B and C are characteristics of a client with anorexia nervosa
will be given priority for the client? D. Low esteem is noted in both eating disorders

a) altered self-image 15. A nursing diagnosis for bulimia nervosa is


b) fluid volume deficit powerlessness related to feeling not in control of eating
c) altered nutrition less than body requirements habits. The goal for this problem is:
d) altered family process a) Patient will learn problem solving skills
b) Patient will have decreased symptoms of a) Revealing personal information to the client
anxiety. b) Focusing on the feelings of the client.
c) Patient will perform self-care activities daily. c) Confronting the client about discrepancies in
d) Patient will verbalize how to set limits on verbal or non-verbal behavior
others. d) The client feels angry towards the nurse who
resembles his mother.
If the client learns problem solving skills, she will gain a sense
of control over her life. A. Countertransference is an emotional reaction of the nurse
on the client based on her unconscious needs and conflicts.
(B) Anxiety is caused by powerlessness.
B and C. These are therapeutic approaches.
(C) Performing self-care activities will not decrease one’s
powerlessness D. This is transference reaction where a client has an
emotional reaction towards the nurse based on her past.
(D) Setting limits to control imposed by others is a necessary
skill but problem-solving skill is the priority. 20. Which is the desired outcome in conducting
16. In the management of bulimic patients, the desensitization:
following nursing interventions will promote a a) The client verbalizes his fears about the
therapeutic relationship EXCEPT: situation
a) Establish an atmosphere of trust b) The client will voluntarily attend group therapy
b) Discuss their eating behavior. in the social hall.
c) Help patients identify feelings associated with c) The client will socialize with others willingly
binge-purge behavior d) The client will be able to overcome his disabling
d) Teach patient about bulimia nervosa fear.
The client will overcome his disabling fear by gradual
The client is often ashamed of her eating behavior. Discussion
exposure to the feared object.
should focus on feelings.
A, B and C are not the desired outcome of desensitization.
A, C and D promote a therapeutic relationship

17. Situation: A 35-year-old male has intense fear of 21. Which of the following should be included in the
riding an elevator. He claims “As if I will die inside.” This health teachings among clients receiving Valium:
has affected his studies. The client is suffering from: a) Avoid taking CNS depressant like alcohol.
a) agoraphobia b) There are no restrictions in activities.
b) social phobia c) Limit fluid intake.
c) Claustrophobia d) Any beverage like coffee may be taken
d) Xenophobia Valium is a CNS depressant. Taking it with other CNS
depressants like alcohol; potentiates its effect.
Claustrophobia is fear of closed space.
B. The client should be taught to avoid activities that require
A. Agoraphobia is fear of open space or being a situation
alertness.
where escape is difficult.
C. Valium causes dry mouth so the client must increase her
B. Social phobia is fear of performing in the presence of
fluid intake.
others in a way that will be humiliating or embarrassing.
D. Stimulants must not be taken by the client because it can
D. Xenophobia is fear of strangers.
decrease the effect of Valium.
18. Initial intervention for the client should be to:
22. Situation: A 20-year-old college student is admitted
a) Encourage to verbalize his fears as much as he to the medical ward because of sudden onset of
wants. paralysis of both legs. Extensive examination revealed
b) Assist him to find meaning to his feelings in no physical basis for the complaint. The nurse plans
relation to his past. intervention based on which correct statement about
c) Establish trust through a consistent approach. conversion disorder?
d) Accept her fears without criticizing.
a) The symptoms are conscious effort to control
The client cannot control her fears although the client knows anxiety
it’s silly and can joke about it. b) The client will experience high level of anxiety in
response to the paralysis.
A. Allow expression of the client’s fears but he should focus on
other productive activities as well. c) The conversion symptom has symbolic meaning
to the client
B and C. These are not the initial interventions. d) A confrontational approach will be beneficial for
19. The nurse develops a countertransference reaction. the client.
This is evidenced by: The client uses body symptoms to relieve anxiety.
A. The condition occurs unconsciously.

B. The client is not distressed by the lost or altered body Psychiatric Nursing Exam
function. 1. A client expresses to the nurse that she constantly
D. The client should not be confronted by the underlying feels irritated and loses her temper. During the course
cause of his condition because this can aggravate the client’s of the interview, the nurse finds that the client takes
anxiety. care of her mother who was confined to bed following a
stroke. The client struggles to balance caring for her
23. Nina reveals that the boyfriend has been pressuring
family and her mother. Which nursing diagnosis would
her to engage in premarital sex. The most therapeutic
the nurse most likely identify for this client?
response by the nurse is:
a) Compromised family adjustment
a) “I can refer you to a spiritual counselor if you
b) Caregiver role strain
like.”
c) Ineffective coping
b) “You shouldn’t allow anyone to pressure you
d) Anxiety
into sex.”
c) “It sounds like this problem is related to your 2. A middle-aged woman's father has passed away, and
paralysis.” her mother requires physical and emotional help due to
d) “How do you feel about being pressured into disabilities. The woman is married and raising two
sex by your boyfriend?” children, along with working full time. All of the factors
described are
Focusing on expression of feelings is therapeutic. The central
force of the client’s condition is anxiety. a) Stressors
A. This is not therapeutic because the nurse passes the b) Demands
responsibility to the counselor. c) Illnesses
d) Stimuli
B. Giving advice is not therapeutic.
3. An adolescent entering high school voices anxiety
C. This is not therapeutic because it confronts the underlying
over changing schools. Stating anxiety is an act of
cause.
a) Valuation
24. Malingering is different from somatoform disorder
b) Adaptation
because the former:
c) Evaluation
a) Has evidence of an organic basis. d) Reaction
b) It is a deliberate effort to handle upsetting
4. You are the nurse caring for a 72-year-old female
events
who is recovering from abdominal surgery on the
c) Gratification from the environment are
Medical Surgical unit. The surgery was very stressful and
obtained.
prolonged and you note on the chart that her blood
d) Stress is expressed through physical symptoms.
sugars are elevated yet she in not been diagnosed with
Malingering is a conscious simulation of an illness while diabetes. To what do you attribute this elevation in
somatoform disorder occurs unconscious. blood sugars?
A. Both disorders do not have an organic or structural basis. a) It is a result of antidiuretic hormone.
C. Both have primary gains. b) She must have had diabetes prior to surgery.
D. This is a characteristic of somatoform disorder. c) She has become a diabetic from the abdominal
surgery.
25. Unlike psychophysiologic disorder Linda may be best d) The blood sugars are probably a result of the
managed with: "fight-or-flight" reaction.
a) medical regimen 5. You walk into your patients' room and find her
b) milieu therapy sobbing uncontrollably. When you ask what the
c) stress management techniques problem is your patient responds "I am so scared. I have
d) psychotherapy never known anyone who goes into a hospital and
Stress management techniques is the best management of comes out alive." On this patient's care plan, you note a
somatoform disorder because the disorder is related to stress nursing diagnosis of "Ineffective coping related to
and it does not have a medical basis. stress". What is the best outcome you can expect for
this patient?
A.This disorder is not supported by organic pathology so no
medical regimen is required. a) Patient will avoid stressful situations.
B and D. Milieu therapy and psychotherapy may be used a b) Patient will start anti-anxiety agent.
therapeutic modalities but these are not the best. c) Patient will adapt relaxation techniques to
reduce stress.
d) Patient will be stress free.
6. The nurse would recognize that short-term things by using a stuffed animal to act as a trusted
pharmacological treatment may be appropriate if an friend. Which technique does this action best describe?
anxious patient's nursing diagnoses includes which of
a) Ventriloquism
the following?
b) Humor
a) Social isolation c) Storytelling
b) Decisional conflict d) Third-party communication
c) Disturbed sleep pattern
12. A nurse is working in a therapeutic counseling
d) Defensive coping
relationship with a client who has a mental illness.
7. A nurse is caring for a client who has been sexually When discussing the terms of the relationship, at which
abused. Which of the following interventions should the point is it appropriate to bring up the termination of the
nurse implement to establish rapport and to relationship?
demonstrate safety?
a) Just before terminating the relationship
a) Respond to shocking information by ignoring or b) At the beginning of the therapeutic counseling
disregarding the account relationship
b) Delay treatment until the client can talk about c) After the client has discussed essential elements
the situation of the work
c) Let the client spend time alone in a quiet area d) During the evaluation phase of the nursing care
d) Assess the client's stress level before plan
performing procedures
13. The nurse is caring for a client who is a doctor in a
8. A 30-year-old client with anxiety and depression is general hospital. He complains about the stressful
undergoing therapeutic counseling with a nurse. The condition of his job. Lately, he has become increasingly
client shares a lot of information with the nurse about susceptible to colds, headaches, muscular tension,
his personal life. Which of the following actions by the excessive tiredness, and many other symptoms. At what
nurse demonstrates healthy and appropriate boundary stage of stress is the client?
setting?
a) Secondary stage
a) The nurse sets a time limit of 45 minutes for a b) Exhaustion stage
discussion with the client c) Alarm stage
b) The nurse arranges to meet the client at a d) Resistance stage
coffee shop to finish the conversation when
14. A nurse has given her client a nursing diagnosis of
time is up
Anxiety because of her behavior when she found out
c) The nurse tells the client about her own feelings
that she was pregnant. Which of the following
of depression
interventions would be most appropriate for this type
d) The female nurse agrees to keep a client's
of nursing diagnosis? Select all that apply.
secret about his attraction to her
a) Assist with admitting the client to the
9. Which of the following actions become barriers to the
psychiatric unit
nurse-client communication? Select all that apply.
b) Encourage the client to talk
a) Asking the client, "Why?" c) Maintain a calm demeanor
b) Using open-ended questions d) Reassure the client that she is safe
c) Providing false reassurance e) Assist the client with identifying coping
d) Active listening mechanisms
e) Giving approval
15. A 51-year-old client is getting ready to undergo a
10. You are beginning a therapeutic relationship with a cardiac catheterization and is very anxious about the
client who has been admitted to the correctional facility procedure. Which intervention can the nurse provide
that you work at. She is a Type I diabetic who will be that will most likely help this client to remain calm?
incarcerated for approximately 4 months. When will
a) Tell the client that he will receive sedative
you begin to plan for the termination phase of this
medications so he will not be alert during the
relationship?
procedure
a) During the introductory phase b) Have the client's family sit next to him during
b) In 3 months the procedure
c) 2 weeks prior to release c) Explain what the client will see, hear, feel and
d) During the working phase experience during the procedure in terms that
he can understand
11. While caring for a child who has come from an
abusive home, a nurse tries to get the child to discuss
d) Give the client literature about the procedure a) Increased heart rate
ahead of time so that he can read about the b) Pinpoint pupils
process c) Changes in appetite
d) Increased respiratory rate
16. A nurse is caring for a 20-year-old client with
e) Vasodilation
testicular cancer. The client has expressed feelings of
anxiety related to his diagnosis and appears restless and 21. A client who is being prepared for surgery is
agitated every time the nurse tries to talk with him. experiencing severe anxiety about the procedure.
Which intervention would be the most appropriate for Which of the following interventions could the nurse
the nurse to help this client with anxiety? employ to decrease this client's anxiety levels? Select all
that apply.
a) Offer information about support groups
b) Explain that the client most likely needs a) Administer sedative medications to help the
antidepressants to get him through this time client sleep
c) Offer to have the provider talk with the client b) Provide reassurance by answering the client's
about his diagnosis questions
d) Tell the client that he will feel better if he talks c) Ask the client to help with certain tasks, such as
about it starting an IV
d) Employ music therapy or aromatherapy if
17. A nurse is working with a client to use biofeedback
available
as a method of controlling anxiety. Which of the
e) Encourage the client's own coping mechanisms
following outcomes would most likely result from
correct use of this mechanism? 22. An emergency department nurse is working with a
client who sought care for a sudden panic attack. The
a) The client being able to recognize that he
client tells the nurse that she feels like she is dying and
suffers from anxiety
is sweating and shaking. Which statement by the nurse
b) A resolution of the client's anxiety symptoms
is best?
c) An ability to recognize and control the body's
stress response a) You are not in a medical emergency, but you
d) A form of sedation that is calming when an are having an abnormal fight-or-flight response
anxiety attack occurs b) I have panic attacks too, and they are terrible
c) It is time to calm down now because you are
18. The nurse is preparing a presentation on stress and
finally in the hospital
anxiety. Which of the following is included as part of
d) This is nothing serious and is related to stress
this presentation? Select all that apply.
23. A client is scheduled for surgery and admits that he
a) Severe anxiety does not lead to psychosis
has never been so anxious in his life. Which of the
b) Severe anxiety can lead to suicidal thoughts
following symptoms are common for a client in this
c) Prolonged anxiety can cause illness
situation? Select all that apply.
d) Stress can cause anxiety
e) Anxiety can be motivating and increase learning a) Nausea
b) Headache
19. A client who has been diagnosed with anxiety tells
c) Blurry vision
the nurse that his thoughts contribute to his problems.
d) Chest pain
Which information can the nurse give the client that is
e) Tachycardia
true about automatic negative thoughts?
24. A nurse is caring for a client who is hospitalized in a
a) When you continue to struggle with anxiety,
very busy unit. The client complains to the nurse that it
you are not putting enough effort into changing
is too noisy to sleep and that he is not getting rest.
b) How you think about the world determines how
Which of the following nursing interventions is most
you feel and behave
appropriate to better help this client sleep?
c) Everything happens for a reason, and good will
come out of the experience a) Help the client to choose foods on the menu
d) Persons with anxiety must accept that things that promote sleep, such as broccoli
are the way they are b) Encourage the client to verbalize his sleep
difficulties
20. A client is suffering from stress and anxiety and is
c) Evaluate for the appropriate timing of client
being seen at a healthcare clinic for help and
care tasks
management. Which best describes the initial physical
d) Provide instructions to the client about the
effects of stress and anxiety in the body? Select all that
importance of going to bed and getting up at
apply.
the same time every day
25. A client with anxiety disorder is in a state of panic c) Help the client to learn about other
after becoming injured while outside. The nurse caring complementary or alternative treatments
for this client would most likely observe which available
behaviors? Select all that apply. d) Plan for a home health nurse to visit the client
daily to watch the client perform the relaxation
a) The client is moody
techniques
b) The client is disorganized
c) The client has a loss of rational thought 30. The nurse is caring for a client with generalized
d) The client demonstrates fear anxiety disorder requiring treatment. Which of the
e) The client needs direction to focus following treatments does the nurse anticipate the
client will need? Select all that apply.
26. A client is experiencing an anxiety attack. The nurse
reviews the client's home medications and notes that
they take an anti-anxiety medication. Which of the
a) Psychoanalysis
following medications is likely prescribed to this client?
b) SSRI medications
a) Amitryptiline c) BuSpar (buspirone)
b) Alprazolam d) Cognitive behavioral therapy
c) Alendronate e) Antipsychotic medications
d) Amiodarone
31. According to Erikson's developmental theory, when
27. A client presents to the emergency department with planning care for a 47-year-old client, which
an anxiety attack, stating that he has had multiple developmental task should a nurse identify as
attacks over the last six months. The client is appropriate for this client?
hyperventilating, so the nurse administers a
a) To develop a basic trust in others
benzodiazepine to help with the acute attack. The nurse
b) To achieve a sense of self-confidence and
anticipates that the client will begin to take which of the
recognition from others
following medications on a regular basis to help with
c) To reflect back on life events to derive pleasure
the anxiety?
and meaning
a) Fentanyl d) To achieve established life goals and consider
b) Famotidine the welfare of future generations
c) Furosemide
32. A nurse observes a 3-year-old client willingly sharing
d) Fluoxetine
candy with a sibling. According to Peplau, which
28. A nurse is working with a client who is having sleep psychological stage of development should the nurse
difficulties at night. The nurse recommends that the determine that this child has completed?
client keep a sleep diary and write down the times he
a) "Learning to count on others"
goes to bed, the times he gets up and how many times
b) "Learning to delay satisfaction"
he awakens each night. The nurse can best describe the
c) "Identifying oneself"
purpose of keeping a sleep diary as which of the
d) "Developing skills in participation"
following?
33. A 12-year-old girl becomes hysterical every time she
a) Assisting the client with making up all of his lost
strikes out in softball, falls down when roller-skating, or
sleep
loses when playing games. According to Peplau's
b) Determining the client's type of sleep disorder
interpersonal theory, in which stage of development
c) Helping the client to discover if he is a morning
should the nurse identify a need for improvement?
person
d) Helping the client see how to improve his sleep a) "Learning to count on others"
b) "Learning to delay satisfaction"
29. A case management nurse is reviewing the chart for
c) "Identifying oneself"
a client in a psychiatric hospital who has been
d) "Developing skills in participation"
diagnosed with panic disorder. The client is anxious and
irritable. The nurse has done teaching on relaxation 34. According to Peplau, a nurse who provides an
techniques, but the client requires close monitoring due abandoned child with parental guidance and praise
to lack of adherence to the regimen. Which action of following small accomplishments is serving which
the case manager will most likely support this client therapeutic role?
best?
a) The role of technical expert
a) Call to inform the provider that the client most b) The role of resource person
likely needs surgery c) The role of surrogate
b) Arrange for the client to be committed to long- d) The role of leader
term inpatient care
35. A physically healthy, 35-year-old single client lives 40. A nursing instructor is teaching about the
with parents who provide total financial support. application of Peplau's theory to nursing care. Which
According to Erikson's theory, which developmental student statement indicates that learning has occurred?
task should a nurse assist the client to accomplish?
a) "The nurse assumes the role of a parenting
a) Establishing the ability to control emotional figure instructing the client in good health
reactions practices."
b) Establishing a strong sense of ethics and b) "The nurse is concerned more about
character structure psychosocial functioning than physiological
c) Establishing and maintaining self-esteem functioning."
d) Establishing a career, personal relationships, c) "The nurse bases the client care plan on
and societal connections standardized nursing approaches and physician
orders."
36. A married, 26-year-old client works as a
d) "The nurse applies principles of human relations
schoolteacher. She and her husband have just had their
to the problems that arise at all levels of
first child. A nurse should recognize that this client is
experience."
successfully accomplishing which stage of Erikson's
developmental theory? 41. According to Peplau, treatment of client symptoms
should involve which nursing action?
a) Industry versus inferiority
b) Identity versus role confusion a) Establishing a therapeutic nurse-client
c) Intimacy versus isolation relationship
d) Generativity versus stagnation b) Using the technique of desensitization
c) Challenging clients' negative thoughts
37. A 10-year-old child wins the science fair competition
d) Uncovering clients' past experiences
and is chosen as a cheerleader for the basketball team.
A nurse should recognize that this child is in the process 42. A nurse is working with a client who suffers from
of successfully accomplishing which stage of Erikson's depression. The client has started taking medications
developmental theory? and is engaged in group therapy, but still tells the nurse,
"I do not like myself. I am annoying, even to me." Which
a) Industry versus inferiority
activities can the nurse suggest that would most likely
b) Identity versus role confusion
increase this client's self-concept? Select all that apply.
c) Intimacy versus isolation
d) Generativity versus stagnation a) Remind the client not to compare self to others
b) Teach the client how to be mindful of negative
38. A 29-year-old client living with parents has few
thoughts
interpersonal relationships. The client states, "I have
c) Have the client recognize personal strengths
trouble trusting people." Based on Erikson's
d) Ask if the client has any friends
developmental theory, which should the nurse
e) List three things that the nurse likes about the
recognize as a true statement about this client?
client
a) The client has not progressed beyond the trust
43. A client in the psychiatric unit has been diagnosed
versus mistrust developmental stage.
with severe, intractable depression and is preparing to
b) Developmental deficits in earlier life stages
undergo electroconvulsive therapy (ECT). Which best
have impaired the client's adult functioning.
describes the role of the nurse during ECT? Select all
c) The client cannot move to the next
that apply.
developmental stage until mastering all earlier
stages. a) Notify the provider immediately if the client is
d) The client's developmental problems began in confused following the procedure
the intimacy versus isolation stage. b) Prepare the client for applying EEG leads to the
head
39. Which statement describes achievement of Erikson's
c) Note the location of resuscitative equipment on
generativity versus stagnation developmental stage?
the nursing unit in case of need
a) "I've been a girl scout leader for troop 259 for 7 d) Record the length of time of the client's seizure
years." e) Administer oxygen as needed
b) "I feel great that I could pay for my bike with my
44. A client who has developed depression after
paper route money."
experiencing chronic back pain is being seen at the
c) "My parents are so pleased that John and I are
acute care clinic. Which best describes how the nurse
going to be married."
would demonstrate psychoeducation when working
d) "I've had a very full life. I'm not afraid to leave
with this client?
this world."
a) Teaching the client about the effects of their c) Do you hear voices or have hallucinations?
mental health issue so it can be better managed d) Have you been having trouble sleeping?
b) Helping the client to find a support group for
50. A client is diagnosed with major depressive disorder.
people who suffer from back pain
Which nursing diagnosis should a nurse assign to this
c) Administering medications that will relieve
client to address a behavioral symptom of this disorder?
some of the back pain
d) Providing information to the client about an a) Altered communication R/T feelings of
upcoming research study based on the effects worthlessness AEB anhedonia
of pain and depression b) Social isolation R/T poor self-esteem AEB
secluding self in room
45. A nurse is assisting a client who has undergone
c) Altered thought processes R/T hopelessness
electroconvulsive therapy for treatment of severe
AEB persecutory delusions
depression. Following the procedure, the client
d) Altered nutrition: less than body requirements
develops postictal agitation. Based on the nurse's
R/T high anxiety AEB anorexia
knowledge of this condition, the nurse would expect to
see which of the following? 51. A nurse assesses a client suspected of having major
depressive disorder. Which client symptom would
a) Coma
eliminate this diagnosis?
b) Hyperactivity and mania
c) Limb contractures a) The client is disheveled and malodorous.
d) Incoherence, disorientation, and motor b) The client refuses to interact with others.
restlessness c) The client is unable to feel any pleasure.
d) The client has maxed-out charge cards and
46. A 30-year-old client has been diagnosed with major
exhibits promiscuous behaviors.
depressive disorder. Which nutrition guideline should
the nurse give to this client that could help with some 52. A nurse reviews the laboratory data of a client
symptoms of depression? suspected of having major depressive disorder. Which
laboratory value would potentially rule out this
a) Fluids such as water and sugar-free juice are
diagnosis?
preferred over caffeinated beverages
b) The best diet for depression is the Western diet a) Thyroid-stimulating hormone (TSH) level of 6.2
c) Some free radicals in the diet combat harmful U/mL
antioxidants b) Potassium (K+) level of 4.2 mEq/L
d) The client should increase carbohydrate intake c) Sodium (Na+) level of 140 mEq/L
and limit fats d) Calcium (Ca2+) level of 9.5 mg/dL
47. An 11-year-old boy has been diagnosed with 53. What is the rationale for a nurse to perform a full
depression after his parents' divorce. The nurse physical health assessment on a client admitted with a
understands that depression in children of this age most diagnosis of major depressive disorder?
commonly manifests as:
a) The attention during the assessment is
a) Poor blood glucose control beneficial in decreasing social isolation.
b) Whining b) Depression can generate somatic symptoms
c) Low self esteem that can mask actual physical disorders.
d) Pouting c) Physical health complications are likely to arise
from antidepressant therapy.
48. A nurse is caring for a client has undergone ECT for
d) Depressed clients avoid addressing physical
the treatment of severe depression and has developed
health and ignore medical problems.
some complications afterward. Which of the following is
a physical side effect that has been associated with 54. A client who has been taking buspirone (BuSpar) as
ECT? prescribed for 2 days is close to discharge. Which
statement indicates to the nurse that the client has an
a) Muscle soreness
understanding of important discharge teaching?
b) Peripheral edema
c) Rash on the face and trunk a. "I cannot drink any alcohol with this
d) Bradycardia medication."
b. "It is going to take 2 to 3 weeks in order
49. A nurse is performing depression screenings in the
for me to begin to feel better."
community. Which question would most likely be
c. "This drug causes physical dependence
included in this screening?
and I need to strictly follow doctor's
a) Do you believe that others are talking or orders."
thinking about you?
b) Are you trying to lose weight?
d. "I can't take this medication with food. 60. A psych nurse is floated to the emergency room to
It needs to be taken on an empty help with the number of psych clients that are being
stomach." triaged. The psych nurse knows to see the client with
which of the following first?
55. A client is admitted to the psychiatric unit with a
diagnosis of major depression. The client is unable to a) State of mania, thinks bugs are crawling all over
concentrate, has no appetite, and is experiencing b) Severe depression and has not eaten for a 2
insomnia. Which should be included in this client's plan days
of care? c) Depression with suicidal ideation
d) ETOH of 200 mg/dl
a. A simple, structured daily schedule with
limited choices of activities 61. Which of the following situations are risk factors for
b. A daily schedule filled with activities to suicide? Select all that apply.
promote socialization
a) The client has an adequate support system
c. A flexible schedule that allows the client
b) The client has a history of suicide attempts
opportunities for decision making
c) The client is under 10 years of age
d. A schedule that includes mandatory
d) The client is terminally ill
activities to decrease social isolation
e) The client has been consistently rejected by
56. The nurse is caring for a 14-year-old client. The peers
client's mother informs the nurse that the client told
62. A client with a history of severe depression and
her that he wants to kill himself. What is the priority?
anxiety is in the hospital after attempting suicide. Which
a) Ask the mother to leave while the nurse evidence would most likely be seen that indicates a
completes a psych assessment crisis in a person with a mental illness? Select all that
b) Ask the client why he feels like he wants to die apply.
c) Consult the child psychiatrist
a) The client has not slept for several nights in a
d) Do not leave the client alone
row
57. The provider has finished seeing four clients in the b) The client is socially withdrawn
emergency room. The nurse should first assess the c) The client is unable to concentrate
client with which condition? d) The client has increased interest in personal
hygiene
a) Suicidal ideation
e) The client is crying
b) Persistent back pain
c) Injury to the right leg with obvious deformity 63. Common symptoms of impending suicide include
d) Fishing hook lodged in his forehead which of the following? Select all that apply.

58. A client with suicidal ideation is undergoing a) Giving away belongings


dialectical behavior therapy. The nurse should educate b) Sudden improvement in a depressed client
the client to expect which type of treatment? c) Canceling social engagements
d) Interacting with peers
a) An inpatient, solitary program
e) Changing a will
b) A 12-step program in the community
c) ECT in conjunction with outpatient therapy 64. The nurse is leading group therapy in a behavioral
d) Both individual and group counseling health unit. Which client concerns the nurse the most?

59. A client with depression has been taken to the a) The client with anorexia meticulously arranging
emergency department by a friend after making food on the tray, who has not eaten a bite
statements that he was contemplating suicide. The b) The client with severe depression who reports a
provider has ordered that the client be admitted for sudden and profound improvement in mood
inpatient treatment but the client refuses. Which of the c) The client on lithium who has discussed the
following best explains the client's rights in this desire to get pregnant
situation? d) The client with a history of schizophrenia whose
father passed last week, and who refuses to get
a) The client, if competent, has the right to refuse
out of bed
inpatient treatment
b) The client does not have the right to refuse 65. A client is undergoing behavioral therapy through
treatment and should be placed in restraints counseling for manic behaviors exhibited during
c) The client may refuse inpatient treatment but episodes of bipolar disorder. Which best describes how
must agree to outpatient therapy cognitive-behavioral therapy is used as treatment for a
d) The client is not safe and the nurse should fill client with bipolar disorder?
out a petition to legally keep the client
a) The client studies a book about thoughts vs. e) The client agrees to follow orders
behavior and implements the ideas
70. A client is undergoing a clinical interview as part of
b) The client meets with a group to discuss
diagnostic testing for bipolar disorder. Which best
thoughts and feelings
describes why a clinical interview would be conducted?
c) The client works at changing personal thoughts
to impact actions a) To test the client's memory function
d) The client initiates a 12-step program to b) To determine how much the client perceives
incorporate a change-through-leading approach and cognitively understands
c) To assess the client's psychiatric and family
66. client with bipolar disorder is seeking treatment for
background
co-occurring substance abuse disorder. During the
d) To measure the client's IQ level
initial assessment and treatment process, the nurse
notes the development of countertransference. Which 71. Which of the following are potential nursing
best describes an example of this response? interventions for a client with bipolar disorder? Select
all that apply.
a) The client blames the nurse for her bipolar
disorder a) Give detailed step-by-step instructions for ADLs
b) The client compares the nurse to her mother b) Encourage a daily routine
c) The client determines that her substance abuse c) Minimize environmental stimuli
disorder was caused by the bipolar disorder d) Assess for suicide risk
d) The nurse feels irritated because the client e) Put client valuables in a safe place
reminds the nurse of her sister
72. A nurse is caring for a client who has been
67. A client who has been diagnosed with bipolar diagnosed with bipolar disorder, type I. Which best
disorder is seeking treatment during the manic phase of describes the difference between bipolar type I and
illness. Based on the nurse's knowledge of this mental type II?
state, the nurse understands that which of the following
is likely during this phase? a) Bipolar I is associated with alternating mania
and depression, while bipolar II is a continuous
a) The client is at risk of destructive behaviors state of hypomania
because of her manic mood b) Bipolar I leads to extreme depression, while
b) The client is more likely to develop chronic bipolar II causes more subdued depression
illnesses, including lung disease and obesity c) Bipolar I describes mania or the high feeling of
c) The client is at risk of suicide during the manic the disease, while bipolar II describes the
phase depressed state
d) The client is not at high risk and her safety is not d) Bipolar I is associated with alternating mania
threatened and depression, while bipolar II is associated
with hypomania and depression
68. A nurse is caring for a client with bipolar disorder
who is frequently manic. What describes the most
appropriate psychotherapy for a client with this
condition? Select all that apply.

a) Family focused therapy


b) Electroconvulsive therapy
c) Cognitive-behavior therapy Psychiatric Nursing Exam
d) 12-step programs 1. A client with a diagnosis of depression who has
e) Interpersonal and social rhythm therapy attempted suicide says to the nurse, "I should have
died. I've always been a failure. Nothing ever goes
69. A client with a history of bipolar disorder has been
right for me." Which response by the nurse
brought in to the hospital because the client was
demonstrates therapeutic communication?
running through a neighborhood without any clothes
on. Which of the following is evidence that the client is a) "You have everything to live for."
voluntarily willing to be admitted to the hospital? Select b) "Why do you see yourself as a failure?"
all that apply. c) "Feeling like this is all part of being
depressed."
d) "You've been feeling like a failure for a
a) The situation is an emergency
while?"
b) The client is not a danger to himself or others
c) The client has a court order 2. The nurse visits a client at home. The client
d) The client states that he is willing to follow the states, "I haven't slept at all the last couple of
regimen of the facility
nights." Which response by the nurse 7. A client is preparing to attend a mN for the first time.
demonstrates therapeutic communication? The nurse should tell the client that which is the first
step in this 12-step program?
a) "I see."
b) "Really?" a) Admitting to having a problem
b) Substituting other activities for gambling
c) "You're having difficulty sleeping?"
c) Stating that the gambling will be stopped
d) "Sometimes I have trouble sleeping too."
d) Discontinuing relationships with people who
3. A client experiencing disturbed thought gamble
processes believes that his food is being poisoned. 8. A client says to the nurse, "The federal guards were
Which communication technique should the nurse sent to kill me." Which is the best response by the nurse
use to encourage the client to eat? to the client's concern?

a) Using open-ended questions and silence a) "I don't believe this is true."
b) Sharing personal preference regarding food b) "The guards are not out to kill you."
choices c) "Do you feel afraid that people are trying to
c) Documenting reasons why the client does hurt you?"
not want to eat d) "What makes you think the guards were sent to
hurt you?"
d) Offering opinions about the necessity of
adequate nutrition 9. A client diagnosed with delirium becomes disoriented
and confused at night. Which intervention should the
4. A client admitted voluntarily for treatment of an nurse implement initially?
anxiety disorder demands to be released from the
hospital. Which action should the nurse take a) Move the client next to the nurses' station.
initially? b) Use an indirect light source and turn off the
television.
a) Contact the client's health care provider c) Keep the television and a soft light on during
(HCP). the night.
b) Call the client's family to arrange for d) Play soft music during the night, and maintain a
transportation. well-lit room.
c) Attempt to persuade the client to stay "for 10. A client is admitted to the mental health unit with a
only a few more days." diagnosis of depression. The nurse should develop a
d) Tell the client that leaving would likely plan of care for the client that includes which
result in an involuntary commitment. intervention?

5. When reviewing the admission assessment, the nurse a) Encouraging quiet reading and writing for the
notes that a client was admitted to the mental health first few days
unit involuntarily. Based on this type of admission, the b) Identification of physical activities that will
nurse should provide which intervention for this client? provide exercise
c) No socializing activities, until the client asks to
1. Monitor closely for harm to self or others.
participate in milieu
2. Assist in completing an application for
d) A structured program of activities in which the
admission.
client can participate
3. Supply the client with written information
about his or her mental illness. 11. A client diagnosed with terminal cancer says to the
4. Provide an opportunity for the family to discuss nurse, "I'm going to die, and I wish my family would
why they felt the admission was needed. stop hoping for a cure! I get so angry when they carry
on like this. After all, I'm the one who's dying." Which
6. When a client is admitted to an inpatient mental
response by the nurse is therapeutic?
health unit with the diagnosis of anorexia nervosa, a
cognitive behavioral approach is used as part of the a) "Have you shared your feelings with your
treatment plan. The nurse plans care based on which family?"
purpose of this approach? b) "I think we should talk more about your anger
with your family."
a) Providing a supportive environment
c) "You're feeling angry that your family continues
b) Examining intrapsychic conflicts and past issues
to hope for you to be cured?"
c) Emphasizing social interaction with clients who
d) "You are probably very depressed, which is
withdraw
understandable with such a diagnosis."
d) Helping the client to examine dysfunctional
thoughts and beliefs 12. On review of the client's record, the nurse notes
that the admission was voluntary. Based on this
information, the nurse plans care anticipating which 17. Which statement about mental illness is true?
client behavior?
a. Mental illness is a matter of individual
a) Fearfulness regarding treatment measures nonconformity with societal norms.
b) Anger and aggressiveness directed toward b. Mental illness is present when irrational and
others illogical behavior occurs.
c) An understanding of the pathology and c. Mental illness changes with culture, time in
symptoms of the diagnosis history, political systems, and the groups
d) A willingness to participate in the planning of defining it.
the care and treatment plan d. Mental illness is evaluated solely by considering
individual control over behavior and appraisal
13. Recognizing the frequency of depression among the
of reality.
American population, the nurse should advocate for
which mental health promotion intervention? 18. When considering stigmatization, which statement
made by the nurse demonstrates a need for immediate
a. Including discussions on depression as part of
intervention by the nurse manager?
school health classes
b. Providing regular depression screening for a. "Depression seems to be a real problem among
adolescent and teenage students the teenage population."
c. Increasing the number of community-based b. "My experience has been that the Irish have a
depression hotlines available to the public problem with alcohol use."
d. Encouraging senior centers to provide c. "Women are at greater risk for developing
information on accessing community suicidal thoughts then acting on them."
depression resources d. "We've admitted several military veterans with
posttraumatic stress disorder this month."
14. Which statement made by a patient demonstrates a
healthy degree of resilience? Select all that apply. 19. When discussing therapy options, the nurse should
provide information about interpersonal therapy to
a. "I try to remember not to take other people's
which patient? Select all that apply.
bad moods personally."
b. "I know that if I get really mad I'll end up being a. The teenager who is the focus of bullying at
depressed." school
c. "I really feel that sometimes bad things are b. The older woman who has just lost her life
meant to happen." partner to cancer
d. "I've learned to calm down before trying to c. The young adult who has begun demonstrating
defend my opinions." hoarding tendencies
e. "I know that discussing issues with my boss d. The adolescent demonstrating aggressive verbal
would help me get my point across." and physical tendencies
e. The middle-aged adult who recently discovered
15. Epidemiological studies contribute to improvements
her partner has been unfaithful
in care for individuals with mental disorders by:
20. When considering the suggestions of Hildegard
a. Providing information about effective nursing
Peplau, which activity should the nurse regularly engage
techniques.
in to ensure that the patient stays the focus of all
b. Identifying risk factors that contribute to the
therapeutic conversations?
development of a disorder.
c. Identifying individuals in the general population a. Assessing the patient for unexpressed concerns
who will develop a specific disorder. and fears
d. Identifying which individuals will respond b. Evaluating the possible need for additional
favorably to a specific treatment. training and education
c. Reflecting on personal behaviors and personal
16. Which statement demonstrates the nurse's
needs
understanding of the effect of environmental factors on
d. Avoiding power struggles with the manipulative
a patient's mental health?
patient
a) "I'll need to assess how the patient's family
21. Which action reflects therapeutic practices
views mental illness."
associated with operant conditioning?
b) "There is a history of depression in the patient's
extended family." a. Encouraging a parent to read to their children
c) "I'm not familiar with the patient's Japanese's to foster a love for learning
cultural view on suicide." b. Encouraging a patient to make daily journal
d) "The patient's ability to pay for mental health entries describing their feelings
services needs to be assessed."
c. Suggesting to a new mother that she spend 27. Which drug group calls for nursing assessment for
time cuddling her newborn often during the day development of abnormal movement disorders among
d. Acknowledging a patient who is often verbally individuals who take therapeutic dosages?
aggressive for complimenting a picture another
a) SSRIs
patient drew
b) antipsychotics
22. A male patient reports to the nurse, "I'm told I have c) benzodiazepines
memories of childhood abuse stored in my unconscious d) tricyclic antidepressants
mind. I want to work on this." Based on this statement,
what information should the nurse provide the patient?
28. The nurse administers each of the following drugs to
a. To seek the help of a trained therapist to help
various patients. The patient who should be most
uncover and deal with the trauma associated
carefully assessed for fluid and electrolyte imbalance is
with those memories.
the one receiving:
b. How to use a defense mechanism such as
suppression so that the memories will be less a) lithium (Eskalith)
threatening. b) clozapine (Clozaril)
c. Psychodynamic therapy will allow the surfacing c) diazepam (Valium)
of those unconscious memories to occur in just d) amitriptyline
a few sessions.
d. Group sessions are valuable to identify
underlying themes of the memories being
suppressed.
29. Which nursing intervention demonstrates the
23. A nurse is assessing a patient who graduated at the ethical principle of beneficence?
top of his class but now obsesses about being
incompetent in his new job. The nurse recognizes that a. Refusing to administer a placebo to a patient.
this patient may benefit from the following type of b. Attending an in-service on the operation of the
psychotherapy: new IV infusion pumps
c. Providing frequent updates to the family of a
a. Interpersonal patient currently in surgery
b. Operant conditioning d. Respecting the right of the patient to make
c. Behavioral decisions about whether or not to have
d. Cognitive-behavioral electroconvulsive therapy
24. Which question should the nurse ask when 30. According to Maslow's hierarchy of needs, the most
assessing for what Sullivan's Interpersonal Theory basic needs category for nurses to address is:
identifies as the most painful human condition?
a. physiological
a. "Is self-esteem important to you?" b. safety
b. "Do you think of yourself as being lonely?" c. love and belonging
c. "What do you do to manage your anxiety?" d. self-actualization
d. "Have you ever been diagnosed with
depression?" 31. When considering facility admissions for mental
healthcare, what characteristic is unique to a voluntary
25. Linda is terrified of spiders and cannot explain why. admission?
Because she lives in a wooded area, she would like to
overcome this overwhelming fear. Her nurse a. The patient poses no substantial threat to
practitioner suggests which therapy? themselves or to others
b. The patient has the right to seek legal counsel
a. Behavioral c. A request in writing is required before
b. Biofeedback admission
c. Aversion d. A mental illness has been previously diagnosed
d. Systematic desensitization
32. In an outpatient psychiatric clinic, a nurse notices
26. Besides antianxiety agents, which classification of that a newly admitted young male patient smiles when
drugs is also commonly given to treat anxiety and he sees her. One day the young man tells the nurse,
anxiety disorders? "You are pretty like my mother." The nurse recognizes
a) Antipsychotics that the male is exhibiting:
b) Mood stabilizers a. Transference
c) Antidepressants b. Id expression
d) Cholinesterase inhibitors c. Countertransference
d. A cognitive distortion a. The mother of an unconscious patient saying
okay to surgery
33. Which drug group calls for nursing assessment for
b. Care given to a heroin overdose victim
development of abnormal movement disorders among
c. Immobilizing a patient who has refused to take
individuals who take therapeutic dosages?
medication
a. SSRIs d. Signing general intake paperwork with specific
b. antipsychotics parameters
c. benzodiazepines
38. Based on Maslow's hierarchy of needs, physiological
d. tricyclic antidepressants
needs for a restrained patient include: Select all that
34. A patient is telling a tearful story. The nurse listens apply.
empathically and responds therapeutically with:
a. Private toileting, oral hydration
a. "The next time you find yourself in a similar b. Checking the tightness of the restraints
situation, please call me." c. Therapeutic communication
b. "I am sorry this situation made you feel so d. Maintaining a patent airway
badly. Would you like some tea?"
39. What assessment question is focused on identifying
c. "Let's devise a plan on how you will react next
a long-term consequence of chronic stress on physical
time in a similar situation."
health?
d. "I am sorry that your friend was so thoughtless.
You should be treated better." a. "Do you have any problems with sleeping well?"
b. "How many infections have you experienced in
35. You realize that your patient who is being treated
the past 6 months?"
for a major depressive disorder requires more teaching
c. "How much moderate exercise do you engage
when she makes the following statement:
in on a regular basis?"
a. "I have been on this antidepressant for 3 days. I d. "What management techniques to you regularly
realize that the full effect may not happen for a use to manage your stress?"
period of weeks."
40. Which nursing assessments are directed at
b. "I am going to ask my nurse practitioner to
monitoring a patient's fight-or-flight response? Select all
discontinue my Prozac today and let me start
that apply.
taking a monoamine oxidase inhibitor
tomorrow." a. Blood pressure
c. "I may ask to have my medication changed to b. Heart rate
Wellbutrin due to the problems I am having c. Respiratory rate
being romantic with my wife." d. Abdominal pain
d. "I realize that there are many antidepressants e. Dilated pupils
and it might take a while until we find the one
41. When considering stress, what is the primary goal of
that works best for me."
making daily entries into a personal journal?
36. A nurse makes a post on a social media page about
a. Providing a distraction from the daily stress
his peer taking care of a patient with a crime-related
b. Expressing emotions to manage stress
gunshot wound in the emergency department. He does
c. Identifying stress triggers
not use the name of the patient. The nurse:
d. Focusing on one's stress
a. Has not violated confidentiality laws because he
did not use the patient's name.
b. Cannot be held liable for violating
confidentiality laws because he was not the
primary nurse for the patient.
c. Has violated confidentiality laws and can be
held liable.
d. Cannot be held liable because postings on a
social media site are excluded from
confidentiality laws.

37. Implied consent occurs when no verbal or written


agreement takes place prior to a caregiver delivering
treatment. Which of the following examples represents
implied consent?
understands that it is important to ensure which
primary intervention?

 Including the client's support system in the


Psychiatric Nursing Exam
teaching
1. A moderately depressed client who was hospitalized
2 days ago suddenly begins smiling and reporting that 9. A depressed client who appeared sullen, distraught,
the crisis is over. The client says to the nurse, "I'm finally and hopeless a few days ago now suddenly appears
cured." How should the nurse interpret this behavior as calm, relaxed, and more energetic. Which is the nurse's
a cue to modify the treatment plan? best initial action with regard to the client's altered
demeanor?
 Increasing the level of suicide precautions
 Engage the client in one-to-one supervision,
2.Which are the most likely characteristics of a client
share with the client the observations that have
who abuses alcohol? Select all that apply.
been assessed, and ask whether the client is
 Male gender thinking about suicide.
 Abuses drugs as well as alcohol
10. Which statement by the nurse indicates a need for
 History of at least one suicide attempt
further teaching concerning family violence?

 "Abusers are more often from low-income


3. The nurse is performing an assessment on a client families."
being admitted with a diagnosis of alcohol dependence
11. When planning the discharge of a client with chronic
who reports it's been 6 hours since the last drink. The
anxiety, the nurse directs the goals at promoting a safe
information supports which assumption about the
environment at home. Which is the most appropriate
appearance of withdrawal symptoms?
maintenance goal?
 Signs may appear at any time.
 Identifying anxiety-producing situations

12. A client is participating in a therapy group and


4. The nurse reviewing a client's diagnostic results focuses on viewing all team members as equally
recognizes that which is a possible positive indication important in helping the clients to meet their goals. The
for a diagnosis of schizophrenia? nurse is implementing which therapeutic approach?

 Atrophy of the lateral and/or third ventricles of  Milieu therapy


the brain
13. The nurse is caring for a client with a diagnosis of
agoraphobia. Which statement made by the client
would support this diagnosis?
5. The client diagnosed with mild depression says to the
nurse, "I haven't had an appetite at all for the last few  "I'd be sure to have a panic attack if I left my
weeks." Which response by the nurse best assesses the house."
client's nutritional issue?
14. The nurse is creating a plan of care for the client
 "You haven't had an appetite at all?" who is upset following the loss of a job and is
verbalizing concerns regarding the ability to meet
6. The mental health nurse is meeting with a client who financial obligations. Which problem is the basis of the
has a long history of abusing drugs. During the session client's concerns?
the client says to the nurse, "I'm feeling much better
now, and I'm ready to go straight." Which response by  Inability to meet role expectations
the nurse would be therapeutic?
15. The nurse is working with a client who is
 "Tell me what makes you feel that you are demonstrating delusional thinking. The client says to
ready." the nurse, "The leaders of a religious cult are being sent
to assassinate me." Which is the best response by the
7. The nurse visits a client at home. The client states, "I nurse?
haven't slept at all the last couple of nights." Which
response by the nurse demonstrates therapeutic  "I don't know about a religious cult. Are you
communication? afraid that people are trying to hurt you?"

 "You're having difficulty sleeping?" 16. During a nursing interview, a client says, "My
daughter was murdered. I can't help wondering if her
8. The nurse is planning relapse prevention information husband killed her, but he's been eliminated as a
for a client diagnosed with schizophrenia. The nurse suspect." Which statement is a therapeutic nursing
response?
 "Have you shared your concerns with the The client is lying on the bed in a fetal position. Which is
police?" the most appropriate nursing intervention?

17. The nurse should include which information in the  Sit beside the client in silence with occasional
medication teaching plan for a client diagnosed with open-ended questions.
schizophrenia?
26. The nurse is caring for a client with anorexia
 Coffee, tea, and soda consumption should be nervosa. Which behavior is characteristic of this
limited. disorder and reflects anxiety management?

18. A client experiencing a great deal of stress and  Observing rigid rules and regulations
anxiety is being taught to use self-control therapy.
27. A nursing student is assisting with the care of a
Which statement by the client indicates a need for
client with a chronic mental illness. The nurse informs
further teaching about the therapy?
the student that a behavior modification approach
 "This form of therapy provides a negative (operant conditioning) will be used in treatment for the
reinforcement when the stimulus is produced." client. Which statement by the student indicates a need
for further information about the therapy?
19. Soon after an assault, a client is assessed in the
emergency department with behavior that is associated  "It uses negative reinforcement."
with severe anxiety. Which client behaviors support this
28. The mother of a teenage client states that her
level of anxiety?
daughter, diagnosed with an anxiety disorder, "eats
 Is pacing while describing the situation using a nothing but junk food, has never liked going to school,
rapid speech pattern and hangs out with the wrong crowd." What discharge
instruction will be most effective in helping the mother
20. The nurse is planning to formulate a psychotherapy
to manage her daughter's condition?
group. Several clients are interested in attending the
session. The nurse plans the group, based on which  Restrict the amount of chocolate and caffeine
management principle? products in the home.

 The group should be limited to no more than 10 29. Which behavior demonstrated by a client diagnosed
members. with depression indicates a need for suicide
precautions?
21. The nurse is creating a plan of care for a client
diagnosed with depression whose food intake is poor.  Asks about how to get a will notarized
The nurse should include which interventions in the
30. The nurse has been closely observing a client who
plan of care? Select all that apply.
has been displaying aggressive behaviors. The nurse
 1.Assist the client in selecting foods from the observes that the behavior displayed by the client is
food menu. escalating. Which nursing intervention is most helpful to
 2.Offer high-calorie fluids throughout the day this client at this time? Select all that apply.
and evening.
 Acknowledge the client's behavior.
 4.Offer small high-calorie, high-protein snacks
 Assist the client to an area that is quiet.
during the day and evening.
 Maintain a safe distance from the client.
22. The client with a diagnosis of dependent personality
disorder is most likely to have problems coping with
which situation?

 Making decisions about living arrangements


after discharge

23. Which client is at greatest risk for committing


suicide?

 A client with metastatic cancer

24. Which pre-electroconvulsive therapy intervention


will the nurse implement for a hospitalized client?

 Assure that an electrocardiogram is performed


within 24 hours.

25. The nurse is caring for a client just admitted to the


mental health unit and diagnosed with catatonic stupor.
4. A 5-year-old boy is diagnosed to have autistic
disorder. Which of the following manifestations may be
Psychiatric Nursing Exam noted in a client with autistic disorder?
1. Situation: A nurse may encounter children with
mental disorders. Her knowledge of these various A. argumentativeness, disobedience,
disorders is vital. When planning school interventions angry outburst
for a child with a diagnosis of attention deficit B. intolerance to change, disturbed
hyperactivity disorder, a guide to remember is to: relatedness, stereotypes
C. distractibility, impulsiveness and over
A. provide as much structure as possible
activity
for the child
D. aggression, truancy, stealing, lying
B. ignore the child's overactivity.
C. encourage the child to engage in any These are manifestations of autistic disorder.
play activity to dissipate energy
A. These manifestations are noted in Oppositional
D. remove the child from the classroom
Defiant Disorder, a disruptive disorder among children.
when disruptive behavior occurs
C. These are manifestations of Attention Deficit Disorder
Decrease stimuli for behavior control thru an environment
that is free of distractions, a calm non -confrontational D. These are the manifestations of Conduct Disorder
approach and setting limit to time allotted for activities.

B. The child will not benefit from a lenient approach.

C. Dissipate energy through safe activities.

D. This indicates that the classroom environment lacks


structure.

DSM-V criteria for ADHD


2. Ritalin is the drug of choice for children with ADHD. A. A persistent pattern of inattention and/or
The side effects of the following may be noted: hyperactivity-impulsivity that interferes with functioning
A. increased attention span and or development, as characterized by either
concentration B. Six or more symptoms of inattentive or hyperactive-
B. increase in appetite impulsive symptoms for children up to age 16, or five or
C. sleepiness and lethargy more for age 17 and older. Symptoms have been
D. bradycardia and diarrhea present for at least 6 months and are inappropriate for
The medication has a paradoxic effect that decrease developmental level. Several hyperactive-impulsive
hyperactivity and impulsivity among children with ADHD. symptoms were present prior to age 12.

B, C, D. Side effects of Ritalin include anorexia, insomnia, C. Several inattentive or hyperactive-impulsive


diarrhea and irritability. symptoms are present in two or more settings (e.g., at
home, school, or work; with friends or relatives; in other
activities).
3. The therapeutic approach in the care of an autistic
D. There is clear evidence that the symptoms interfere
child include the following EXCEPT:
with, or reduce the quality of, social, academic, or
A. Engage in diversionary activities when acting -out occupational functioning.

B. Provide an atmosphere of acceptance E. The symptoms do not occur exclusively during the
course of schizophrenia or another psychotic disorder
C. Provide safety measures
and are not better explained by another mental
D. Rearrange the environment to activate the child disorder.

The child with autistic disorder does not want change. Inattentive Examples for ADHD
Maintaining a consistent environment is therapeutic.
*Fails to give close attention to details
A. Angry outburst can be rechanneled through safe activities.
*Difficulty sustaining attention in tasks or play
B. Acceptance enhances a trusting relationship.
*Avoids, dislikes tasks that require sustained mental
C. Ensure safety from self-destructive behaviors like head effort
banging and hair pulling.
*Often does not seem to listen

*Poor follow through to activities


*Difficulty with organization *School/community resources and accommodations

*Loses things necessary for tasks

*Easily distracted Pharmacological Therapies for ADHD

*Forgetful *STIMULANT Medications

(SIDE EFFECTS: Insomnia, nervousness, agitation,


palpitations, anorexia, weight loss, changes in BP/pulse,
Hyperactivity Examples for ADHD
impaired growth rates)
*Fidgets
*Ritalin
*Trouble staying in seat
*Concerta
*Runs about or climbs excessively and inappropriately
*Metadate CD
*Difficulty playing quietly
*Focalin
*Always on the "go"
*Dexedrine
*Talks excessively
*Adderall

*Daytrana
Impulsivity Examples for ADHD
*Vyvanse
*Blurts out answers before question is completed

*Difficulty waiting for turn


*NONSTIMULANT Medication
*Interrupts or intrudes on others
(SIDE EFFECTS: Suicide, hepatic damage)

*atomoxetine (Strattera)
Screening and Diagnosis for ADHD
*selective norepinephrine reuptake inhibitor
*Usually brought for evaluation when behaviors
interfere with school when it Interferes with the daily
functioning of teachers or parents" (Maternal Child...) What are the goals of medications for ADHD?

*Complete history evaluation done *Improve impulse control

*Testing on child *ability to concentrate

*Questionnaires administered to parents, caregivers, *decrease distractibility


teachers:
*About 80 percent of children who need medication for
*Child Behavior checklist ADHD still need it as teenagers. Over 50 percent need
medication as adults.
*Connors-Wells Self-Report scales

*Conner’s' Rating Scales


Cornerstone medication for ADHD/ADD
*ADHD Rating Scale-IV
*Methylphenidate (Ritalin)
Treatment for ADHD
*Daily dosage above 60 mg not recommended.
*Management is a collaborative effort
Estimated dose range .3-.6 mg/kg/dose
*Multimodal Therapy including;
SIDE EFFECTS: Insomnia, decreased appetite, weight
*Pharmacotherapy loss, headache, irritability, stomachache, and rebound
agitation or exaggeration of pre-medication symptoms
*Behavioral Interventions, such as: as it is wearing off.
*Behavior Modification techniques *Works quickly (within 30-60 minutes).
*Cognitive behavior therapy *Use cautiously in patients with marked anxiety, motor
*Patient, Parent, Caregiver education tics or with family history of Tourette syndrome, or
history of substance abuse.
*Coaching and skills-training programs
*Don't use if glaucoma or on MAOI
*Effective in 70-80% of children 4) Saliency

*Approved for 6yo+ *Consequences should be meaningful and noticeable to


child

Stimulant medication for ADHD/ADD that is approved


for 3 yo? Four Hallmarks of effective psychosocial interventions
within behavioral plans
Dextroamphetamine (Dexedrine)
1) Consistency
*Stimulant
*All caregivers/teachers must adhere and administer
*Give last dose in early afternoon
the behavior plan consistently
SIDE EFFECTS: appetite suppression, insomnia
2) Immediacy
*full drug effect takes 2 days
*Feedback and consequences must be administered
*Approved for 3+ immediately

3) Specificity

amphetamine (Adderall) *Caregivers/teachers must be explicit about behaviors


being targeted
*Stimulant: Schedule II Medication
4) Saliency
*Used for ADHA and Narcolepsy
*Consequences should be meaningful and noticeable to
*Can be abused as a performance enhancer (both child
physical and cognitive) and effect of euphoria.

*Side Effects and Assessment


What are the main side effects of the medications used
*Monitor LFTs for ADHD/ADD?
*Hyper or Hypotension from vagal stimulation, Insomnia
Raynauds phenomen, tachycardia
Decreased appetite
*Erectile dysfunction/priapism
Weight loss
*Insomnia and increased alertness
Headache
*appetite suppression
Inhibits Growth
*may take 2 weeks for full effect

*Approved for 3yo +

Behavior Management Plan for ADHD/ADD

*Minimize environmental distractions

*Four Hallmarks of effective psychosocial interventions


within behavioral plans

1) Consistency

*All caregivers/teachers must adhere and administer


the behavior plan consistently

2) Immediacy

*Feedback and consequences must be administered


immediately

3) Specificity

*Caregivers/teachers must be explicit about behaviors


being targeted
6. The nurse is planning care for a client who has a
phobic disorder manifested by a fear of elevators.
Psychiatric Nursing Exam Which goal would need to be accomplished first?
1. The client with anorexia nervosa is improving if:
A. The client demonstrates the relaxation
A. She eats meals in the dining room.
response when asked.
B. Weight gain
B. verbalizes the underlying cause of the disorder.
C. She attends ward activities.
C. rides the elevator in the company of the nurse.
D. She has a more realistic self-concept.
D. role plays the use of an elevator
it is the best indication of the client's improvement. The goal
is for the client to gain 1-2 pounds per week.
7.The nurse is caring for a female client who was
admitted for Anorexia Nervosa. The nurse enters the
clients room and notes that the client is engaged in
2. A nursing diagnosis for bulimia nervosa is
rigorous push-ups. Which nursing action is most
powerlessness related to feeling not in control of eating
appropriate?
habits. The goal for this problem is:
A. Interrupt the client and weigh her immediately
A. Patient will learn problem solving skills
B. Interrupt the client and offer to take her for a
B. Patient will have decreased symptoms of
walk
anxiety.
C. Allow the client to complete her exercise
C. Patient will perform self-care activities daily.
program
D. Patient will verbalize how to set limits on
D. Tell the client that she is not allowed to exercise
others
rigorously.
This will help the patient gain a sense of control in life.
The rigorous pushups can be really harmful to her health. The
nurse must provide for appropriate exercise and place limits
on rigorous activities. weighing the client immediately after
3.The nurse develops a countertransference reaction. reinforces the client’s preoccupation with weight
This is evidenced by:

A. Revealing personal information to the client


B. Focusing on the feelings of the client. Psychiatric Nursing Exam
1. A hospitalized client is started on phenelzine for the
C. Confronting the client about discrepancies in
treatment of depression. The nurse should instruct the
verbal or non-verbal behavior
client that which foods are acceptable to consume while
D. The client feels angry towards the nurse who
taking this medication? Select all that apply.
resembles his mother.
A. Countertransference is an emotional reaction of the nurse a) Figs
on the client based on her unconscious needs and conflicts b) Yogurt
c) Crackers
d) Aged cheese
4. ___________________ techniques is the best e) Tossed salad
management of somatoform disorder because the f) Oatmeal raisin cookies
disorder is related to _____________ and it does not 2. A client with schizophrenia has been started on
have a medical basis medication therapy with clozapine. The nurse should
Stress management techniques are the best management of assess the results of which laboratory study to monitor
somatoform disorders because the disorder is related to for adverse effects from this medication?
stress/anxiety
a) Platelet count
b) Blood glucose level
c) Liver function tests
5. A client with panic disorder experiences an acute
d) White blood cell count
attack while the nurse is completing an admission
assessment. List the following interventions according 3. A client is admitted with a recent history of severe
to their level of priority. anxiety following a home invasion and robbery. During
the initial assessment interview, which statement by
a. Remain with the client.
the client would indicate to the nurse the possible
b. Encourage physical activity.
diagnosis of posttraumatic stress disorder? Select all
c. Encourage low, deep breathing.
that apply.
d. Reduce external stimuli.
e. Teach coping measures a) "I'm afraid of spiders."
b) "I keep reliving the robbery." 8. Which are characteristics of the termination stage of
c) "I see his face everywhere I go." group development? Select all that apply.
d) "I don't want anything to eat now."
a) The group evaluates the experience.
e) "I might have died over a few dollars in my
b) The real work of the group is accomplished.
pocket."
c) Group interaction involves superficial
f) "I have to wash my hands over and over again
conversation.
many times."
d) Group members become acquainted with each
4. A client with anorexia nervosa is a member of a other.
predischarge support group. The client verbalizes that e) Some structuring of group norms, roles, and
she would like to buy some new clothes, but her responsibilities takes place.
finances are limited. Group members have brought f) The group explores members' feelings about
some used clothes to the client to replace the client's the group and the impending separation.
old clothes. The client believes that the new clothes
9. The nurse employed in a mental health unit of a
were much too tight and has reduced her calorie intake
hospital is the leader of a group psychotherapy session.
to 800 calories daily. How should the nurse evaluate
What is the nurse's role during the termination stage of
this behavior?
group development?
a) Normal behavior
a) Acknowledging that the group has identified
b) Evidence of the client's disturbed body image
goals
c) Regression as the client is moving toward the
b) Encouraging the accomplishment of the group's
community
work
d) Indicative of the client's ambivalence about
c) Acknowledging the contributions of each group
hospital discharge
member
5. A client is unwilling to go out of the house for fear of d) Encouraging members to become acquainted
"making a fool of myself in public." Because of this fear, with one another
the client remains homebound. Based on these data,
10. Which statement demonstrates the best
which mental health disorder is the client experiencing?
understanding of the nurse's role regarding ensuring
a) Agoraphobia that each client's rights are respected?
b) Social phobia
a) "Autonomy is the fundamental right of each
c) Claustrophobia
and every client."
d) Hypochondriasis
b) "A client's rights are guaranteed by both state
6. A client is admitted to the mental health unit with a and federal laws."
diagnosis of depression. The nurse should develop a c) "Being respectful and concerned will ensure
plan of care for the client that includes which that I'm attentive to my client's rights."
intervention? d) "Regardless of the client's condition, all nurses
have the duty to respect client rights."
a) Encouraging quiet reading and writing for the
first few days 11. The nurse is working with a client who despite
b) Identification of physical activities that will making a heroic effort was unable to rescue a neighbor
provide exercise trapped in a house fire. Which client focused action
c) No socializing activities, until the client asks to should the nurse engage in during the working phase of
participate in milieu the nurse-client relationship?
d) A structured program of activities in which the
a) Exploring the client's ability to function
client can participate
b) Exploring the client's potential for self-harm
7. When a client is admitted to an inpatient mental c) Inquiring about the client's perception or
health unit with the diagnosis of anorexia nervosa, a appraisal of why the rescue was unsuccessful
cognitive behavioral approach is used as part of the d) Inquiring about and examining the client's
treatment plan. The nurse understands that which is feelings for any that may block adaptive coping
the purpose of this approach?
12. A client's unresolved feelings related to loss would
a) Providing a supportive environment be most likely observed during which phase of the
b) Examining intrapsychic conflicts and past issues therapeutic nurse-client relationship?
c) Emphasizing social interaction with clients who
a) Trusting
withdraw
b) Working
d) Helping the client to examine dysfunctional
c) Orientation
thoughts and beliefs
d) Termination
13. The nurse in the mental health unit recognizes c. Suggest that patients talk to vocational
which as being therapeutic communication techniques? specialists for additional training.
Select all that apply. d. Establish a therapeutic relationship with patients
and encourage participation.
a) Restating
b) Listening 3. A patient with a diagnosis of hypochondriasis has
c) Asking the client, "Why?" made multiple clinic visits and undergone diagnostic
d) Maintaining neutral responses tests for “cancer,” with no evidence of organic disease.
e) Providing acknowledgment and feedback Today he declares, “I have a brain tumor. I can feel it
f) Giving advice and approval or disapproval growing. My
appointment is tomorrow, but I can’t wait!” What is the
14. The nurse is providing care to a client admitted to most therapeutic response?
the hospital with a diagnosis of acute anxiety disorder. a. Present reality: “Sir, you have been seen many
The client says to the nurse, "I have a secret that I want times in this clinic and had many diagnostic tests.
to tell you. You won't tell anyone about it, will you?" The results have always been negative.”
What is the most appropriate nursing response? b. Encourage expression of feelings: “Let me spend
some time with you. Tell me about what you are
a) "No, I won't tell anyone." feeling and why you think you have a brain
b) "I cannot promise to keep a secret." tumor.”
c) "It depends on what the secret is about." c. Set boundaries: “Sir, I will take your vital signs,
d) "If you tell me the secret, I may need to but then I am going to call your case manager so
document it." that you can discuss the scheduled
appointment.”
15. A client diagnosed with terminal cancer says to the d. Respect the patient's wishes: “Sir, sit down and I
nurse, "I'm going to die, and I wish my family would will make sure that you see the health care
stop hoping for a cure! I get so angry when they carry provider right away. Don’t worry; we will take
on like this. After all, I'm the one who's dying." Which care of you.”
response by the nurse is therapeutic?
4. A patient who was recently diagnosed with
a) "Have you shared your feelings with your conversion disorder is experiencing a sudden loss of
family?" vision after witnessing a violent fight between her
b) "I think we should talk more about your anger husband and adult-age son. What is the priority
with your family." therapeutic approach to use with this patient?
c) "You're feeling angry that your family continues a. Reassure her that her blindness is temporary and
to hope for you to be cured?" will resolve with time.
d) "You are probably very depressed, which is b. Gently point out that she seems to be able to see
understandable with such a diagnosis." well enough to function independently.
c. Encourage expression of feelings and link
emotional trauma to the blindness.
d. Teach ways to cope with blindness, such as
Psychiatric Nursing Exam methodically arranging personal items.
1. The nurse is working at a community clinic that
specializes in assisting patients who need medication 5. The charge nurse is reviewing the assignment sheet
and therapy for mental health disorders. Which patient for an acute psychiatric
is the most likely candidate for depot antipsychotic unit. Which experienced team member should be
therapy? reassigned?
a. Older man with psychosis secondary to dementia a. Male LVN assigned to an older male patient with
who lives with his daughter chronic depression and excessive rumination
b. Homeless veteran with schizophrenia who b. Young male psychiatric nursing assistant
occasional sleeps in a nearby shelter assigned to a female adolescent with anorexia
c. Housewife with bipolar disorder who is prone to nervosa
psychotic features during the manic phase c. Female RN assigned to a newly admitted female
d. Student with recently diagnosed schizophrenia patient who has command hallucinations and
who lives at home with his parents delusions of persecution
2. The nurse is caring for patients who have d. Older female RN with medical-surgical
schizophrenia. In addition to medication, experience assigned to a male patient with
multidisciplinary nondrug therapies are available. What Alzheimer disease
is the nurse's most important role in helping the
patients to benefit from this comprehensive approach? 6. The nurse arrives home and finds that a neighbor's
a. Help identify patients who would benefit from (Jane’s) house is on fire. A
conventional psychotherapy. fireman is physically restraining Jane as she screams and
b. Refer patients to a psychiatric nurse specialist for thrashes around to
education about the disease. get free to run back into the house. What is the nurse's
best action?
a. Make eye contact and encourage Jane to 11. The nurse is talking to the primary caregiver of
verbalize feelings. Martha, who was diagnosed 8 years ago with Alzheimer
b. Physically restrain Jane so that the fireman can disease. The caregiver says, “We love Martha, but my
resume his job. daughter needs help with her kids, and my husband's
c. Use a firm tone of voice and give Jane simple health is poor. I really need help.” Which member of the
commands. health care team should the nurse consult first?
d. Use a gentle persuading tone and ask Jane to be a. Health care provider to review long-term
calm. prognosis and new treatments for Alzheimer
disease
7. There is a patient on the medical-surgical unit who b. Psychiatric clinical nurse specialist to design
has been there for several behavioral modification therapies for Martha
months. He is hostile, rude, and belligerent, and no one c. Clinical psychologist to assess for major
likes to interact with depression and need for treatment for the
him. How should the charge nurse handle the caregiver
assignment? d. Social worker to identify and arrange placement
a. Rotate the assignment schedule so that no one for Martha in an acceptable nursing home
has to care for him more than once or twice a
week. 12. The patient has a panic disorder, and it appears that
b. Pair a float nurse and a nursing student and he is having some problems controlling his anxiety.
assign the patient to that team because they will Which symptoms are cause for greatest concern?
have a fresh perspective toward the patient. a. His heart rate is increased, and he reports chest
c. Identify two or three experienced nurses as tightness.
primary caregivers and develop a plan that b. He demonstrates tachypnea and carpopedal
includes psychosocial interventions. spasms.
d. Assign self as primary caregiver and role-model c. He is pacing to and fro and pounding his fists
how patients should be treated. together.
d. He is muttering to himself and is easily startled.
8. The charge nurse is reviewing medication
prescriptions for several patients 13. The nurse is interviewing a patient with suicidal
on the acute psychiatric unit. Which prescription is the ideations and a history of
nurse most likely to major depression. Which comment is cause for greatest
question? concern?
a. Fluoxetine for a middle-aged patient with a. “I have had problems with depression most of
depression my adult life.”
b. Chlorpromazine for a young patient with b. “My father and my brother both committed
schizophrenia suicide.”
c. Loxapine for an older adult patient with c. “My wife is having health problems, and she
dementia and psychosis relies on me.”
d. Lorazepam for a young patient with generalized d. “I am afraid to kill myself, and I wished I had
anxiety disorder more courage.”

9. A patient diagnosed with paranoid schizophrenia tells 14. A patient comes into the walk-in clinic and tells the
the nurse that, “Dr. nurse that he would like
Smith has killed several other patients, and now he is to be admitted to an alcohol rehabilitation program.
trying to kill me.” What Which question is the
is the best response? most important to ask?
a. “I have worked here a long time. No one has a. “What made you decide to enter a program at
died. You are safe here.” this time?”
b. “What has Dr. Smith done to make you think he b. “How much alcohol do you usually consume in a
would like to kill you?” day?”
c. “All of the staff, including Dr. Smith, are here to c. “When was the last time you had a drink?”
ensure your safety.” d. “Have you been in a rehabilitation program
d. “Whenever you are concerned or nervous, talk before?”
to me or any of the nurses.”
15. The nurse is working with a health care provider
10. A nursing student reports to the nurse that he has who recently started treating patients with depression.
observed several types of behavior among the patients. Which action by the provider would prompt the nurse
Which patient needs priority assessment? to intervene?
a. A patient who is having command hallucinations a. Tells patient and family that it may take 4 to 8
b. A patient who is demonstrating clang weeks before the antidepressant medication
associations begins to relieve symptoms
c. A patient who is verbalizing ideas of reference b. Prescribes 3 months of antidepressants for a
d. A patient who is using neologisms patient newly diagnosed with depression and
gives a 3-month follow-up appointment
c. Instructs the patient that the initial dose is low a. Psychiatric nursing assistant uses a quick-
but will gradually be increased to reach a release knot to tie restraints.
maintenance dosage b. Health care provider (HCP) secures the restraint
d. Tells the patient and the family to watch for and to the side rail.
immediately report anxiety, agitation, c. RN checks the pulses distal to the restraints.
irritability, or suicidal thoughts d. LPN/LVN explains to the patient why he is being
restrained.
16. A patient on the acute psychiatric unit develops
neuroleptic malignant syndrome. Which task should be 21. A well-known celebrity is admitted to the psychiatric
delegated to the psychiatric nursing assistant (PNA)? unit. Several RNs from other units drop by and express
a. Wiping the patient's body with cool moist an interest in seeing the patient. What is the best
towels response?
b. Monitoring and interpreting vital signs every 15 a. “Please be discreet and do not interrupt the
minutes work flow.”
c. Attaching the patient to the electrocardiogram b. “How did you find out that the patient was
(ECG) monitor admitted to this unit?”
d. Transporting the patient to the medical c. “Please wait. I need to call the nursing
intensive care unit supervisor about this request.”
d. “I’m sorry; the patient has asked that only
17. A newly graduated nurse has just started working at family be allowed to visit.”
the acute psychiatric unit. Which patient would be the
best to assign to this nurse? 22. An LPN/LVN complains to the charge nurse that she
a. Patient who is frequently admitted for is always assigned to
borderline personality disorder and suicidal the same patient with chronic depression. What should
gesture the charge nurse do?
b. Patient admitted yesterday for disorganized 1. Look at the assignment sheet and see if there is any
schizophrenia and psychosis way to switch
c. Patient newly admitted to determine assignments with another LPN/LVN.
differential diagnosis of depression, dementia, 423
or delirium 2. Tell her to care for the patient today but that her
d. Patient newly diagnosed with major depression request will be considered
and rumination about loss and suicide for future assignments.
3. Remind her that continuity of care and patient-
18. Which task can be delegated to a medical-surgical centered care are the
unlicensed assistive personnel (UAP) who has been primary goals.
temporarily floated to the acute psychiatric unit to 4. Explain that patients with chronic conditions are
help? more likely to fall under
a. Performing one-to-one observation of a patient the LPN/LVN scope of practice.
who is suicidal
b. Assisting the occupational therapist to conduct 23. Which person is displaying behaviors that most
a craft class strongly suggest the need for additional screening for
c. Accompanying an older adult patient who possible substance abuse?
wanders on a walk outside a. Person with cancer progressively needs more
d. Assisting the medication nurse who is having pain medication to achieve relief
problems with a patient b. College student reports occasionally smoking
marijuana during semester break
19. The nurse has identified a patient who may be a c. Stay-at-home mom reports drinking while her
candidate for substance addiction treatment. Which kids are in school and after they go to bed
health care team member should the nurse contact to d. Person with a fractured leg reports taking
increase the likelihood of a successful long-term opioids and tapering off when pain subsides
outcome?
a. Call a social worker who can locate an 24. The emergency department (ED) nurse is calling to
immediately available treatment program. report on a patient who will be admitted to the acute
b. Call admissions to obtain patient's voluntary psychiatric unit. He has a history of bipolar disorder and
consent to enter treatment program. was in an altercation that resulted in the death of
c. Consult a pharmacist about medication therapy another. He has contusions, abrasions, and minor
to counter addiction. lacerations. What is the priority question that the
d. Contact the health care provider to initiate receiving nurse should ask?
admission to a medical detoxification unit. a. “When will the patient be transferred?”
b. “Will a police officer be with him while he is on
20. The team must apply restraints to a combative the unit?”
patient to prevent harm to c. “Why isn’t the patient being admitted to the
others or to self. Which action requires the charge trauma unit?”
nurse's intervention?
d. “What is the patient's current mood and c. A patient who is taking loxapine and has a
behavior?” protruding tongue with lip smacking and spastic
facial distortions
25. A patient needs clonazepam 0.25 mg PO. The d. A patient who is taking clozapine and reports a
pharmacy delivers lorazepam 2-mg tablets. A nursing sore throat, fever, malaise, and flulike
student asks the nurse if clonazepam and lorazepam are symptoms that began about 6 weeks ago after
interchangeable or if they are the same drug. Place the starting the new antipsychotic medication;
following steps in the correct sequence so that the white blood cell count is 2000/mm3 (2.0 ×
nurse can teach the nursing student how to prevent 109/L) _____, _____, _____, _____
medication errors.
a. Advise the pharmacy of any corrections as a,d,c,b (1,4,3,2)
appropriate.
b. Recognize that “look-alike, sound-alike” drugs 29. The patient tells the nurse that he drinks 3 or 4
increase the chances of error. servings of alcohol every day. He also reports frequently
c. Consult a medication book to verify the purpose taking acetaminophen for stress-related headaches.
of the drugs and generic and brand names. Based on this information, which laboratory test results
d. Check the original medication order to verify are the most important to follow up on?
what was prescribed. a. Renal function tests
e. Write an incident report, as appropriate, if a b. Liver function tests
system error is occurring. c. Cardiac enzymes
f. Call the health care provider (HCP) for d. Serum electrolytes
clarification of the order as appropriate.
_____, _____, _____, _____, _____, _____ 30. The nurse is reviewing the principle of “least
b, d, c, f, a, e (2,4,3,6,1,5) restrictive” interventions with the staff. Place the
following interventions in the correct order, with 1
26. The vital signs of a 23-year-old man with no known being the least restrictive and 6 being the most
health problems are unexpectedly abnormal. When the restrictive.
nurse mentions the vital signs, he says, “Well, I was a a. Escort the patient to a quiet room for a time
little nervous, so I smoked four or five cigarettes right out.
before I came into the clinic.” Which vital signs would b. Restrain the patient's arms and legs with soft
be consistent with the patient's use of cigarettes? cloth restraints.
a. Blood pressure of 90/60 mm Hg; pulse of 60 c. Verbally instruct the patient to stop the
beats/min unacceptable behavior (i.e., yelling, arguing)
b. Temperature of 100.6°F (38.1°C); respirations of and move to another part of the day room.
40 breaths/min d. Accompany the patient out into the garden
c. Blood pressure of 140/90 mm Hg; pulse of 120 courtyard.
beats/min e. Restrain the patient's upper extremities with
d. Temperature of 97.4°F (36.3°C); respirations of wrist restraints.
12 breaths/min f. Place the patient in isolation room with
psychiatric nursing assistant observing.
27. A patient is displaying muscle spasms of the tongue, _____, _____, _____, _____, _____, _____
face, and neck, and his eyes are locked in an upward
gaze. He has been prescribed haloperidol. What is the c, d, a, f, e, b (3, 4, 1, 6, 5, 2)
priority action by the nurse?
a. Maintain eye contact and stay with him until 31. The home health nurse is reviewing the plan of care
the spasms pass. for a 62-year-old patient who lives with his wife at
b. Place the patient on aspiration precautions until home. The health care provider (HCP) recently
the spasms subside. prescribed rivastigmine twice daily for the patient.
c. Obtain an order for intramuscular or IV Based on this information, what additional assessment
diphenhydramine. would the nurse plan to perform first?
d. Obtain an order for and administer an a. Assess for psychotic features, such as
antiseizure medication. hallucinations.
b. Perform a comprehensive pain assessment.
28. Several patients are taking antipsychotic c. Assess for cognitive deficits and memory loss.
medications and are having medication side effects. d. Observe for fine and gross motor deficits.
Place the following patients in priority order for
additional assessment and appropriate interventions, 32. Which behavior would be the most problematic and
with 1 being the most critical and 4 being the least. require vigilance to prevent danger to self or others?
a. A patient who is taking trifluoperazine and has a a. Avolition
temperature of 103.6°F (39.8°C) with b. Echolalia
tachycardia, muscular rigidity, and dysphagia c. Motor agitation
b. A patient who is taking fluphenazine and has d. Stupor
dry mouth and dry eyes, urinary hesitancy,
constipation, and photosensitivity
33. A patient comes in to the clinic with nausea, problem. Based on the ethical principle of negligence,
constipation, and “excruciating stomach pain.” Over a what should Nurse A do first?
period of several years, this patient has come in two or a. Talk to Nurse B and give counsel about the
three times a month with the same report, but multiple ethical issues of taking patients’ medications.
diagnostic tests have consistently yielded negative b. Continue to assess Nurse B's behavior for other
results for physical disorders. What is the priority signs and symptoms of abuse.
nursing intervention for this patient? c. Work closely with Nurse B to give support and
a. Advocate for the patient to have a psychiatric help to reduce stress of workload.
consultation. d. Report facts to the nursing supervisor to include
b. Make appointment as soon as possible with date, time, circumstance, and behaviors.
same health care provider (HCP) for continuity
of care. 38. A male-to-female transgender patient (transwoman)
c. Perform a physical assessment to identify any is admitted to an acute care psychiatric unit for
physical abnormalities. depression and suicidal ideations. On her arrival, several
d. Assess for concurrent symptoms of depression other patients display suspicion and contempt and
or anxiety. verbal harassment is directed toward the woman. What
should the charge nurse do first?
34. An older man was admitted for palliative care of a. Isolate the patient and explain that the action is
terminal pancreatic cancer. His wife stated, “We don’t meant for her safety and privacy.
want hospice; he wants treatment.” The patient b. Make a general announcement to all patients
requested discharge and home health visits. Several and staff that bullying will not be tolerated.
hours after discharge, the man committed suicide with a c. Assess the patient's reaction to the comments
gun. Which people should participate in a root cause and nonverbal behaviors.
analysis of this sentinel event? Select all that apply. d. Gently suggest that the patient could
a. The wife and all immediate family members temporarily adopt natal gender appearance.
b. Only the health care provider (HCP) who
discharged the patient 39. You arrive home and find that the house of your
c. Any nurse who cared for the patient during neighbor (Jane) is on fire. A fireman is physically
hospitalization restraining her from running back into the house. What
d. The case manager who arranged home visits for is the best response?
the patient a. Jane, come and sit in my house until this is over
e. Only the nurse who discharged the patient with
f. All HCPs who were involved in the care of the b. Jane, calm down and let the fireman do his job
patient c. Jane, look at me and hold my hand
d. Jane, tell me why you are struggling so hard
35. An adolescent girl is admitted to the medical- 40. An LPN complains to you that she is always assigned
surgical unit for diagnostic evaluation and nutritional to the same patient with chronic depression. What
support related to anorexia nervosa. She is mildly should you do?
dehydrated, her potassium level is 3.5 mEq/L (3.5 a. Look at the assignment sheet and see if there is
mmol/L), and she has experienced weight loss of more any way to switch assignments with another
than 25% within the past 3 months. What is the primary LPN
collaborative goal? b. Tell her to care for the patient today, but that
a. Assist her to increase feelings of control. you will remember the request for future
b. Decrease power struggles over eating. assignments
c. Resolve dysfunctional family roles. c. Remind her that continuity of care and patient-
d. Restore normal nutrition and weight. centered care are the primary goals
d. Explain that patients with chronic conditions are
36. In caring for a patient who is admitted to a medical more likely to fall under the LPN scope of
surgical unit for treatment of anorexia nervosa, which practice
task can be delegated to unlicensed assistive personnel
(UAP)?
a. Sitting with the patient during meals and for 1
to 1½ hours after meals
b. Observing for and reporting ritualistic behaviors
related to food
c. Obtaining special food for the patient when she
requests it
d. Weighing the patient daily and reinforcing that
she is underweight

37. Nurse B frequently asks to be assigned to care for


patients who require opioids for pain; drug counts
involving Nurse B frequently show discrepancies. Nurse
A suspects that Nurse B may have a substance abuse
c. Why don't we go to the game room and see
what is happening there today?
d. I do not hear anything, but you cannot leave the
hospital

Psychiatric Nursing Exam 5. Which of the following are ways nurses create
Personality disorder barriers to therapeutic communication? Select all
1. A client with obsessive-compulsive disorder (OCD) is that apply.
in the hospital after having been diagnosed with a a. Using clichés
chronic illness. Which best describes what the nurse b. Active listening
would see in this client while trying to provide care? c. Changing the subject
a) Anxiety because they have no control over their d. Making eye contact
illness e. Offering support
b) Questioning so much that the nurse has
difficulty leaving the room 6. A client has been diagnosed with antisocial
c) Requesting help with minor tasks and personality disorder and becomes very angry with
portraying a helpless role the nurse and is starting to become aggressive.
d) Refusing to trust the nurse and acting paranoid Which principle of management would most likely
Rationale: (A)- A person with an obsessive-compulsive be implemented in this situation?
disorder (OCD) may feel frustrated with their diagnosis when a. Remain calm and put space between the nurse
they are unable to control their health or symptoms. The and the client
nurse can best help this client by providing many details b. change the client that the nurse is competent in
about his treatment and helping to have a small element of self-defense
control in certain areas.
c. Promise the client that he will not be harmed if
he cooperates
2. A 23-year-old client is being seen for treatment
d. Have the client stay in a private room that can
after purposely cutting the skin on her arm. The
be locked
client has impulsive behavior, is angry, and has been
losing friends because of intense and needy
7. Piaget's stages are: stage: birth to 18-24 months
behavior. Which type of personality disorder does
a. Preoperational stage: 2 to 7 years
this best describe?
b. Concrete operational stage: 7 to 11 years
a) Borderline personality disorder
c. Formal operational stage: ages 12 and up
b) Antisocial personality disorder
c) Histrionic personality disorder
d) Schizotypal personality disorder
Rationale: (A)- A borderline personality disorder is
Psychiatric Nursing Exam
manifested by varying degrees of moodiness, neediness, and Exam for Schizophrenia
self-image. The client with a borderline personality disorder 1. Nurse Dorothy is evaluating care of a client with
has mood instability and is often emotionally unstable in his schizophrenia, the nurse should keep which point in
or her behavior and relationships. The condition can also lead mind?
to acute psychotic episodes that can be dangerous, such as
when a client purposely self-mutilates. a) Frequent reassessment is needed and is based
on the client’s response to treatment.
3. A 23-year-old client is being seen for treatment b) The family does not need to be included in the
after purposely cutting the skin on her arm. The care because the client is an adult.
client has impulsive behavior, is angry, and has been c) The client is too ill to learn about his illness.
losing friends because of intense and needy d) Relapse is not an issue for a client with
behavior. Which type of personality disorder does schizophrenia.
this best describe?
a. Borderline personality disorder 2. Gio told his nurse that the FBI is monitoring and
b. Antisocial personality disorder recording his every movement and that microphones
c. Histrionic personality disorder have been plated in the unit walls. Which action would
d. Schizotypal personality disorder be the most therapeutic response?

4. A client with schizoaffective disorder has been a) Confront the delusional material directly by
admitted to the inpatient mental health center of telling Gio that this simply is not so.
the hospital. The client tells the nurse that he hears b) Tell Gio that this must seem frightening to him
voices telling him to leave the hospital. Which but that you believe he is safe here.
response from the nurse is best? c) Tell Gio to wait and talk about these beliefs in
a. I do not hear anything, but I can see how you his one-on-one counselling sessions.
may feel worried about being in the hospital d) Isolate Gio when he begins to talk about these
right now beliefs.
b. You know that those voices aren't real, don't
you?
3. Which of the following client behaviors documented
in Gio’s chart would validate the nursing diagnosis of a) Conclusive evidence indicates a specific gene
Risk for other-directed violence? transmits the disorder.
b) Incidence of this disorder is variable in all
a) Gio’s description of being endowed with families.
superpowers c) There is a little evidence that genes play a role
b) Frequent angry outburst noted toward peers in transmission.
and staff d) Genetic factors can increase the vulnerability
c) Refusal to eat cafeteria food for this disorder.
d) Refusal to join in group activities
9. Ramsay is diagnosed with schizophrenia paranoid
4. Nurse Winona educates the family about symptom type and is admitted in the psychiatric unit of Medical
management for when the schizophrenic client Center. Which of the following nursing interventions
becomes upset or anxious. Which of the following would be most appropriate?
would Nurse Winona state is helpful?
a) Establishing a non-demanding relationship
a) Call the therapist to request a medication b) Encouraging involvement in group activities
change. c) Spending more time with Ramsay
b) Encourage the use of learned relaxation d) Waiting until Ramsay initiates interaction
techniques.
c) Request that the client be hospitalized until the 10. A client tells the nurse that psychotropic medicines
crisis is over. are dangerous and refuses to take them. Which
d) Wait before the anxiety worsens before intervention should the nurse use first?
intervening.
a) Ask the client about any previous problems with
5. Drogo who has had auditory hallucinations for many psychotropic medications.
years tells Nurse Khally that the voices prevents his b) Ask the client if an injection is preferable.
participation in a social skills training program at the c) Insist that the client take medication as
community health center. Which intervention is most prescribed.
appropriate? d) Withhold the medication until client is less
suspicious.
a) Let Drogo analyze the content of the voices.
b) Advise Drogo to participate in the program 11. Upon Sam’s admission for acute psychiatric
when the voices cease. hospitalization, Nurse Jona documents the following:
c) Advise Drogo to take his medications as Client refuses to bathe or dress, remains in room most
prescribed. of the day, speaks infrequently to peers or staff. Which
d) Teach Drogo to use thought stopping nursing diagnosis would be the priority at this time?
techniques.
a) Anxiety
6. Cersei is diagnosed as having disorganized b) Decisional conflict
schizophrenia. Which behaviors would Nurse Sansa c) Self-care deficit
most likely assess in the client? d) Social isolation

a) Absence of acute symptoms, impaired role 12. Which statement is correct about a 25-year-old
function client with newly diagnosed schizophrenia?
b) Extreme social withdrawal, odd mannerisms
and behavior a) Age of onset is typical for schizophrenia.
c) Psychomotor immobility; presence of waxy b) Age of onset is later than usual for
flexibility schizophrenia.
d) Suspiciousness toward others, increased c) Age of onset is earlier than usual for
hostility schizophrenia.
d) Age of onset follows no predictable pattern in
7. Jaime has a diagnosis of schizophrenia with negative schizophrenia.
symptoms. In planning care for the client, Nurse
Brienne would anticipate a problem with: 13. Which factor is associated with increased risk for
schizophrenia?
a) auditory hallucinations.
b) bizarre behaviors. a) Alcoholism
c) ideas of reference. b) Adolescent pregnancy
d) motivation for activities. c) Overcrowded schools
d) Poverty
8. The family of a schizophrenic client asks the nurse if
there is a genetic cause of this disorder. To answer the 14. Nurse Arya assesses for evidence of positive
family, which fact would the nurse cite? symptoms of schizophrenia in a newly admitted client.
Which of the following symptoms are considered B. Respiratory arrest
positive evidence? Select all that apply. C. Tourette syndrome
D. Retinal pigmentation
a) Anhedonia
b) Delusions 5.A client with paranoid personality disorder is admitted
c) Flat affect to a psychiatric facility. Which remark by the nurse
d) Hallucinations would best establish rapport and encourage the client
e) Loose associations to confide in the nurse?
f) Social withdrawal A. "I get upset once in a while, too."
B. "I know just how you feel. I'd feel the same way
15. A client with schizophrenia is referred for in your situation."
psychosocial rehabilitation. Which of the following are C. "I worry, too, when I think people are talking
typical of this type of program? Select all that apply. about me."
D. "At times, it's normal not to trust anyone."
a) Analyzing family issues and past problems
b) Developing social skills and supports 6. How soon after chlorpromazine (Thorazine)
c) Learning how to live independently in a administration should the nurse expect to see a client's
community delusional thoughts and hallucinations eliminated?
d) Learning job skills for employment A. Several minutes
e) Treating family members affected by the illness B. Several hours
f) Participating in in-depth psychoanalytical C. Several days
counselling D. Several weeks

Psychiatric Nursing Exam 7.A client is about to be discharged with a prescription


SCHIZOPHRENIA QUESTIONS for the antipsychotic agent haloperidol (Haldol), 10 mg
1. The nurse is caring for a client with schizophrenia. by mouth twice per day. During a discharge teaching
Which of the following outcomes is the least desirable? session, the nurse should provide which instruction to
A. The client spends more time by himself. the client?
B. The client doesn't engage in delusional thinking. A. Take the medication 1 hour before a meal.
C. The client doesn't harm himself or others. B. Decrease the dosage if signs of illness decrease.
D. The client demonstrates the ability to meet his C. Apply a sunscreen before being exposed to the
own self-care needs. sun.
D. Increase the dosage up to 50 mg twice per day
2. The nurse formulates a nursing diagnosis of Impaired if signs of illness don't decrease.
verbal communication for a client with schizotypal
personality disorder. Based on this nursing diagnosis,
which nursing intervention is most appropriate? 8. A client with paranoid schizophrenia repeatedly uses
A. Helping the client to participate in social profanity during an activity therapy session. Which
interactions response by the nurse would be most appropriate?
B. Establishing a one-on-one relationship with the A. "Your behavior won't be tolerated. Go to your
client room immediately."
C. Establishing alternative forms of B. "You're just doing this to get back at me for
communication making you come to therapy."
D. Allowing the client to decide when he wants to C. "Your cursing is interrupting the activity. Take
participate in verbal communication with the time out in your room for 10 minutes."
nurse D. "I'm disappointed in you. You can't control
yourself even for a few minutes."
3. Since admission 4 days ago, a client has refused to
take a shower, stating, "There are poison crystals 9.Which of the following is one of the advantages of the
hidden in the shower head. They'll kill me if I take a newer antipsychotic medication risperidone
shower." Which nursing action is most appropriate? (Risperdal)?
A. Dismantling the showerhead and showing the A. The absence of anticholinergic effects
client that there is nothing in it B. A lower incidence of extrapyramidal effects
B. Explaining that other clients are complaining C. Photosensitivity and sedation
about the client's body odor D. No incidence of neuroleptic malignant
C. Asking a security officer to assist in giving the syndrome
client a shower
D. Accepting these fears and allowing the client to 10.The etiology of schizophrenia is best described by:
take a sponge bath A. genetics due to a faulty dopamine receptor.
B. environmental factors and poor parenting.
4. Drug therapy with thioridazine (Mellaril) shouldn't C. structural and neurobiological factors.
exceed a daily dose of 800 mg to prevent which adverse D. a combination of biological, psychological, and
reaction? environmental factors.
A. Hypertension
11.A client with schizophrenia who receives
fluphenazine (Prolixin) develops pseudoparkinsonism 17.The nurse is assigned to a client with catatonic
and akinesia. What drug would the nurse administer to schizophrenia. Which intervention should the nurse
minimize extrapyramidal symptoms? include in the client's plan of care?
A. benztropine (Cogentin) A. Meeting all of the client's physical needs
B. dantrolene (Dantrium) B. Giving the client an opportunity to express
C. clonazepam (Klonopin) concerns
D. diazepam (Valium) C. Administering lithium carbonate (Lithonate) as
prescribed
12.A client with a diagnosis of paranoid schizophrenia D. Providing a quiet environment where the client
comments to the nurse, "How do I know what is really can be alone
in those pills?" Which of the following is the best
response? 18.A client with a history of medication noncompliance
A. Say, "You know it's your medicine." is receiving outpatient treatment for chronic
B. Allow him to open the individual wrappers of undifferentiated schizophrenia. The physician is most
the medication. likely to prescribe which medication for this client?
C. Say, "Don't worry about what is in the pills. It's A. chlorpromazine (Thorazine)
what is ordered." B. imipramine (Tofranil)
D. Ignore the comment because it's probably a C. lithium carbonate (Lithane)
joke. D. fluphenazine decanoate (Prolixin Decanoate)

13.A client tells the nurse that people from Mars are 19.Propranolol (Inderal) is used in the mental health
going to invade the earth. Which response by the nurse setting to manage which of the following conditions?
would be most therapeutic? A. Antipsychotic-induced akathisia and anxiety
A. "That must be frightening to you. Can you tell B. The manic phase of bipolar illness as a mood
me how you feel about it?" stabilizer
B. "There are no people living on Mars." C. Delusions for clients suffering from
C. "What do you mean when you say they're going schizophrenia
to invade the earth?" D. Obsessive-compulsive disorder (OCD) to reduce
D. "I know you believe the earth is going to be ritualistic behavior
invaded, but I don't believe that."
20.Every day for the past 2 weeks, a client with
14.A client with schizophrenia tells the nurse he hears schizophrenia stands up during group therapy and
the voices of his dead parents. To help the client ignore screams, "Get out of here right now! The elevator
the voices, the nurse should recommend that he: bombs are going to explode in 3 minutes!" The next
A. sit in a quiet, dark room and concentrate on the time this happens, how should the nurse respond?
voices. A. "Why do you think there is a bomb in the
B. listen to a personal stereo through headphones elevator?"
and sing along with the music. B. "That is the same thing you said in yesterday's
C. call a friend and discuss the voices and his session."
feelings about them. C. "I know you think there are bombs in the
D. engage in strenuous exercise. elevator, but there aren't."
D. "If you have something to say, you must do it
15.A client with schizophrenia is receiving antipsychotic according to our group rules."
medication. Which nursing diagnosis may be 21.A 26-year-old client is admitted to the psychiatric
appropriate for this client? unit with acute onset of schizophrenia. His physician
A. Ineffective protection related to blood prescribes the phenothiazine chlorpromazine
dyscrasias (Thorazine), 100 mg by mouth four times per day.
B. Urinary frequency related to adverse effects of Before administering the drug, the nurse reviews the
antipsychotic medication client's medication history. Concomitant use of which
C. Risk for injury related to a severely decreased drug is likely to increase the risk of extrapyramidal
level of consciousness effects?
D. Risk for injury related to electrolyte A. guanethidine (Ismelin)
disturbances B. droperidol (Inapsine)
C. lithium carbonate (Lithonate)
16.A client with persistent, severe schizophrenia has D. alcohol
been treated with phenothiazines for the past 17 years.
Now the client's speech is garbled as a result of drug- 22.A client, age 36, with paranoid schizophrenia
induced rhythmic tongue protrusion. What is another believes the room is bugged by the Central Intelligence
name for this extrapyramidal symptom? Agency and that his roommate is a foreign spy. The
A. Dystonia client has never had a romantic relationship, has no
B. Akathisia contact with family members, and hasn't been
C. Pseudoparkinsonism employed in the last 14 years. Based on Erikson's
D. Tardive dyskinesia
theories, the nurse should recognize that this client is in
which stage of psychosocial development?
A. Autonomy versus shame and doubt 28.Which information is most important for the nurse
B. Generativity versus stagnation to include in a teaching plan for a schizophrenic client
C. Integrity versus despair taking clozapine (Clozaril)?
D. Trust versus mistrust A. Monthly blood tests will be necessary.
B. Report a sore throat or fever to the physician
23.During a group therapy session in the psychiatric immediately.
unit, a client constantly interrupts with impulsive C. Blood pressure must be monitored for
behavior and exaggerated stories that cast her as a hero hypertension.
or princess. She also manipulates the group with D. Stop the medication when symptoms subside.
attention-seeking behaviors, such as sexual comments
and angry outbursts. The nurse realizes that these
behaviors are typical of: 29.Important teaching for clients receiving antipsychotic
A. paranoid personality disorder. medication such as haloperidol (Haldol) includes which
B. avoidant personality disorder. of the following instructions?
C. histrionic personality disorder. A. Use sunscreen because of photosensitivity.
D. borderline personality disorder. B. Take the antipsychotic medication with food.
C. Have routine blood tests to determine levels of the
medication.
24.The nurse is teaching a psychiatric client about her D. Abstain from eating aged cheese.
prescribed drugs, chlorpromazine and benztropine.
Why is benztropine administered? 30.Positive symptoms of schizophrenia include which of
A. To reduce psychotic symptoms the following?
B. To reduce extrapyramidal symptoms A. Hallucinations, delusions, and disorganized thinking
C. To control nausea and vomiting B. Somatic delusions, echolalia, and a flat affect
D. To relieve anxiety C. Waxy flexibility, alogia, and apathy
D. Flat affect, avolition, and anhedonia
25.A client is admitted to the psychiatric unit with a
tentative diagnosis of psychosis. Her physician
prescribes the phenothiazine thioridazine (Mellaril) 50 31.A client with chronic schizophrenia receives 20 mg of
mg by mouth three times per day. Phenothiazines differ fluphenazine decanoate (Prolixin Decanoate) by I.M.
from central nervous system (CNS) depressants in their injection. Three days later, the client has muscle
sedative effects by producing: contractions that contort the neck. This client is
A. deeper sleep than CNS depressants. exhibiting which extrapyramidal reaction?
B. greater sedation than CNS depressants. A. Dystonia
C. a calming effect from which the client is easily B. Akinesia
aroused. C. Akathisia
D. more prolonged sedative effects, making the D Tardive dyskinesia
client more difficult to arouse.

32.Hormonal effects of the antipsychotic medications


26.A woman is admitted to the psychiatric emergency include which of the following?
department. Her significant other reports that she has A. Retrograde ejaculation and gynecomastia
difficulty sleeping, has poor judgment, and is incoherent B. Dysmenorrhea and increased vaginal bleeding
at times. The client's speech is rapid and loose. She C .Polydipsia and dysmenorrhea
reports being a special messenger from the Messiah. D. Akinesia and dysphasia
She has a history of depressed mood for which she has
been taking an antidepressant. The nurse suspects 33.A client is unable to get out of bed and get dressed
which diagnosis? unless the nurse prompts every step. This is an example
A. Schizophrenia of which behavior?
B. Paranoid personality A. Word salad
C. Bipolar illness B. Tangential
D. Obsessive-compulsive disorder (OCD) C. Perseveration
D. Avolition

27.A client with paranoid schizophrenia is admitted to 33.An agitated and incoherent client, age 29, comes to
the psychiatric unit of a hospital. Nursing assessment the emergency department with complaints of visual
should include careful observation of the client's: and auditory hallucinations. The history reveals that the
A. thinking, perceiving, and decision-making skills. client was hospitalized for paranoid schizophrenia from
B. verbal and nonverbal communication ages 20 to 21. The physician prescribes haloperidol
processes. (Haldol), 5 mg I.M. The nurse understands that this drug
C. affect and behavior. is used for this client to treat:
A. D psychomotor activity. A. dyskinesia.
B. dementia. 39.A client begins clozapine (Clozaril) therapy after
C. psychosis. several other antipsychotic agents fail to relieve her
D. tardive dyskinesia. psychotic symptoms. The nurse instructs her to return
for weekly white blood cell (WBC) counts to assess for
34.Yesterday, a client with schizophrenia began which adverse reaction?
treatment with haloperidol (Haldol). Today, the nurse A. Hepatitis
notices that the client is holding his head to one side B. Infection
and complaining of neck and jaw spasms. What should C. Granulocytopenia
the nurse do? D Systemic dermatitis
A. Assume that the client is posturing.
B. Tell the client to lie down and relax. 40.Which non-antipsychotic medication is used to treat
C. Evaluate the client for adverse reactions to some clients with schizoaffective disorder?
haloperidol. A. phenelzine (Nardil)
D. Put the client on the list for the physician to see B. chlordiazepoxide (Librium)
tomorrow. C. lithium carbonate (Lithane)
D. imipramine (Tofranil)
35.A client receiving fluphenazine decanoate (Prolixin
Decanoate) therapy develops pseudoparkinsonism. The 41.A client diagnosed with schizoaffective disorder is
physician is likely to prescribe which drug to control this suffering from schizophrenia with elements of which of
extrapyramidal effect? the following disorders?
A. phenytoin (Dilantin) A. Personality disorder
B. amantadine (Symmetrel) B. Mood disorder
C. benztropine (Cogentin) C. Thought disorder
D. diphenhydramine (Benadryl) D. Amnestic disorder

42.When teaching the family of a client with


36.Important teaching for a client receiving risperidone schizophrenia, the nurse should provide which
(Risperdal) would include advising the client to: information?
A. double the dose if missed to maintain a therapeutic A. Relapse can be prevented if the client takes the
level. medication.
B. be sure to take the drug with a meal because it's very B. Support is available to help family members meet
irritating to the stomach. their own needs.
C. discontinue the drug if the client reports weight gain. C. Improvement should occur if the client has a
D. notify the physician if the client notices an increase in stimulating environment.
bruising D. Stressful family situations can precipitate a relapse in
the client
37.A client is admitted to the psychiatric hospital with a
diagnosis of catatonic schizophrenia. During the 43.A client is admitted to the psychiatric unit with active
physical examination, the client's arm remains psychosis. The physician diagnoses schizophrenia after
outstretched after the nurse obtains the pulse and ruling out several other conditions. Schizophrenia is
blood pressure, and the nurse must reposition the arm. characterized by:
This client is exhibiting: A. loss of identity and self-esteem.
A. suggestibility. B. multiple personalities and decreased self-esteem.
B. negativity. C. disturbances in affect, perception, and thought
C. waxy flexibility. content and form.
D. retardation D. persistent memory impairment and confusion.

38.A client with borderline personality disorder 44.The nurse is providing care to a client with a
becomes angry when he is told that today's catatonic type of schizophrenia who exhibits extreme
psychotherapy session with the nurse will be delayed 30 negativism. To help the client meet his basic needs, the
minutes because of an emergency. When the session nurse should:
finally begins, the client expresses anger. Which A. ask the client which activity he would prefer to do
response by the nurse would be most helpful in dealing first.
with the client's anger? B. negotiate a time when the client will perform
A. "If it had been your emergency, I would have made activities.
the other client wait." C. tell the client specifically and concisely what needs to
B. "I know it's frustrating to wait. I'm sorry this be done.
happened." D. prepare the client ahead of time for the activity
C. "You had to wait. Can we talk about how this is
making you feel right now?" 45.The nurse is caring for a client who experiences false
D. "I really care about you and I'll never let this happen sensory perceptions with no basis in reality. These
again." perceptions are known as:
A. delusions.
B. hallucinations.
C. loose associations. 3. Mrs. Mendoza is a 75-year-old client who has
D. neologisms. dementia of the Alzheimer's type and confabulates. The
nurse understands that this client:
46.The nurse is aware that antipsychotic medications
may cause which of the following adverse effects? A. Denies confusion by being jovial.
A. Increased production of insulin B. Pretends to be someone else.
B. Lower seizure threshold C. Rationalizes various behaviors.
C. Increased coagulation time D. fills in memory gaps with fantasy.
D Increased risk of heart failure
4. Which ability should Nurse Rebecca expect from a
47.A client is admitted with a diagnosis of delusions of client in the mild stage of dementia of the Alzheimer's
grandeur. This diagnosis reflects a belief that one is: type?
A. highly important or famous.
B. being persecuted. A. Remembering the daily schedule
C. connected to events unrelated to oneself. B. Recalling past events
D. responsible for the evil in the world. C. Coping the anxiety
D. Solving problems of daily living
48.A man with a 5-year history of multiple psychiatric 5. 82-year-old Mr. Robeson together with his daughter
admissions is brought to the emergency department by arrived at the medical-surgical unit for diagnostic
the police. He was found wandering the streets confirmation and management of probable delirium.
disheveled, shoeless, and confused. Based on his Which statement by the client's daughter best supports
previous medical records and current behavior, he is the diagnosis?
diagnosed with chronic undifferentiated schizophrenia.
The nurse should assign the highest priority to which A. "Maybe it's just caused by aging. This usually
nursing diagnosis? happens by age 82."
A. Anxiety B. "The changes in his behavior came on so quickly! I
B. Impaired verbal communication wasn't sure what was happening."
C. Disturbed thought processes C. "Dad just didn't seem to know what he was doing. He
D. Self-care deficit: Dressing/grooming would forget what he had for breakfast."
D. "Dad has always been so independent. He's lived
49.A client's medication order reads, "Thioridazine alone for years since mom died."
(Mellaril) 200 mg P.O. q.i.d. and 100 mg P.O. p.r.n." The
nurse should: 6. Mrs. Jordan is an elderly client diagnosed with
A. administer the medication as prescribed. Alzheimer's disease. She becomes agitated and
B. question the physician about the order. combative when a nurse approaches to help with
C. administer the order for 200 mg P.O. q.i.d. but not for morning care. The most appropriate nursing
100 mg P.O. p.r.n. intervention in this situation would be to:
D. administer the medication as prescribed but observe
the client closely for adverse effects. A. tell the client firmly that it is time to get dressed.
B. obtain assistance to restrain the client for safety.
C. remain calm and talk quietly to the client.
Dementia & delirium D. call the doctor and request an order for sedation.
1. A student nurse was asked which of the following
best describes dementia. Which of the following best 7. Which goal is a priority for a client with a DSM-IV-TR
describes the condition? diagnosis of delirium and the nursing diagnosis Acute
confusion related to recent surgery secondary to
A. Memory loss occurring as part of the natural traumatic hip fracture?
consequence of aging
B. Difficulty coping with physical and psychological A. The client will complete activities of daily living.
change B. The client will maintain safety.
C. Severe cognitive impairment that occurs rapidly C. The client will remain oriented.
D. Loss of cognitive abilities, impairing ability to perform D. The client will understand communication.
activities of daily living
8. Which of the following is not included in the care of
2. Which of the following will Nurse Dory use when plan of a client with a moderate cognitive impairment
communicating with a client who has cognitive involving dementia of the Alzheimer's type?
impairment.
A. Daily structured schedule
A. Complete explanations with multiple details B. Positive reinforcement for performing activities of
B. Pictures or gestures instead of words daily living
C. Stimulating words and phrases to capture the client's C. Stimulating environment
attention D. Use of validation techniques
D. Short words and simple sentences
9. In clients with a cognitive impairment disorder, the d) Make an appointment for the client to meet with
phenomenon of increased confusion in the early a dietitian to discuss the nutritional effects of
evening hours is called: taking this many drug

A. Aphasia. 2.A nurse is assisting a client who has undergone


B. Agnosia. electroconvulsive therapy for treatment of severe
C. Sundowning. depression. Following the procedure, the client
D. Confabulation. develops postictal agitation. Based on the nurse’s
knowledge of this condition, the nurse would expect to
10. Which of the following outcome criteria is see which of the following?
appropriate for the client with dementia? a) Hyperactivity and mania
b) Coma
A. The client will return to an adequate level of self- c) Limb contractures
functioning. d) Incoherence, disorientation, and motor
B. The client will learn new coping mechanisms to restlessness
handle anxiety.
C. The client will seek out resources in the community 3.A client in the psychiatric unit has been diagnosed
for support. with severe, intractable depression and is preparing to
D. The client will follow an established schedule for undergo electroconvulsive therapy (ECT). Which best
activities of daily living. describes the role of the nurse during ECT? Select all
that apply.
11. During the home visit of a client with dementia, the a) Prepare the client for applying EEG leads to the
nurse notes that an adult daughter persistently corrects head
her father's misperceptions of reality, even when the b) Notify the provider immediately if the client is
father becomes upset and anxious. Which intervention confused following the procedure
should the nurse teach the caregiver? c) Note the location of resuscitative equipment on
the nursing unit in case of need
A. Anxiety-reducing measures d) Administer oxygen as needed
B. Positive reinforcement e) Record the length of time of the client’s seizure
C. Reality orientation techniques
D. Validation techniques 4.A nurse is working with a client who suffers from
depression. The client has started taking medications
12. Mr. Lim who is diagnosed with moderate dementia and is engaged in group therapy, but still tells the nurse,
has frequent catastrophic reactions during shower time. “I do not like myself. I am annoying, even to me.” Which
Which of the following interventions should be activities can the nurse suggest that would most likely
implemented in the plan of care? Select all that apply. increase this client’s self-concept? Select all that apply.
a) Have the client recognize personal strengths
A. Assign consistent staff members to assist the client. b) Teach the client how to be mindful of negative
B. Accomplish the task quickly, with several staff thoughts
members assisting. c) Ask if the client has any friends
C. Schedule the client's shower at the same time of day. d) Remind the client not to compare self to others
D. Sedate the client 30 minutes prior to showering. e) List three things that the nurse likes about the
E. Tell the client to remain calm while showering. client
F. Use a calm, supportive, quiet manner when assisting
the client. 5.A nurse is counseling a client who has been diagnosed
with depression to attend a support group as part of
treatment. Which best describes how the nurse would
Psychiatric Nursing Exam explain what to expect for the client?
Depression a) A presentation where group members watch an
1.A nurse is caring for a client who was diagnosed with audiovisual demonstration
depression and anxiety three years ago. The client b) A small group where the client will have to
currently has prescriptions for four different facilitate the discussion between members
medications for control of symptoms. Which nursing c) A large group of up to 50 people meeting and
intervention is most appropriate to reduce the risks mingling together
associated with taking many different medications in d) A small group where the client may need to talk
this situation? about his or her mental health issues
a) Help the client to organize the medications and
make a list of their uses and side effects 6.A client who has developed depression after
b) Tell the client that he should never take over-the- experiencing chronic back pain is being seen at the
counter medications when using this many acute care clinic. Which best describes how the nurse
prescription drugs would demonstrate psychoeducation when working
c) Have the client contact the provider and ask to be with this client?
changed to an all-in-one drug instead of taking 4 a) Helping the client to find a support group for
different kinds people who suffer from back pain
b) Providing information to the client about an What describes the difference between stress response
upcoming research study based on the effects syndrome and clinical depression?
of pain and depression A) Stress response syndrome causes hopelessness and
c) Administering medications that will relieve loss of interest in activities, but not suicidal ideation
some of the back pain such as with clinical depression
d) Teaching the client about the effects of their B) Stress response syndrome often appears after a life-
mental health issue so it can be better managed threatening event, while clinical depression appears
after a life-changing event
7. A 30-year-old client has been diagnosed with major C) Stress response syndrome involves an increase in
depressive disorder. Which nutrition guideline should pulse, blood pressure and breathing rate, while clinical
the nurse give to this client that could help with some depression involves a decrease in these vital signs
symptoms of depression? D) Stress response syndrome typically lasts for months
a) Fluids such as water and sugar-free juice are to years, while clinical depression may last for several
preferred over caffeinated beverages weeks
b) The best diet for depression is the Western diet
c) The client should increase carbohydrate intake 3. A client is being seen for care at her primary care
and limit fats clinic. The client tells the nurse that she has difficulty
d) Some free radicals in the diet combat harmful sleeping because of nightmares. Identify which
antioxidants description best explains the difference between night
terrors and nightmares.
8.A nurse is performing depression screenings in the A) Night terrors are frightening but are not
community. Which question would most likely be remembered by the client, while nightmares can be
included in this screening? easily remembered
a) Do you hear voices or have hallucinations? B) Night terrors cause a person to sleep walk while
b) Do you believe that others are talking or nightmares usually result in thrashing in bed
thinking about you? C) Night terrors result in the person waking up while
c) Have you been having trouble sleeping? nightmares result in the person staying asleep
d) Are you trying to lose weight? D) Night terrors are the sign of a more significant
mental illness while nightmares are considered benign
9.A nurse is caring for a client has undergone ECT for
the treatment of severe depression and has developed 4. A 6-year-old child has been brought in for care and
some complications afterward. Which of the following is treatment after suffering from physical abuse by her
a physical side effect that has been associated with father. The child is experiencing a post-traumatic fight-
ECT? or-flight response. Which signs or symptoms would the
a) Bradycardia nurse most likely expect to initially see in this client?
b) Rash on the face and trunk A) Crying and screaming
c) Muscle soreness B) Dissociation
d) Peripheral edema C) Somnolence
D) Stuttering and refusal to respond to caregivers
10.An 11-year-old boy has been diagnosed with
depression after his parents’ divorce. The nurse 5. A nurse is caring for a client with bipolar disorder
understands that depression in children of this age most who is frequently manic. What describes the most
commonly manifests as: appropriate psychotherapy for a client with this
a) Whining condition? Select all that apply.
b) Low self esteem A) Cognitive-behavior therapy
c) Poor blood glucose control B) 12-step programs
d) Pouting C) Interpersonal and social rhythm therapy
D) Family focused therapy
Psychiatric Nursing Exam E) Electroconvulsive therapy

1. A client is suffering from stress and anxiety and is 6. A client with bipolar disorder is seeking treatment for
being seen at a healthcare clinic for help and co-occurring substance abuse disorder. During the
management. Which best describes the initial physical initial assessment and treatment process, the nurse
effects of stress and anxiety in the body? Select all that notes the development of countertransference. Which
apply. best describes an example of this response?
A) Increased heart rate A) The nurse feels irritated because the client reminds
B) Pinpoint pupils the nurse of her sister
C) Vasodilation B) The client compares the nurse to her mother
D) Changes in appetite C) The client blames the nurse for her bipolar disorder
E) Increased respiratory rate D) The client determines that her substance abuse
disorder was caused by the bipolar disorder
2. A client is experiencing stress response syndrome
after losing his job where he had worked for 21 years. 7. A client who has developed depression after
experiencing chronic back pain is being seen at the
acute care clinic. Which best describes how the nurse C) Record the length of time of the client’s seizure
would demonstrate psychoeducation when working D) Administer oxygen as needed
with this client? E) Notify the provider immediately if the client is
A) Teaching the client about the effects of their mental confused following the procedure
health issue so it can be better managed
B) Helping the client to find a support group for people 12. Positive symptoms of schizophrenia
who suffer from back pain  Hallucinations
C) Administering medications that will relieve some of  Delusions
the back pain  Disorganized speech
D) Providing information to the client about an  Bizarre behavior
upcoming research study based on the effects of pain 13. Negative symptoms of schizophrenia
and depression  Decreased emotional range
 Loss of interest
8. A nurse is assisting a client who has undergone  Lack of inertia
electroconvulsive therapy for treatment of severe 14.A nurse is caring for a client with schizophrenia who
depression. Following the procedure, the client tells her he believes that everyone else in the inpatient
develops postictal agitation. Based on the nurse’s unit is secretly laughing at him behind his back. Which
knowledge of this condition, the nurse would expect to response by the nurse is best?
see which of the following? A) There are some people here who are laughing but I
A) Incoherence, disorientation, and motor restlessness do not think they are laughing at you.
B) Hyperactivity and mania B) Did you hear someone say something about you?
C) Limb contractures C) Do you think they do not like you?
D) Coma D) They are probably just laughing about someone else.

9. A nurse is working with a client who suffers from 15. A 50-year-old client with schizophrenia is being seen
depression. The client has started taking medications by the mental health nurse. The client is demonstrating
and is engaged in group therapy, but still tells the nurse, signs of altered thought processes. Which
“I do not like myself. I am annoying, even to me.” Which communication pattern would most be associated with
activities can the nurse suggest that would most likely schizophrenia or psychosis? Select all that apply.
increase this client’s self-concept? Select all that apply A) Word salad
A) Teach the client how to be mindful of negative B) Poverty of speech
thoughts C) Flight of ideas
B) List three things that the nurse likes about the client D) Neologisms
C) Ask if the client has any friends E) Clanging
D) Have the client recognize personal strengths
E) Remind the client not to compare self to others 16. A client with catatonic schizophrenia is in the
hospital on the mental health unit. The client has not
10. A nurse is caring for a client who was diagnosed moved for three days. Which of the following nursing
with depression and anxiety three years ago. The client interventions best demonstrates that the nurse is
currently has prescriptions for four different upholding the client’s safety in this situation?
medications for control of symptoms. Which nursing A) Perform range of motion exercises and apply
intervention is most appropriate to reduce the risks sequential compression devices
associated with taking many different medications in B) Administer methylphenidate to control behavior
this situation? C) Place the client in seclusion until the catatonia has
A) Help the client to organize the medications and make resolved
a list of their uses and side effects D) Start an IV to administer a bolus of normal saline
B) Have the client contact the provider and ask to be solution
changed to an all-in-one drug instead of taking 4
different kinds 17. Cluster A personality disorders
C) Tell the client that he should never take over-the-  Odd & eccentric
counter medications when using this many prescription  Schizoid
drugs  Schizotypical
D) Make an appointment for the client to meet with a  Paranoid
dietitian to discuss the nutritional effects of taking this 18. Cluster C personality disorders
many drug
 Anxious & fearful
11. A client in the psychiatric unit has been diagnosed  Obsessive-compulsive
with severe, intractable depression and is preparing to  Avoidant
undergo electroconvulsive therapy (ECT). Which best  Dependant
describes the role of the nurse during ECT? Select all 19. Apraxia
that apply.
 Difficulty with motor tasks
A) Prepare the client for applying EEG leads to the head
20. Aphasia
B) Note the location of resuscitative equipment on the
 difficulty with speech
nursing unit in case of need
21. Agnosia 28. Symptoms of alcohol withdrawal
 Difficulty with recognition  Tremor
22 Anomia  Anxiety
 Difficulty with naming  N/V
 Insomnia
23. A 68-year-old client is being seen because of  Hypertension
problems with memory loss. Which best describes an  Diaphoresis
example of the difference between normal memory  Hallucinations
changes with aging versus dementia?  Seizures
A) Normal memory aging involves recollective memory 29. Delirium Tremens
decline while dementia involves reconstructive memory  Medical emergency
decline  3-10 days after last drink
B) Normal memory changes may be noted as occasional  Confusion/disorintation
forgetfulness while dementia results in rapid loss of all  Agitation
things familiar  High fever
C) Normal memory loss affects interpersonal skills while  Autonomic instability
dementia affects social skills  Seizures
D) Normal memory loss involves forgetting short term  Deadly
memories while dementia involves forgetting long term 30. Nursing interventions for alcohol withdrawal
memories  Frequent assessments
 Seizures and fall precautions
24. Which medical condition has been shown to cause
 Reorient as needed
dementia in some adults?
 Maintain safety
A) Lyme disease
 Self-harm assessment
B) Trichomonas
 Long-term therapy
C) Varicella
31. Medications for alcohol withdrawal
D) Tuberculosis
 Benzos
 Antiemetics
25. A 14-year-old teen has been hospitalized for acute
exacerbation of ADHD symptoms. The nurse tries to  Thiamine
help the client to engage with others on the unit. Which  B12
of the following interventions would be most  Disulfiram
appropriate for supporting social interaction for this
client? 32. Which of the following situations best describes
A) Help the child join a structured game with other acute grief?
children A) A person is suddenly overwhelmed and starts to cry
B) Bring the client to sit with another teen who is when she sees a picture of her deceased friend
playing a video game B) A nurse sits with a client while he cries over the
C) Have the client sit and talk with another client in a death of his father
one-to-one setting C) A client feels extreme pain over the death of a child
D) Avoid social interactions until the ADHD symptoms four months earlier
are under control D) A client commits suicide because he can no longer
handle the sadness of a break up
26. A nurse is helping a client who has been developing
dementia. The client expresses fears about his condition 33. A nurse is caring for a woman whose mother died
to the nurse. Which nursing interventions would be six months ago and who is grieving over the loss. The
most appropriate in this case? Select all that apply. nurse assesses the woman for any signs of complicated
A) Help orient the client to reality grief patterns. Based on the nurse’s understanding of
B) Gently provide assistance with communication when this condition, the nurse knows to look for what signs
needed that indicate complicated grief? Select all that apply.
C) Use simple, short words A) A complete focus on the deceased person
D) Support and reassure the client B) An increase in sexual activity
E) Avoid activities that tax the client's memory C) A lack of trust in others
27. A client with Alzheimer’s disease has had difficulties D) Feelings of detachment from the world
eating and is not getting enough nutrients in his diet. E) Lack of emotion such as crying or sorrow
The client’s daughter asks the nurse if there is anything
that can be done to improve his nutrition intake. Which 34. Who is at high risk for suicide?
recommendation should the nurse give?  Personal of family history
A) Use more salt when cooking and serving food  Mental health disorders
B) Help the client choose his own eating utensils  Terminal illness
C) Provide stand-by assistance when the client eats to  Physical disabilities
offer support  Elderly and adolescents
D) Limit calories to have better control of behavior 35. Signs of impending suicide attempt
 Giving away prized possesions
 Changing will/life insurance b) aphasia
 Loss of interest in life c) agnosia
 Asking about methods d) amnesia
 Notes to loved ones
 Sudden massive improvements in mood 7. She tearfully tells the nurse “I can’t take it when she
accuses me of stealing her things.” Which response by
the nurse will be most therapeutic?
Psychiatric Nursing 2
1. Situation : The nurse assigned in the detoxification a) ”Don’t take it personally. Your mother does not
unit attends to various patients with substance-related mean it.”
disorders. A 45 years old male revealed that he b) “Have you tried discussing this with your
experienced a marked increase in his intake of alcohol mother?”
to achieve the desired effect This indicates: c) “This must be difficult for you and your
mother.”
a) withdrawal d) “Next time ask your mother where her things
b) tolerance were last seen.”
c) intoxication
d) psychological dependence 8. The primary nursing intervention in working with a
client with moderate stage dementia is ensuring that
2. The client admitted for alcohol detoxification the client:
develops increased tremors, irritability, hypertension
and fever. The nurse should be alert for impending: a) receives adequate nutrition and hydration
b) will reminisce to decrease isolation
a) delirium tremens c) remains in a safe and secure environment
b) Korsakoff’s syndrome d) independently performs self care
c) esophageal varices
d) Wernicke’s syndrome 9. She says to the nurse who offers her breakfast, “Oh
no, I will wait for my husband. We will eat together”
3. The care for the client places priority to which of the The therapeutic response by the nurse is:
following:
a) “Your husband is dead. Let me serve you your
a) Monitoring his vital signs every hour breakfast.”
b) Providing a quiet, dim room b) “I’ve told you several times that he is dead. It’s
c) Encouraging adequate fluids and nutritious time to eat.”
foods c) “You’re going to have to wait a long time.”
d) Administering Librium as ordered d) “What made you say that your husband is alive?

4. Another client is brought to the emergency room by 10. Dementia unlike delirium is characterized by:
friends who state that he took something an hour ago.
He is actively hallucinating, agitated, with irritated nasal a) slurred speech
septum. b) insidious onset
c) clouding of consciousness
a) Heroin d) sensory perceptual change
b) cocaine
c) LSD 11. Situation: A 17 year old gymnast is admitted to the
d) marijuana hospital due to weight loss and dehydration secondary
to starvation. Which of the following nursing diagnoses
5. A client is admitted with needle tracts on his arm, will be given priority for the client?
stuporous and with pin point pupil will likely be
managed with: a) altered self-image
b) fluid volume deficit
a) Naltrexone (Revia) c) altered nutrition less than body requirements
b) Narcan (Naloxone) d) altered family process
c) Disulfiram (Antabuse)
d) Methadone (Dolophine) 12. What is the best intervention to teach the client
when she feels the need to starve?
6. Situation: An old woman was brought for evaluation
due to the hospital for evaluation due to increasing a) Allow her to starve to relieve her anxiety
forgetfulness and limitations in daily function. The b) Do a short term exercise until the urge passes
daughter revealed that the client used her toothbrush c) Approach the nurse and talk out her feelings
to comb her hair. She is manifesting: d) Call her mother on the phone and tell her how
she feels
a) apraxia
13. The client with anorexia nervosa is improving if:
20. Which is the desired outcome in conducting
a) She eats meals in the dining room. desensitization:
b) Weight gain
c) She attends ward activities. a) The client verbalize his fears about the situation
d) She has a more realistic self concept. b) The client will voluntarily attend group therapy
in the social hall.
14. The characteristic manifestation that will c) The client will socialize with others willingly
differentiate bulimia nervosa from anorexia nervosa is d) The client will be able to overcome his disabling
that bulimic individuals fear.

a) have episodic binge eating and purging 21. Which of the following should be included in the
b) have repeated attempts to stabilize their weight health teachings among clients receiving Valium:
c) have peculiar food handling patterns
d) have threatened self-esteem a) Avoid taking CNS depressant like alcohol.
b) There are no restrictions in activities.
15. A nursing diagnosis for bulimia nervosa is c) Limit fluid intake.
powerlessness related to feeling not in control of eating d) Any beverage like coffee may be taken
habits. The goal for this problem is:
22. Situation: A 20 year old college student is admitted
a) Patient will learn problem solving skills to the medical ward because of sudden onset of
b) Patient will have decreased symptoms of paralysis of both legs. Extensive examination revealed
anxiety. no physical basis for the complaint. The nurse plans
c) Patient will perform self care activities daily. intervention based on which correct statement about
d) Patient will verbalize how to set limits on conversion disorder?
others.
a) The symptoms are conscious effort to control
16. In the management of bulimic patients, the anxiety
following nursing interventions will promote a b) The client will experience high level of anxiety in
therapeutic relationship EXCEPT: response to the paralysis.
c) The conversion symptom has symbolic meaning
a) Establish an atmosphere of trust to the client
b) Discuss their eating behavior. d) A confrontational approach will be beneficial for
c) Help patients identify feelings associated with the client.
binge-purge behavior
d) Teach patient about bulimia nervosa 23. Nina reveals that the boyfriend has been pressuring
her to engage in premarital sex. The most therapeutic
17. Situation: A 35 year old male has intense fear of response by the nurse is:
riding an elevator. He claims “ As if I will die inside.” This
has affected his studies The client is suffering from: a) “I can refer you to a spiritual counselor if you
like.”
a) agoraphobia b) “You shouldn’t allow anyone to pressure you
b) social phobia into sex.”
c) Claustrophobia c) “It sounds like this problem is related to your
d) xenophobia paralysis.”
d) “How do you feel about being pressured into
18. Initial intervention for the client should be to: sex by your boyfriend?”

a) Encourage to verbalize his fears as much as he 24. Malingering is different from somatoform disorder
wants. because the former:
b) Assist him to find meaning to his feelings in
relation to his past. a) Has evidence of an organic basis.
c) Establish trust through a consistent approach. b) It is a deliberate effort to handle upsetting
d) Accept her fears without criticizing. events
c) Gratification from the environment are
19. The nurse develops a countertransference reaction. obtained.
This is evidenced by: d) Stress is expressed through physical symptoms.

a) Revealing personal information to the client 25. Unlike psychophysiologic disorder Linda may be best
b) Focusing on the feelings of the client. managed with:
c) Confronting the client about discrepancies in
verbal or non-verbal behavior a) medical regimen
d) The client feels angry towards the nurse who b) milieu therapy
resembles his mother. c) stress management techniques
d) psychotherapy
Anxiety Disorders and Stress: 75 Questions 6. Question
1. Question Marty is pacing and complains of racing thoughts. Nurse
Chuck is a 20-year-old student diagnosed with Lally asks the client if something upsetting happened,
obsessive-compulsive behavior. A psychiatrist and Marty's response is vague and not focused on the
prescribes clomipramine (Anafranil) to treat his question. Nurse Lally assess Marty's level of anxiety as:
condition. Nurse Nicolette understands the rationale for A. Mild
this treatment is that the clomipramine: B. Moderate
A. Increases dopamine levels C. Severe
B. Increases serotonin levels D. Panic
C. Decreases norepinephrine levels
D. Decreases GABA levels 7. Question
Nurse Martha is teaching her students about anxiety
2. Question medications; she explains that benzodiazepines affect
A nurse at Nurseslabs Medical Center is developing a which brain chemical?
care plan for a female client with post-traumatic stress A. Acetylcholine
disorder. Which of the following would she do initially? B. Gamma-aminobutyric acid (GABA)
A. Instruct the client to use distraction techniques to C. Norepinephrine
cope with flashbacks. D. Serotonin
B. Encourage the client to put the past in proper
perspective. 8. Question
C. Encourage the client to verbalize thoughts and Mandy, a nurse who works at Nurseslabs Rehabilitation
feelings about the trauma. Center is assessing a client for recent stressful life
D. Avoid discussing the traumatic event with the client. events. She recognizes that stressful life events are
both:
3. Question A. Desirable and growth-promoting
A group of community nurses sees and plans care for B. Positive and negative
various clients with different types of problems. Which C. Undesirable and harmful
of the following clients would they consider the most D. Predictable and controllable
vulnerable to post-traumatic stress disorder?
A. An eight (8)-year-old boy with asthma who has 9. Question
recently failed a grade in school. During a community visit, volunteer nurses teach stress
B. A 20-year-old college student with DM who management to the participants. The nurses will most
experienced date rape. likely advocate which belief as a method of coping with
C. A 40-year-old widower who has recently lost his wife stressful life events?
to cancer. A. Avoidance of stress is an important goal for living.
D. A wife of an individual with a severe substance abuse B. Control over one's response to stress is possible.
problem. C. Most people have no control over their level of
stress.
4. Question D. Significant others are important to provide care and
Which outcome is most appropriate for Francis who has concern.
a dissociative disorder?
A. Francis will deal with uncomfortable emotions on a 10. Question
conscious level. Genevieve only attends social events when a family
B. Francis will modify stress with the use of relaxation member is also present. She exhibits behavior typical of
techniques. which anxiety disorder?
C. Francis will identify his anxiety responses. A. Agoraphobia
D. Francis will use problem-solving strategies when B. Generalized anxiety disorder
feeling stressed. C. Obsessive-compulsive disorder
D. Post-traumatic stress disorder
5. Question
The psychiatric nurse uses cognitive-behavioral 11. Question
techniques when working with a client who experiences Mr. Johnson was recently admitted to a psychiatric unit
panic attacks. Which of the following techniques are because of severe obsessive-compulsive behavior.
common to this theoretical framework? Select all that Which initial response by the nurse would be most
apply. therapeutic for him?
A. Administering anti-anxiety medication as prescribed. A. Accepting the client's ritualistic behaviors.
B. Encouraging the client to restructure thoughts. B. Challenging the client's need for rituals.
C. Helping the client to use controlled relaxation C. Expressing concern about the harmfulness of the
breathing. client's rituals.
D. Helping the client examine evidence of stressors. D. Limiting the client's rituals that are excessive.
E. Questioning the client about early childhood
relationships. 12. Question
F. Teaching the client about anxiety and panic.
Nurse Vicky is assessing a newly admitted client for Nurse Wayne is planning a psychoeducational
symptoms of post-traumatic stress disorder (PTSD). discussion for a group of adolescent clients with
Which symptoms are typically seen with this diagnosis? anorexia nervosa. Which of the following topics would
Select all that apply. Nurse Wayne select to enhance understanding about
A. Anger with numbing of other emotions. central issues in this disorder?
B. Exaggerated startle response. A. Anger management
C. Feeling that one is having a heart attack. B. Parental expectations
D. Frequent thoughts about contamination. C. Peer pressure and substance abuse
E. Frequent nightmares. D. Self-control and self-esteem
F. Survivor's guilt.
19. Question
13. Question Nurse Gina understands that her client Glenda who is
Jordanne is a client with a fear of air travel. She is being bulimic feels shame and guilt over binge eating and
treated in a mental institution for phobic disorder. The purging. This disorder is therefore considered:
treatment method involves systematic desensitization. A. Ego-distorting
The nurse would consider the treatment successful if: B. Ego-dystonic
A. Jordanne plans a trip requiring air travel. C. Ego-enhancing
B. Jordanne takes a short trip on an airplane. D. Ego-syntonic
C. Jordanne recognizes the unrealistic nature of the fear
of riding on airplanes. 20. Question
D. Jordanne verbalizes a decreased fear of air travel. The psychoanalytic theory explains the etiology of
anorexia nervosa as:
14. Question A. The achievement of secondary gain through control
Nurse Kerrick observes Toni who is hospitalized on an of eating.
eating disorder unit during mealtimes and for 1 hour B. A conflict between mother and child over separation
after eating. An explanation for this intervention is: and individualization.
A. To develop a trusting relationship. C. Family dynamics that lead to enmeshment of
B. To maintain focus on the importance of nutrition. members.
C. To prevent purging behaviors. D. The incorporation of thinness as an ideal body image.
D. To reinforce the behavioral contact.
21. Question
15. Question The school nurse assesses for anorexia nervosa in an
Marlyn is diagnosed with anorexia nervosa and is adolescent girl. Which of the following findings are
admitted to the special eating disorder unit. The initial characteristic of this disorder? Select all that apply.
treatment priority for her is: A. Bradycardia
A. To determine her current body image. B. Hypotension
B. To identify family interaction patterns. C. Chronic pain in one or more sites
C. To initiate a refeeding program. D. Fear of having a serious illness
D. To promote the client's independence. E. Irregular or absent menses
F. Refusal to maintain a minimally normal weight

16. Question 22. Question


The nurse evaluates the treatment of Mrs. Montez with Mr. Bartowski who is newly diagnosed with rheumatoid
somatoform disorder as successful if: arthritis asks the community nurse how stress can affect
A. Mrs. Montez practices self-medication rather than his disease. The nurse would explain that:
changing health care providers. A. The psychological experience of stress will not affect
B. Mrs. Montez recognizes that physical symptoms symptoms of physical disease.
increase anxiety level. B. Psychological stress can cause painful emotions,
C. Mrs. Montez researches treatment protocols for which are harmful to a person with an illness.
various illnesses. C. Stress can overburden the body's immune system,
D. Mrs. Montez verbalizes anxiety directly rather than and therefore one can experience increased symptoms.
displacing it. D. The body's stress response is stimulated when there
are major disruptions in one's life.
17. Question
Which of the following attitudes from a nurse would
hinder a discussion with an adolescent client about
sexuality?
A. Accepting 23. Question
B. Matter-of-fact During a mother's class, the nurse who is teaching the
C. Moralistic participants on stress management is questioned about
D. Non Judgemental the use of alternative treatments, such as herbal
therapy and therapeutic touch. She explains that the
18. Question advantage of these methods would include all of the
following except:
A. They are congruent with many cultural belief 28. Question
systems. Letty says, "Give me ten (10) minutes to recall the name
B. They encourage the consumer to take an active role of our college professor who failed many students in
in health management. our anatomy class." She is operating on her:
C. They promote interrelationships within the mind- A. Subconscious
body-spirit. B. Conscious
D. They usually work better than traditional medical C. Unconscious
practice. D. Ego

24. Question 29. Question


David is preoccupied with numerous bodily complaints The superego is that part of the psyche that:
even after a careful diagnostic workup reveals no A. Uses defensive function for protection.
physiologic problems. Which nursing intervention B. Is impulsive and without morals.
would be therapeutic for him? C. Determines the circumstances before making
A. Acknowledge that the complaints are real to the decisions.
client, and refocus the client on other concerns and D. The censoring portion of the mind.
problems.
B. Challenge the physical complaints by confronting the 30. Question
client with the normal diagnostic findings. Primary level of prevention is exemplified by:
C. Ignore the client's complaints, but request that the A. Helping the client resume self-care.
client keeps a list of all symptoms. B. Ensuring the safety of a suicidal client in the
D. Listen to the client's complaints carefully, and institution.
question him about specific symptoms. C. Teaching the client stress management techniques.
D. Case finding and surveillance in the community.
25. Question
Nurse Kenzo is teaching a client about sertraline 31. Question
(Zoloft), which has been prescribed for depression. A Situation: In a home visit done by the nurse, she
significant side effect is an interference with sexual suspects that the wife and her child are victims of
arousal by inhibiting erectile function. How should abuse. Which of the following is the most appropriate
Nurse Kenzo approach this topic? for the nurse to ask?
A. Nurse Kenzo should avoid mentioning the sexual side A. "Are you being threatened or hurt by your partner?"
effects to prevent the client from having anxiety about B. "Are you frightened of your partner?"
potential erectile problems. C. "Is something bothering you?"
B. Nurse Kenzo should advise the client to report any D. "What happens when you and your partner argue?"
changes in sexual functioning in case medication
adjustments are needed. 32. Question
C. Nurse Kenzo should explain that the client's sexual The wife admits that she is a victim of abuse and opens
desire will probably decrease while on this medication. up about her persistent distaste for sex. This sexual
D. Nurse Kenzo should tell the client that sexual side disorder is:
effects are expected, but that they will decrease when A. Sexual desire disorder
his depression lifts. B. Sexual arousal disorder
C. Orgasm disorder
26. Question D. Sexual Pain Disorder
Mental health is defined as:
A. The ability to distinguish what is real from what is 33. Question
not. What would be the best approach for a wife who is still
B. A state of well-being where a person can realize his living with her abusive husband?
own abilities can cope with normal stresses of life and A. "Here's the number of a crisis center that you can call
work productively. for help ."
C. Is the promotion of mental health, prevention of B. "It's best to leave your husband."
mental disorders, nursing care of patients during illness C. "Did you discuss this with your family?"
and rehabilitation. D. " Why do you allow yourself to be treated this way."
D. Absence of mental illness.
34. Question
27. Question Which comment about a 3-year-old child if made by the
Which of the following describes the role of a parent may indicate child abuse?
technician? A. "Once my child is toilet trained, I can still expect her
A. Administers medications to a schizophrenic patient. to have some."
B. The nurse feeds and bathes a catatonic client. B. "When I tell my child to do something once, I don't
C. Coordinates diverse aspects of care rendered to the expect to have to tell."
patient. C. "My child is expected to try to do things such as dress
D. Disseminates information about alcohol and its and feed."
effects. D. "My three (3)-year-old loves to say NO."
35. Question D. Remove the child from the classroom when
The primary nursing intervention for a victim of child disruptive behavior occurs.
abuse is:
A. Assess the scope of the problem. 42. Question
B. Analyze the family dynamics. The child with conduct disorder will likely demonstrate:
C. Ensure the safety of the victim. A. Easy distractibility to external stimuli.
D. Teach the victim coping skills. B. Ritualistic behaviors
C. Preference for inanimate objects.
36. Question D. Serious violations of age-related norms.
Situation: A 30-year-old male employee frequently
complains of low back pain that leads to frequent 43. Question
absences from work. Consultation and tests reveal Ritalin is the drug of choice for children with ADHD.
negative results. The client has which somatoform Which of the following side effects may be noted?
disorder? A. Increased attention span and concentration
A. Somatization Disorder B. Increase in appetite
B. Hypochondriasis C. Sleepiness and lethargy
C. Conversion Disorder D. Bradycardia and diarrhea
D. Somatoform Pain Disorder
44. Question
37. Question School phobia is usually treated by:
Freud explains anxiety as: A. Returning the child to the school immediately with
A. Strives to gratify the needs for satisfaction and family support.
security. B. Calmly explaining why attendance in school is
B. Conflict between id and superego. necessary.
C. A hypothalamic-pituitary-adrenal reaction to stress. C. Allowing the child to enter the school before the
D. A conditioned response to stressors. other children.
D. Allowing the parent to accompany the child in the
38. Question classroom.
The following are the appropriate nursing diagnosis for
the client except: 45. Question
A. Ineffective individual coping A 10-year-old child has very limited vocabulary and
B. Alteration in comfort, pain interaction skills. She has an I.Q. of 45. She is diagnosed
C. Altered role performance to have Mental retardation of this classification:
D. Impaired social interaction A. Profound
B. Mild
39. Question C. Moderate
The following statements describe somatoform D. Severe
disorders:
A. Physical symptoms are explained by organic causes. 46. Question
B. It is a voluntary expression of psychological conflicts. The nurse teaches the parents of a mentally retarded
C. Expression of conflicts through bodily symptoms. child regarding her care. The following guidelines may
D. Management entails a specific medical treatment. be taught except:
A. Overprotection of the child.
40. Question B. Patience, routine, and repetition.
What would be the best response to the client's C. Assisting the parents set realistic goals.
repeated complaints of pain: D. Giving reasonable compliments.
A. "I know the feeling is real tests revealed negative
results." 47. Question
B. "I think you're exaggerating things a little bit." The parents express apprehensions on their ability to
C. "Try to forget this feeling and have activities to take it care for their maladaptive child. The nurse identifies
off your mind." what nursing diagnosis:
D. "So tell me more about the pain." A. Hopelessness
B. Altered parenting role
41. Question C. Altered family process
Situation: A nurse may encounter children with mental D. Ineffective coping
disorders. Her knowledge of these various disorders is
vital. When planning school interventions for a child 48. Question
with a diagnosis of attention deficit hyperactivity A 5-year-old boy is diagnosed to have an autistic
disorder, a guide to remember is to: disorder. Which of the following manifestations may be
A. Provide as much structure as possible for the child. noted in a client with autistic disorder?
B. Ignore the child's overactivity. A. Argumentativeness, disobedience, angry outburst
C. Encourage the child to engage in any play activity to B. Intolerance to change, disturbed relatedness,
dissipate energy. stereotypes
C. Distractibility, impulsiveness, and overactivity
D. Aggression, truancy, stealing, lying
56. Question
49. Question Situation: An old woman was brought for evaluation
The therapeutic approach in the care of an autistic child due to the hospital for evaluation due to increasing
includes the following except: forgetfulness and limitations in daily function. The
A. Engage in diversionary activities when acting -out. daughter revealed that the client used her toothbrush
B. Provide an atmosphere of acceptance. to comb her hair. She is manifesting:
C. Provide safety measures. A. Apraxia
D. Rearrange the environment to activate the child. B. Aphasia
C. Agnosia
50. Question D. Amnesia
According to Piaget, a 5-year-old is at what stage of
development: 57. Question
A. Sensorimotor stage She tearfully tells the nurse "I can't take it when she
B. Concrete operations accuses me of stealing her things." Which response by
C. Pre-operational the nurse will be most therapeutic?
D. Formal operation A. "Don't take it personally. Your mother does not mean
it."
51. Question B. "Have you tried discussing this with your mother?"
Situation: The nurse assigned to the detoxification unit C. "This must be difficult for you and your mother."
attends to various patients with substance-related D. "Next time ask your mother where her things were
disorders. A 45 years old male revealed that he last seen."
experienced a marked increase in his intake of alcohol
to achieve the desired effect. This indicates: 58. Question
A. Withdrawal The primary nursing intervention in working with a
B. Tolerance client with moderate stage dementia is ensuring that
C. Intoxication the client:
D. Psychological dependence A. Receives adequate nutrition and hydration.
B. Will reminisce to decrease isolation.
52. Question C. Remains in a safe and secure environment.
The client admitted for alcohol detoxification develops D. Independently performs self-care.
increased tremors, irritability, hypertension, and fever.
The nurse should be alert for impending: 59. Question
A. Delirium tremens She says to the nurse who offers her breakfast, "Oh no, I
B. Korsakoff's syndrome will wait for my husband. We will eat together" The
C. Esophageal varices therapeutic response by the nurse is:
D. Wernicke's syndrome A. "Your husband is dead. Let me serve you your
breakfast."
53. Question B. "I've told you several times that he is dead. It's time
The care for the client places priority on which of the to eat."
following: C. "You're going to have to wait a long time."
A. Monitoring his vital signs every hour. D. "What made you say that your husband is alive
B. Providing a quiet, dim room.
C. Encouraging adequate fluids and nutritious foods. 60. Question
D. Administering Librium as ordered. Dementia, unlike delirium, is characterized by:
A. Slurred speech
54. Question B. Insidious onset
Another client is brought to the emergency room by C. Clouding of consciousness
friends who state that he took something an hour ago. D. Sensory perceptual change
He is actively hallucinating, agitated, with an irritated
nasal septum. 61. Question
A. Heroin Situation: A 17-year-old gymnast is admitted to the
B. Cocaine hospital due to weight loss and dehydration secondary
C. LSD to starvation. Which of the following nursing diagnoses
D. Marijuana will be given priority for the client?
A. Altered self-image
55. Question B. Fluid volume deficit
A client is admitted with needle tracks on his arm, C. Altered nutrition less than body requirements
stuporous and with pinpoint pupil will likely be D. Altered family process
managed with:
A. naltrexone (Revia) 62. Question
B. naloxone hydrochloride (Narcan) What is the best intervention to teach the client when
C. disulfiram (Antabuse) she feels the need to starve?
D. methadone (Dolophine) A. Allow her to starve to relieve her anxiety.
B. Do a short-term exercise until the urge passes. B. Focusing on the feelings of the client.
C. Approach the nurse and talk out her feelings. C. Confronting the client about discrepancies in verbal
D. Call her mother on the phone and tell her how she or non-verbal behavior.
feels. D. Revealing personal information to the client.

63. Question 70. Question


The client with anorexia nervosa is improving if: Which is the desired outcome in conducting
A. She eats meals in the dining room. desensitization:
B. Weight gain A. The client verbalizes his fears about the situation.
C. She attends ward activities. B. The client will voluntarily attend group therapy in the
D. She has a more realistic self-concept. social hall.
C. The client will socialize with others willingly.
64. Question D. The client will be able to overcome his disabling fear.
The characteristic manifestation that will differentiate
bulimia nervosa from anorexia nervosa is that bulimic 71. Question
individuals: Which of the following should be included in the health
A. Have episodic binge eating and purging. teachings among clients receiving Valium:
B. Have repeated attempts to stabilize their weight. A. Avoid taking CNS depressants like alcohol.
C. Have peculiar food handling patterns. B. There are no restrictions in activities.
D. Have threatened self-esteem. C. Limit fluid intake.
D. Any beverage like coffee may be taken.
65. Question
A nursing diagnosis for bulimia nervosa is 72. Question
powerlessness related to feeling not in control of eating Situation: A 20-year-old college student is admitted to
habits. The goal for this problem is: the medical ward because of sudden onset of paralysis
A. Patient will learn problem-solving skills. of both legs. Extensive examination revealed no physical
B. Patient will have decreased symptoms of anxiety. basis for the complaint. The nurse plans intervention
C. Patient will perform self-care activities daily. based on which correct statement about conversion
D. Patient will verbalize how to set limits on others. disorder?
A. The symptoms are conscious effort to control
66. Question anxiety.
In the management of bulimic patients, the following B. The client will experience a high level of anxiety in
nursing interventions will promote a therapeutic response to the paralysis.
relationship except: C. The conversion symptom has symbolic meaning to
A. Establish an atmosphere of trust. the client.
B. Discuss their eating behavior. D. A confrontational approach will be beneficial for the
C. Help patients identify feelings associated with binge- client.
purge behavior.
D. Teach the patient about bulimia nervosa. 73. Question
Nikki reveals that the boyfriend has been pressuring her
67. Question to engage in premarital sex. The most therapeutic
Situation: A 35-year-old male has an intense fear of response by the nurse is:
riding an elevator. He claims " As if I will die inside." This A. "I can refer you to a spiritual counselor if you like."
has affected his studies The client is suffering from: B. "You shouldn't allow anyone to pressure you into
A. Agoraphobia sex."
B. Social phobia C. "It sounds like this problem is related to your
C. Claustrophobia paralysis."
D. Xenophobia D. "How do you feel about being pressured into sex by
your boyfriend?"
68. Question
Situation: A 24-year-old female has an intense fear of 74. Question
spiders. Initial intervention for the client should be to: Malingering is different from somatoform disorder
A. Encourage to verbalize her fears as much as she because the former:
wants. A. Has evidence of an organic basis.
B. Assist her to find meaning to her feelings in relation B. It is a deliberate effort to handle upsetting events.
to her past. C. Gratification from the environment is obtained.
C. Establish trust through a consistent approach. D. Stress is expressed through physical symptoms.
D. Accept her fears without criticizing.
75. Question
69. Question Unlike psychophysiologic disorder Linda may be best
The nurse develops a countertransference reaction. This managed with:
is evidenced by: A. Medical regimen
A. The client feels angry towards the nurse who B. Milieu therapy
resembles his mother. C. Stress management techniques
D. Psychotherapy d) Helping the client examine evidence of stressors
e) Questioning the client about early childhood
Stress, Anxiety, Eating & Mind-Body Disorders relationships
f) Teaching the client about anxiety and panic
1. Adam is a 20-year-old student diagnosed of having
obsessive-compulsive behavior. A psychiatrist 6. Marty is pacing and complains of racing thoughts.
prescribes clomipramine (Anafranil) to treat his Nurse Lally asks the client if something upsetting
condition. Nurse Anna understands the rationale for happened, and Marty’s response is vague and not
this treatment is that the clomipramine: focused on the question. Nurse Lally assess Marty’s
level of anxiety as:
a) increases dopamine levels.
b) increases serotonin levels. a) mild.
c) decreases norepinephrine levels. b) moderate.
d) decreases GABA levels. c) severe.
d) panic.
2. Nurse Sarah is developing a care plan for a female
client with post-traumatic stress disorder. Which of the 7. Nurse Martha is teaching her students about anxiety
following would she do initially? medications, she explains that benzodiazepines affect
which brain chemical?
a) Instruct the client to use distraction techniques
to cope with flashbacks. a) Acetylcholine
b) Encourage the client to put the past in proper b) Gamma-aminobutyric acid (GABA)
perspective. c) Norepinephrine
c) Encourage the client to verbalize thoughts and d) Serotonin
feelings about the trauma.
d) Avoid discussing the traumatic event with 8. Nurse Mandy is assessing a client for recent stressful
client. life events. She recognizes that stressful life events are
both:
3. A group of community nurses sees and plans care for
various clients with different types of problems. Which a) desirable and growth-promoting.
of the following clients would they consider the most b) positive and negative.
vulnerable to post-traumatic stress disorder? c) undesirable and harmful.
d) predictable and controllable.
a) An 8 year-old boy with asthma who has recently
failed a grade in school 9. During a community visit, volunteer nurses teach
b) A 20 year-old college student with DM who stress management to the participants. The nurses will
experienced date rape most likely advocate which belief as a method of coping
c) A 40 year-old widower who has recently lost his with stressful life events?
wife to cancer
d) A wife of an individual with a severe substance a) Avoidance of stress is an important goal for
abuse problem living.
b) Control over one’s response to stress is
4. Which outcome is most appropriate for Francis who possible.
has a dissociative disorder? c) Most people have no control over their level of
stress.
a) Francis will deal with uncomfortable emotions d) Significant others are important to provide care
on a conscious level. and concern.
b) Francis will modify stress with the use of
relaxation techniques. 10. Genevieve only attends social events when a family
c) Francis will identify his anxiety responses. member is also present. She exhibits behavior typical of
d) Francis will use problem-solving strategies when which anxiety disorder?
feeling stressed.
a) Agoraphobia
5. The psychiatric nurse uses cognitive-behavioral b) Generalized anxiety disorder
techniques when working with a client who experiences c) Obsessive-compulsive disorder
panic attacks. Which of the following techniques are d) Post-traumatic stress disorder
common to this theoretical framework? Select all that
apply. 11. Mr. Johnson is newly admitted to a psychiatric unit
because of severe obsessive compulsive behavior.
a) Administering anti-anxiety medication as Which initial response by the nurse would be most
prescribed therapeutic for him?
b) Encouraging the client to restructure thoughts
c) Helping the client to use controlled relaxation a) Accepting the client’s ritualistic behaviors
breathing b) Challenging the client’s need for rituals
c) Expressing concern about the harmfulness of b) Matter-of-fact
the client’s rituals c) Moralistic
d) Limiting the client’s rituals that are excessive d) Nonjudgemental

12. Nurse Vicky is assessing a newly admitted client for 18. Nurse David is planning a psychoeducational
symptoms of post-traumatic stress disorder (PTSD). discussion for a group of adolescent clients with
Which symptoms are typically seen with this diagnosis? anorexia nervosa. Which of the following topics would
Select all that apply. Nurse David select to enhance understanding about
central issues in this disorder?
a) Anger with numbing of other emotions
b) Exaggerated startle response a) Anger management
c) Feeling that one is having a heart attack b) Parental expectations
d) Frequent thoughts about contamination c) Peer pressure and substance abuse
e) Frequent nightmares d) Self-control and self-esteem
f) Survivor’s guilt
19. Nurse Ginia understands that her client Glenda who
13. Jordanne is a client with a fear of air travel. She is is bulimic feels shame and guilt over binge eating and
being treated in a mental institution for phobic purging. This disorder is therefore considered:
disorder. The treatment method involves systematic
desensitization. The nurse would consider the a) ego-distorting.
treatment successful if: b) ego-dystonic.
c) ego-enhancing.
a) Jordanne plans a trip requiring air travel. d) ego-syntonic.
b) Jordanne takes a short trip in an airplane.
c) Jordanne recognizes the unrealistic nature of 20. The psychoanalytic theory explains the etiology of
the fear of riding on airplanes. anorexia nervosa as:
d) Jordanne verbalizes a decreased fear about air
travel. a) the achievement of secondary gain through
control of eating.
14. Nurse Kerrick observes Toni who is hospitalized on b) a conflict between mother and child over
an eating disorder unit during mealtimes and for 1 hour separation and individualization.
after eating. An explanation for this intervention is: c) family dynamics that lead to enmeshment of
members.
a) to develop trusting relationship. d) the incorporation of thinness as an ideal body
b) to maintain focus on importance of nutrition. image.
c) to prevent purging behaviors.
d) to reinforce the behavioral contact. 21. The school nurse assesses for anorexia nervosa in an
adolescent girl. Which of the following findings are
15. Marlyn is diagnosed of anorexia nervosa and is characteristic of this disorder? Select all that apply.
admitted in the special eating disorder unit. The initial
treatment priority for her is: a) Bradycardia
b) Hypotension
a) to determine her current body image. c) Chronic pain in one or more sites
b) to identify family interaction patterns. d) Fear of having a serious illness
c) to initiate a refeeding program. e) Irregular or absent menses
d) to promote the client’s independence. f) Refusal to maintain minimally normal weight

16. The nurse evaluates the treatment of Mrs. Montez 22. Mr. Bartowski who is newly diagnosed with
with somatoform disorder as successful if: rheumatoid arthritis asks the community nurse how
stress can affect his disease. The nurse would explain
a) Mrs. Montez practices self-medication rather that:
than changing health care providers.
b) Mrs. Montez recognizes that physical symptoms a) the psychological experience of stress will not
increase anxiety level. affect symptoms of physical disease.
c) Mrs. Montez researches treatment protocols b) psychological stress can cause painful emotions,
for various illnesses. which are harmful to a person with an illness.
d) Mrs. Montez verbalizes anxiety directly rather c) stress can overburden the body’s immune
than displacing it. system, and therefore one can experience
increased symptoms.
17. Which of the following attitudes from a nurse would d) the body’s stress response is stimulated when
hinder a discussion with an adolescent client about there are major disruptions in one’s life.
sexuality?
23. During a mother’s class, the nurse who is teaching
a) Accepting the participants on stress management is questioned
about the use of alternative treatments, such as herbal 2. Rachel is being treated in a chemical dependency
therapy and therapeutic touch. She explains that the unit. She tells the nurse that she only uses drugs when
advantage of these methods would include all of the under stress and therefore does not have a substance
following except: problem. Which defense mechanism is the client using?

a) they are congruent with many cultural belief A. Undoing


systems. B. Compensation
b) they encourage the consumer to take an active C. Denial
role in health management. D. Suppression
c) they promote interrelationships within the
mind-body-spirit. 3. Nurse Chelsey is teaching a community group about
d) they usually work better than traditional substance abuse. She explains that a genetic
medical practice. component has been implicated in which of the
24. David is preoccupied with numerous bodily following commonly abused substances?
complaints even after a careful diagnostic workup
reveals no physiologic problems. Which nursing A. Heroin
intervention would be therapeutic for him? B. Alcohol
C. Marijuana
a) Acknowledge that the complaints are real to the D. Barbiturates
client, and refocus the client on other concerns
and problems. 4. Nurse Michael recommends that the family of a client
b) Challenge the physical complaints by with substance-related disorder attend a support group,
confronting the client with the normal such as Al-Anon and Alateen. The purpose of these
diagnostic findings. groups is to help family members understand the
c) Ignore the client’s complaints, but request that problem and to:
the client keep a list of all symptoms.
d) Listen to the client’s complaints carefully, and A. Maintain focus on changing their own behaviors
question him about specific symptoms. B. Learn how to assist the abuser in getting help
C. Prevent substance problems in vulnerable family
25. Nurse Kenzo is teaching a client about sertraline members
(Zoloft), which has been prescribed for depression. A D. Change the problem behaviors of the abuser
significant side effect is interference with sexual arousal
by inhibiting erectile function. How should the Nurse 5. 5. Nurse Christine is teaching a client about disulfiram
Kenzo approach this topic? (Antabuse), which the client is taking to deter his use of
alcohol. She explains that using alcohol when taking this
a) Nurse Kenzo should avoid mentioning the medication can result in:
sexual side effects to prevent the client from
having anxiety about potential erectile A. Increased pulse and blood pressure
problems. B. Abdominal cramps and diarrhea
b) Nurse Kenzo should advise the client to report C. Drowsiness and decreased respiration
any changes in sexual functioning in case D. Flushing, vomiting, and dizziness
medication adjustments are needed.
c) Nurse Kenzo should explain that the client’s
sexual desire will probably decrease while on
this medication.
d) Nurse Kenzo should tell the client that sexual
side effects are expected, but that they will
decrease when his depression lifts.

Substance abuse disorders


1.Nurse James has observed a co-worker arriving to
work drunk at least three times in the past month.
Which action by Nurse James would best ensure client
safety and obtain necessary assistance for the co-
worker?

A. Warn the co-worker that this practice is unsafe


B. Report the coworker’s behavior to the appropriate
supervisor
C. Make general statements about safety issues at the
next staff meeting
D. Ignore the co worker’s behavior, and frequently
assess the clients assigned to the co-worker
Psychiatric Nursing 2 2. Denial
Substance Related Disorders 3. Suppression
1. The nurse understands that the essential difference 4. Undoing
between substance abuse and substance dependence is 8. The nurse is teaching a community group about
that substance dependence: substance abuse. She explains that a genetic
1. includes characteristics of tolerance and component has been implicated with which of the
withdrawal. following commonly abused substances?
2. includes characteristics of adverse consequences 1. Alcohol
and repeated use. 2. Barbiturates
3. produces less severe symptoms than that of 3. Heroin
abuse. 4. Marijuana
4. requires long-term treatment in a hospital-based
program. 9. The nurse recommends that the family of a client
with a substance-related disorder attend a support
2. The sister of a client with a substance-related group, such as Alanon or Alateen. The purpose of these
disorder tells the nurse that she calls out sick for the groups is to help family members understand the
client occasionally when he has too much to drink and problem and to:
cannot work. This behavior can be described as: 1. change the problem behaviors of the abuser.
1. caretaking. 2. learn how to assist the abuser in getting help.
2. codependent. 3. maintain focus on changing their own
3. helpful. behaviors.
4. supportive. 4. prevent substance problems in vulnerable
family members.
3. When a client abuses a CNS depressant, withdrawal
symptoms will be caused by which of the following? 10. The nurse is assessing a client who is a chronic
1. Acetylcholine excess alcohol abuser. Which problems are related to thiamin
2. Dopamine depletion deficiency?
3. Serotonin inhibition 1. Cardiovascular symptoms, such as decreased
4. Norepinephrine rebound hemoglobin and hematocrit levels
2. CNS symptoms, such as ataxia and peripheral
4. The general classification of drugs belonging to the neuropathy
opioid category is analgesic and: 3. Gastrointestinal symptoms, such as nausea and
1. depressant. vomiting
2. hallucinogenic. 4. Respiratory symptoms, such as cough and sore
3. stimulant. throat
4. tranquilizing.
11. When assessing a client who abuses barbiturates
5. The community nurse practicing primary prevention and benzodiazepine, the nurse would observe for
of alcohol abuse would target which groups for evidence of which withdrawal symptoms?
educational efforts? 1. Anxiety, tremors, and tachycardia
1. Adolescents in their late teens and young adults 2. Respiratory depression, stupor, and bradycardia
in their early twenties 3. Muscle aches, cramps, and lacrimation
2. Elderly men who live in retirement communities 4. Paranoia, depression, and agitation
3. Women working in careers outside the home
4. Women working in the home
12. When teaching an adolescent health class about the
6. A staff nurse has observed a coworker arriving to dangers of inhalant abuse, the nurse warns about the
work drunk at least three times in the past month. possibility of:
Which action by the nurse would best ensure client 1. contracting an infectious disease, such as
safety and obtain necessary assistance for the hepatitis or AIDS
coworker? 2. recurrent flashback events
1. Ignore the coworker's behavior, and frequently 3. psychological dependence after initial use
assess the clients assigned to the coworker. 4. sudden death from cardiac or respiratory
2. Make general statements about safety issues at depression
the next staff meeting.
3. Report the coworker's behavior to the 13. Which medication is commonly used in treatment
appropriate supervisor. programs for heroin abusers to produce a noneuphoric
4. Warn the coworker that this practice is unsafe. state and to replace heroin use?
1. diazepam
7. A client being treated in a chemical dependency unit 2. carbamazepine
tells the nurse that he only uses drugs when under 3. clonidine
stress and therefore does not have a substance 4. methadone
problem. Which defense mechanism is the client using?
1. Compensation
14. The nurse administers bromocriptine (Parlodel) to a
client undergoing detoxification for amphetamine 4. The superego is that part of the psyche that:
abuse. The rationale for this medication is to:
1. aid in GABA inhibition. 1. Uses defensive function for protection.
2. prevent norepinephrine excess. 2. Is impulsive and without morals.
3. restore depleted dopamine levels. 3. Determines the circumstances before making
4. treat psychotic symptoms. decisions.
4. The censoring portion of the mind.
15. The nurse is teaching a client about disulfiram
(Antabuse), which the client is taking to deter his use of 5. Primary level of prevention is exemplified by:
alcohol. She explains that using alcohol when taking this
medication can result in: 1. Helping the client resume self care.
1. abdominal cramps and diarrhea. 2. Ensuring the safety of a suicidal client in the
2. drowsiness and decreased respiration. institution.
3. flushing, vomiting, and dizziness. 3. Teaching the client stress management
4. increased pulse and blood pressure. techniques
4. Case finding and surveillance in the community
16. During an initial assessment of a client admitted to a
substance abuse unit for detoxification and treatment, 6. Situation: In a home visit done by the nurse, she
the nurse asks questions to determine patterns of use suspects that the wife and her child are victims of
of substances. Which of the following questions are abuse. Which of the following is the most appropriate
most appropriate at this time? Select all that apply. for the nurse to ask?
1. How long have you used substances?
2. How often do you use substances? 1. “Are you being threatened or hurt by your
3. How do you get substances into your body? partner?
4. Do you feel bad or guilty about your use of 2. “Are you frightened of you partner”
substances? 3. “Is something bothering you?”
5. How much of each substance do you use? 4. “What happens when you and your partner
6. Have you ever felt you should cut down argue?”
substance use?
7. What substances do you use? 7. The wife admits that she is a victim of abuse and
8. opens up about her persistent distaste for sex. This
Exam for Psychiatric Nursing sexual disorder is:
1. Mental health is defined as:
1. Sexual desire disorder
1. The ability to distinguish what is real from what 2. Sexual arousal Disorder
is not. 3. Orgasm Disorder
2. A state of well-being where a person can realize 4. Sexual Pain Disorder
his own abilities can cope with normal stresses
of life and work productively. 8. What would be the best approach for a wife who is
3. Is the promotion of mental health, prevention still living with her abusive husband?
of mental disorders, nursing care of patients
during illness and rehabilitation 1. “Here’s the number of a crisis center that you
4. Absence of mental illness can call for help .”
2. “Its best to leave your husband.”
2. Which of the following describes the role of a 3. “Did you discuss this with your family?”
technician? 4. “ Why do you allow yourself to be treated this
way”
1. Administers medications to a schizophrenic
patient. 9. Which comment about a 3 year old child if made by
2. The nurse feeds and bathes a catatonic client the parent may indicate child abuse?
3. Coordinates diverse aspects of care rendered to
the patient 1. “Once my child is toilet trained, I can still expect
4. Disseminates information about alcohol and its her to have some”
effects. 2. “When I tell my child to do something once, I
don’t expect to have to tell”
3. Liza says, “Give me 10 minutes to recall the name of 3. “My child is expected to try to do things such
our college professor who failed many students in our as, dress and feed.”
anatomy class.” She is operating on her: 4. “My 3 year old loves to say NO.”

1. Subconscious 10. The primary nursing intervention for a victim of


2. Conscious child abuse is:
3. Unconscious
4. Ego 1. Assess the scope of the problem
2. Analyze the family dynamics 3. encourage the child to engage in any play
3. Ensure the safety of the victim activity to dissipate energy
4. Teach the victim coping skills 4. remove the child from the classroom when
disruptive behavior occurs
11. Situation: A 30 year old male employee frequently
complains of low back pain that leads to frequent 17. The child with conduct disorder will likely
absences from work. Consultation and tests reveal demonstrate:
negative results. The client has which somatoform
disorder? 1. Easy distractibility to external stimuli.
2. Ritualistic behaviors
1. Somatization Disorder 3. Preference for inanimate objects.
2. Hypochondriaisis 4. Serious violations of age related norms.
3. Conversion Disorder
4. Somatoform Pain Disorder 18. Ritalin is the drug of choice for chidren with ADHD.
The side effects of the following may be noted:
12. Freud explains anxiety as:
1. increased attention span and concentration
1. Strives to gratify the needs for satisfaction and 2. increase in appetite
security 3. sleepiness and lethargy
2. Conflict between id and superego 4. bradycardia and diarrhea
3. A hypothalamic-pituitary-adrenal reaction to
stress 19. School phobia is usually treated by:
4. A conditioned response to stressors
1. Returning the child to the school immediately
13. The following are appropriate nursing diagnosis for with family support.
the client EXCEPT: 2. Calmly explaining why attendance in school is
necessary
1. Ineffective individual coping 3. Allowing the child to enter the school before
2. Alteration in comfort, pain the other children
3. Altered role performance 4. Allowing the parent to accompany the child in
4. Impaired social interaction the classroom

14. The following statements describe somatoform 20. A 10 year old child has very limited vocabulary and
disorders: interaction skills. She has an I.Q. of 45. She is diagnosed
to have Mental retardation of this classification:
1. Physical symptoms are explained by organic
causes 1. Profound
2. It is a voluntary expression of psychological 2. Mild
conflicts 3. Moderate
3. Expression of conflicts through bodily 4. Severe
symptoms
4. Management entails a specific medical 21. The nurse teaches the parents of a mentally
treatment retarded child regarding her care. The following
guidelines may be taught except:
15. What would be the best response to the client’s
repeated complaints of pain: 1. overprotection of the child
2. patience, routine and repetition
1. “I know the feeling is real tests revealed 3. assisting the parents set realistic goals
negative results.” 4. giving reasonable compliments
2. “I think you’re exaggerating things a little bit.”
3. “Try to forget this feeling and have activities to 22. The parents express apprehensions on their ability
take it off your mind” to care for their maladaptive child. The nurse identifies
4. “So tell me more about the pain” what nursing diagnosis:

16. Situation: A nurse may encounter children with 1. hopelessness


mental disorders. Her knowledge of these various 2. altered parenting role
disorders is vital. When planning school interventions 3. altered family process
for a child with a diagnosis of attention deficit 4. ineffective coping
hyperactivity disorder, a guide to remember is to:
23. A 5 year old boy is diagnosed to have autistic
1. provide as much structure as possible for the disorder. Which of the following manifestations may be
child noted in a client with autistic disorder?
2. ignore the child’s overactivity.
1. argumentativeness, disobedience, angry
outburst 5) A 75 year old client is admitted to the hospital with
2. intolerance to change, disturbed relatedness, the diagnosis of dementia of the Alzheimer’s type and
stereotypes depression. The symptom that is unrelated to
3. distractibility, impulsiveness and overactivity depression would be?
4. aggression, truancy, stealing, lying
1. Apathetic response to the environment
24. The therapeutic approach in the care of an autistic 2. “I don’t know” answer to questions
child include the following EXCEPT: 3. Shallow of labile effect
4. Neglect of personal hygiene
1. Engage in diversionary activities when acting -
out 6) The client with confusion says to the nurse, “I haven’t
2. Provide an atmosphere of acceptance had anything to eat all day long. When are they going to
3. Provide safety measures bring breakfast?” The nurse saw the client in the day
4. Rearrange the environment to activate the child room eating breakfast with other clients 30 minutes
before this conversation. Which response would be best
25. According to Piaget a 5 year old is in what stage of for the nurse to make?
development:
1. “You know you had breakfast 30 minutes ago.”
1. Sensory motor stage 2. “I am so sorry that they didn’t get you
2. Concrete operations breakfast. I’ll report it to the charge nurse.”
3. Pre-operational 3. “I’ll get you some juice and toast. Would you
4. Formal operation like something else?”
4. “You will have to wait a while; lunch will be
here in a little while.”
Exam for Psychiatric Nursing
Alzheimer’s Disease 7) The nurse is caring for a client with stage III
1) The client with Alzheimer’s disease is being assisted Alzheimer’s disease. A characteristic of this stage is:
with activities of daily living when the nurse notes that
the client uses her toothbrush to brush her hair. The 1. Memory loss
nurse is aware that the client is exhibiting: 2. Failing to recognize familiar objects
3. Wandering at night
1. Agnosia 4. Failing to communicate
2. Apraxia
3. Anomia 8) The primary nursing intervention in working with a
4. Aphasia client with moderate stage dementia is ensuring that
the client:
2) A client with Alzheimer’s disease is awaiting
placement in a skilled nursing facility. Which long-term 1. receives adequate nutrition and hydration
plans would be most therapeutic for the client? 2. will reminisce to decrease isolation
3. remains in a safe and secure environment
1. Placing mirrors in several locations in the home 4. independently performs self care
2. Placing a picture of herself in her bedroom
3. Placing simple signs to indicate the location of 9) During the evaluation of the quality of home care for
the bedroom, bathroom, and so on a client with Alzheimer’s disease, the priority for the
4. Alternating healthcare workers to prevent nurse is to reinforce which statement by a family
boredom member?

3) The client with dementia is experiencing confusion 1. At least 2 full meals a day is eaten.
late in the afternoon and before bedtime. The nurse is 2. We go to a group discussion every week at our
aware that the client is experiencing what is known as: community center.
3. We have safety bars installed in the bathroom
1. Chronic fatigue syndrome and have 24 hour alarms on the doors.
2. Normal aging 4. The medication is not a problem to have it
3. Sundowning taken 3 times a day.
4. Delusions
10) Signs of Alzheimer’s include which of these
4) Which age group has the highest rate of Alzheimer’s symptoms?
cases reported?
1. Loss of memory
1. 85 and older 2. Increase in irritability
2. 74 to 84 3. Restlessness
3. 65 to 74 4. All of the above
4. 55 to 65
11) Which neurotransmitter has been implicated in the
development of Alzheimer’s disease? 18) Rosana is in the second stage of Alzheimer’s disease
who appears to be in pain. Which question by Nurse
1. Acetylcholine Jenny would best elicit information about the pain?
2. Dopamine
3. Epinephrine “Where is your pain located?”
4. Serotonin “Do you hurt? (pause) “Do you hurt?”
“Can you describe your pain?”
12) Alzheimer’s is an INSIDIOUS disease. This means: “Where do you hurt?”
19) Rosana is in the second stage of Alzheimer’s disease
1. that it is terminal who appears to be in pain. Which question by Nurse
2. that is can be cured Jenny would best elicit information about the pain?
3. that it sneaks up on a person over time
4. that it only affects the elderly 1. “Where is your pain located?”
5. none of the above 2. “Do you hurt? (pause) “Do you hurt?”
3. “Can you describe your pain?”
13) Edward, a 66 year old client with slight memory 4. “Where do you hurt?”
impairment and poor concentration is diagnosed with
primary degenerative dementia of the Alzheimer’s type. 20) How is Alzheimer’s diagnosed?
Early signs of this dementia include subtle personality
changes and withdrawal from social interactions. To 1. Mental-status tests
assess for progression to the middle stage of 2. Blood tests
Alzheimer’s disease, the nurse should observe the client 3. Neurological tests
for: 4. All of the above

1. Occasional irritable outbursts. 21) The usual span of years that Alzheimer’s may
2. Impaired communication. progress in the patient is:
3. Lack of spontaneity.
4. Inability to perform self-care activities. 1. three to five years
2. two to twenty years
14) Which of the following is not directly related with 3. fifty to sixty years
Alzheimer’s disease? 4. 6 months to one year
5. eight to ten years
1. Senile plaques
2. Diabetes mellitus 22) Scientists believe that _________________ develop
3. Tangles in the brain of an Alzheimer’s patient, and may be a
4. Dementia cause of the disease.

15) Alzheimer’s is the most common form of which of 1. cholesterols


these? 2. tumors
3. ruptured blood vessels
1. Malnutrition 4. plaques and tangles
2. Dementia
3. Fatigue 23) To encourage adequate nutritional intake for a
4. Psychosis female client with Alzheimer’s disease, the nurse
should:
16) Which nursing intervention is most appropriate for a
client with Alzheimer’s disease who has frequent 1. stay with the client and encourage him to eat.
episodes emotional lability? 2. help the client fill out his menu.
3. give the client privacy during meals.
1. Attempt humor to alter the client mood. 4. fill out the menu for the client.
2. Explore reasons for the client’s altered mood.
3. Reduce environmental stimuli to redirect the 24) A 93 year-old female with a history of Alzheimer’s
client’s attention. Disease gets admitted to an Alzheimer’s unit. The
4. Use logic to point out reality aspects. patient has exhibited signs of increased confusion and
limited stability with gait. Moreover, the patient is
17) Which of the following is the most common cause of refusing to use a w/c. Which of the following is the most
dementia among elderly persons? appropriate course of action for the nurse?

1. Parkinson’s disease 1. Recommend the patient remain in her room at


2. Multiple sclerosis all times.
3. Amyotrophic lateral sclerosis (Lou Gerhig’s 2. Recommend family members bring pictures to
disease) the patient’s room.
4. Alzheimer’s disease
3. Recommend a speech therapy consult to the 3. Remain calm and talk quietly to the client.
doctor. 4. Call the doctor and request an order for
4. Recommend the patient attempt to walk sedation.
pushing the w/c for safety.
25) The doctor has prescribed Exelon (rivastigmine) for 32) Thomas Elison is a 79 year old man who is admitted
the client with Alzheimer’s disease. Which side effect is with diagnosis of dementia. The doctor orders a series
most often associated with this drug? of laboratory tests to determine whether Mr. Elison’s
dementia is treatable. The nurse understands that the
1. Urinary incontinence most common cause of dementia in this population is:
2. Headaches
3. Confusion 1. AIDS
4. Nausea 2. Alzheimer’s disease
3. Brain tumors
26) A patient with Stage One Alzheimers might exhibit 4. Vascular disease
these behaviors:
33) A patient who has been admitted to the medical
1. forgetting names unit with new-onset angina also has a diagnosis of
2. missing appointments Alzheimer’s disease. Her husband tells you that he
3. getting lost while driving rarely gets a good night’s sleep because he needs to be
4. all of the above sure she does not wander during the night. He insists on
5. none of the above checking each of the medications you give her to be
sure they are the same as the ones she takes at home.
27) Which of the following diseases has not been Based on this information, which nursing diagnosis is
directly linked with Bell’s palsy? most appropriate for this patient?

1. AIDS 1. Decreased Cardiac Output related to poor


2. Diabetes myocardial contractility
3. Lyme disease 2. Caregiver Role Strain related to continuous
4. Alzheimer’s disease need for providing care
3. Ineffective Therapeutic Regimen Management
28) The symptom of dementia that involved a more related to poor patient memory
confused state after dark is called: 4. Risk for Falls related to patient wandering
behavior during the night
1. dark retreat
2. sundowning 34) Physiologically, what happens to the brain as
3. agitation Alzheimer’s progresses?
4. dark reaction
1. Tissue swells
29) Which of these is the strongest risk factor for 2. Fluid collects
developing the Alzheimer’s disease? 3. Many cells die
4. Brain-stem atrophies
1. Heredity
2. Age 35) The nurse is aware that the following ways in
3. Exposure to toxins vascular dementia different from Alzheimer’s disease is:
4. None of the above
1. Vascular dementia has more abrupt onset
30) The priority of care for a client with Alzheimer’s 2. The duration of vascular dementia is usually
disease is brief
3. Personality change is common in vascular
1. Help client develop coping mechanism dementia
2. Encourage to learn new hobbies and interest 4. The inability to perform motor activities occurs
3. Provide him stimulating environment in vascular dementia
4. Simplify the environment to eliminate the need
to make chores 36) A 65 years old client is in the first stage of
Alzheimer’s disease. Nurse Patricia should plan to focus
31) An elderly client with Alzheimer’s disease becomes this client’s care on:
agitated and combative when a nurse approaches to
help with morning care. The most appropriate nursing 1. Offering nourishing finger foods to help
intervention in this situation would be to: maintain the client’s nutritional status.
2. Providing emotional support and individual
1. Tell the client family that it is time to get counseling.
dressed. 3. Monitoring the client to prevent minor illnesses
2. Obtain assistance to restrain the client for from turning into major problems.
safety.
4. Suggesting new activities for the client and
family to do together. 1. Deterioration of nasal septum
2. Acute fluid and electrolyte imbalances
37) A nurse caring to a client with Alzheimer’s disease 3. Extra pyramidal tract symptoms
overheard a family member say to the client, “if you 4. Esophageal varices
pee one more time, I won’t give you any more food and
drinks”. What initial action is best for the nurse to take? 3. A tentative diagnosis of opiate addiction, Nurse
Candy should assess a recently hospitalized client for
1. Take no action because it is the family member signs of opiate withdrawal. These signs would include:
saying that to the client
2. Talk to the family member and explain that 1. Rhinorrhea, convulsions, subnormal
what she/he has said is not appropriate for the temperature
client 2. Nausea, dilated pupils, constipation
3. Give the family member the number for an 3. Lacrimation, vomiting, drowsiness
Elder Abuse Hot line 4. Muscle aches, papillary constriction, yawning
4. Document what the family member has said
4. A 48 year old male client is brought to the psychiatric
38) Alzheimer’s disease is the secondary diagnosis of a emergency room after attempting to jump off a bridge.
client admitted with myocardial infarction. Which The client’s wife states that he lost his job several
nursing intervention should appear on this client’s plan months ago and has been unable to find another job.
of care? The primary nursing intervention at this time would be
to assess for:
1. Perform activities of daily living for the client to
decease frustration. 1. A past history of depression
2. Provide a stimulating environment. 2. Current plans to commit suicide
3. Establish and maintain a routine. 3. The presence of marital difficulties
4. Try to reason with the client as much as 4. Feelings of excessive failure
possible.
5. Before helping a male client who has been sexually
39) As the manager in a long-term-care (LTC) facility, assaulted, nurse Maureen should recognize that the
you are in charge of developing a standard plan of care rapist is motivated by feelings of:
for residents with Alzheimer’s disease. Which of these
nursing tasks is best to delegate to the LPN team 1. Hostility
leaders working in the facility? 2. Inadequacy
3. Incompetence
1. Check for improvement in resident memory 4. Passion
after medication therapy is initiated.
2. Use the Mini-Mental State Examination to 6. When working with children who have been sexually
assess residents every 6 months. abused by a family member it is important for the nurse
3. Assist residents to toilet every 2 hours to to understand that these victims usually are
decrease risk for urinary intolerance. overwhelmed with feelings of:
4. Develop individualized activity plans after
consulting with residents and family. 1. Humiliation
2. Confusion
40) The nurse would expect a client with early 3. Self blame
Alzheimer’s disease to have problems with: 4. Hatred

1. Balancing a checkbook. 7. Joy who has just experienced her second


2. Self-care measures. spontaneous abortion expresses anger towards her
3. Relating to family members. physician, the hospital and the “rotten nursing care”.
4. Remembering his own name When assessing the situation, the nurse recognizes that
the client may be using the coping mechanism of:
Psychiatric Nursing Practice Exam 3
1. Projection
1. Francis who is addicted to cocaine withdraws from 2. Displacement
the drug. Nurse Ron should expect to observe: 3. Denial
4. Reaction formation
1. Hyperactivity
2. Depression 8. The most critical factor for nurse Linda to determine
3. Suspicion during crisis intervention would be the client’s:
4. Delirium
1. Available situational supports
2. Nurse John is aware that a serious effect of inhaling 2. Willingness to restructure the personality
cocaine is? 3. Developmental theory
4. Underlying unconscious conflict 3. Delusion of grandeur
4. Delusion of persecution
9. Nurse Trish suggests a crisis intervention group to a
client experiencing a developmental crisis. These groups 15. Andy is admitted to the psychiatric unit with a
are successful because the: diagnosis of borderline personality disorder. Nurse
Hilary should expects the assessment to reveal:
1. Crisis intervention worker is a psychologist and
understands behavior patterns 1. Coldness, detachment and lack of tender
2. Crisis group supplies a workable solution to the feelings
client’s problem 2. Somatic symptoms
3. Client is encouraged to talk about personal 3. Inability to function as responsible parent
problems 4. Unpredictable behavior and intense
4. Client is assisted to investigate alternative interpersonal relationships
approaches to solving the identified problem
16. PROPRANOLOL (Inderal) is used in the mental health
10. Nurse Ronald could evaluate that the staff’s setting to manage which of the following conditions?
approach to setting limits for a demanding, angry client
was effective if the client: 1. Antipsychotic – induced akathisia and anxiety
2. Obsessive – compulsive disorder (OCD) to
1. Apologizes for disrupting the unit’s routine reduce ritualistic behavior
when something is needed 3. Delusions for clients suffering from
2. Understands the reason why frequent calls to schizophrenia
the staff were made 4. The manic phase of bipolar illness as a mood
3. Discuss concerns regarding the emotional stabilizer
condition that required hospitalizations
4. No longer calls the nursing staff for assistance 17. Which medication can control the extra pyramidal
effects associated with antipsychotic agents?
11. Nurse John is aware that the therapy that has the
highest success rate for people with phobias would be: 1. Clorazepate (Tranxene)
2. Amantadine (Symmetrel)
1. Psychotherapy aimed at rearranging 3. Doxepin (Sinequan)
maladaptive thought process 4. Perphenazine (Trilafon)
2. Psychoanalytical exploration of repressed
conflicts of an earlier development phase 18. Which of the following statements should be
3. Systematic desensitization using relaxation included when teaching clients about monoamine
technique oxidase inhibitor (MAOI) antidepressants?
4. Insight therapy to determine the origin of the
anxiety and fear 1. Don’t take aspirin or nonsteroidal anti-
inflammatory drugs (NSAIDs)
12. When nurse Hazel considers a client’s placement on 2. Have blood levels screened weekly for
the continuum of anxiety, a key in determining the leucopenia
degree of anxiety being experienced is the client’s: 3. Avoid strenuous activity because of the cardiac
effects of the drug
1. Perceptual field 4. Don’t take prescribed or over the counter
2. Delusional system medications without consulting the physician
3. Memory state
4. Creativity level 19. Kris periodically has acute panic attacks. These
attacks are unpredictable and have no apparent
13. In the diagnosis of a possible pervasive association with a specific object or situation. During an
developmental autistic disorder. The nurse would find it acute panic attack, Kris may experience:
most unusual for a 3 year old child to demonstrate:
1. Heightened concentration
1. An interest in music 2. Decreased perceptual field
2. An attachment to odd objects 3. Decreased cardiac rate
3. Ritualistic behavior 4. Decreased respiratory rate
4. Responsiveness to the parents
20. Initial interventions for Marco with acute anxiety
14. Malou with schizophrenia tells Nurse Melinda, “My include all except which of the following?
intestines are rotted from worms chewing on them.”
This 1. Touching the client in an attempt to comfort
statement indicates a: him
2. Approaching the client in calm, confident
1. Jealous delusion manner
2. Somatic delusion
3. Encouraging the client to verbalize feelings and 27. Nursing preparation for a client undergoing
concerns electroconvulsive therapy (ECT) resemble those used
4. Providing the client with a safe, quiet and for:
private place
21. Nurse Jessie is assessing a client suffering from 1. General anesthesia
stress and anxiety. A common physiological response to 2. Cardiac stress testing
stress and anxiety is: 3. Neurologic examination
4. Physical therapy
1. Uticaria
2. Vertigo 28. Jose who is receiving monoamine oxidase inhibitor
3. Sedation antidepressant should avoid tyramine, a compound
4. Diarrhea found in which of the following foods?

22. When performing a physical examination on a 1. Figs and cream cheese


female anxious client, nurse Nelli would expect to find 2. Fruits and yellow vegetables
which of the following effects produced by the 3. Aged cheese and Chianti wine
parasympathetic system? 4. Green leafy vegetables

1. Muscle tension 29. Erlinda, age 85, with major depression undergoes a
2. Hyperactive bowel sounds sixth electroconvulsive therapy (ECT) treatment. When
3. Decreased urine output assessing the client immediately after ECT, the nurse
4. Constipation expects to find:

23. Which of the following drugs have been known to 1. Permanent short-term memory loss and
be effective in treating obsessive-compulsive disorder hypertension
(OCD)? 2. Permanent long-term memory loss and
hypomania
1. Divalproex (depakote) and Lithium (lithobid) 3. Transitory short-term memory loss and
2. Chlordiazepoxide (Librium) and diazepam permanent long-term memory loss
(valium) 4. Transitory short and long term memory loss and
3. Fluvoxamine (Luvox) and clomipramine confusion
(anafranil)
4. Benztropine (Cogentin) and diphenhydramine 30. Barbara with bipolar disorder is being treated with
(benadryl) lithium for the first time. Nurse Clint should observe the
client for which common adverse effect of lithium?
24. Tony with agoraphobia has been symptom-free for 4
months. Classic signs and symptoms of phobia include: 1. Polyuria
2. Seizures
1. Severe anxiety and fear 3. Constipation
2. Withdrawal and failure to distinguish reality 4. Sexual dysfunction
from fantasy
3. Depression and weight loss 31. Nurse Fred is assessing a client who has just been
4. Insomnia and inability to concentrate admitted to the ER department. Which signs would
suggest an overdose of an antianxiety agent?
25. Which nursing action is most appropriate when
trying to diffuse a client’s impending violent behavior? 1. Suspiciousness, dilated pupils and incomplete
BP
1. Place the client in seclusion 2. Agitation, hyperactivity and grandiose ideation
2. Leaving the client alone until he can talk about 3. Combativeness, sweating and confusion
his feelings 4. Emotional lability, euphoria and impaired
3. Involving the client in a quiet activity to divert memory
attention
4. Helping the client identify and express feelings 32. Discharge instructions for a male client receiving
of anxiety and anger tricyclic antidepressants include which of the following
information?
26. Rosana is in the second stage of Alzheimer’s disease
who appears to be in pain. Which question by Nurse 1. Restrict fluids and sodium intake
Jenny would best elicit information about the pain? 2. Don’t consume alcohol
3. Discontinue if dry mouth and blurred vision
1. “Where is your pain located?” occur
2. “Do you hurt? (pause) “Do you hurt?” 4. Restrict fluid and sodium intake
3. “Can you describe your pain?”
4. “Where do you hurt?”
33. Important teaching for women in their childbearing The client’s employer expects the client to return to
years who are receiving antipsychotic medications work following inpatient treatment. The client tells the
includes which of the following? nurse, “I’m no good. I’m a failure”. According to
cognitive theory, these statements reflect:
1. Increased incidence of dysmenorrhea while
taking the drug 1. Learned behavior
2. Occurrence of incomplete libido due to 2. Punitive superego and decreased self-esteem
medication adverse effects 3. Faulty thought processes that govern behavior
3. Continuing previous use of contraception during 4. Evidence of difficult relationships in the work
periods of amenorrhea environment
4. Instruction that amenorrhea is irreversible
40. The nurse describes a client as anxious. Which of the
34. A client refuses to remain on psychotropic following statement about anxiety is true?
medications after discharge from an inpatient
psychiatric unit. Which information should the 1. Anxiety is usually pathological
community health nurse assess first during the initial 2. Anxiety is directly observable
follow-up with this client? 3. Anxiety is usually harmful
4. Anxiety is a response to a threat
1. Income level and living arrangements
2. Involvement of family and support systems 41. A client with a phobic disorder is treated by
3. Reason for inpatient admission systematic desensitization. The nurse understands that
4. Reason for refusal to take medications this approach will do which of the following?

35. The nurse understands that the therapeutic effects 1. Help the client execute actions that are feared
of typical antipsychotic medications are associated with 2. Help the client develop insight into irrational
which neurotransmitter change? fears
3. Help the client substitutes one fear for another
1. Decreased dopamine level 4. Help the client decrease anxiety
2. Increased acetylcholine level
3. Stabilization of serotonin 42. Which client outcome would best indicate
4. Stimulation of GABA successful treatment for a client with an antisocial
personality disorder?
36. Which of the following best explains why tricyclic
antidepressants are used with caution in elderly 1. The client exhibits charming behavior when
patients? around authority figures
2. The client has decreased episodes of impulsive
1. Central Nervous System effects behaviors
2. Cardiovascular system effects 3. The client makes statements of self-satisfaction
3. Gastrointestinal system effects 4. The client’s statements indicate no remorse for
4. Serotonin syndrome effects behaviors

37. A client with depressive symptoms is given 43. The nurse is caring for a client with an autoimmune
prescribed medications and talks with his therapist disorder at a medical clinic, where alternative medicine
about his belief that he is worthless and unable to cope is used as an adjunct to traditional therapies. Which
with life. Psychiatric care in this treatment plan is based information should the nurse teach the client to help
on which framework? foster a sense of control over his symptoms?

1. Behavioral framework 1. Pathophysiology of disease process


2. Cognitive framework 2. Principles of good nutrition
3. Interpersonal framework 3. Side effects of medications
4. Psychodynamic framework 4. Stress management techniques

38. A nurse who explains that a client’s psychotic 44. Which of the following is the most distinguishing
behavior is unconsciously motivated understands that feature of a client with an antisocial personality
the client’s disordered behavior arises from which of disorder?
the following?
1. Attention to detail and order
1. Abnormal thinking 2. Bizarre mannerisms and thoughts
2. Altered neurotransmitters 3. Submissive and dependent behavior
3. Internal needs 4. Disregard for social and legal norms
4. Response to stimuli
45. Which nursing diagnosis is most appropriate for a
39. A client with depression has been hospitalized for client with anorexia nervosa who expresses feelings of
treatment after taking a leave of absence from work. guilt about not meeting family expectations?
"Yes, I see. Go on."
1. Anxiety
2. Disturbed body image 2.A nurse states to a client, "Things will look better
3. Defensive coping tomorrow after a good night's sleep." This is an example
4. Powerlessness of which communication technique?
The nontherapuetic technique of giving false
46. A nurse is evaluating therapy with the family of a reassurance
client with anorexia nervosa. Which of the following
would indicate that the therapy was successful? 3.A client diagnosed with post-traumatic stress disorder
is admitted to an inpatient psychiatric unit for
1. The parents reinforced increased decision evaluation and medication stabilization. Which
making by the client therapeutic communication technique used by the
2. The parents clearly verbalize their expectations nurse is an example of broad opening?
for the client "what would you like to talk about?"
3. The client verbalizes that family meals are now
enjoyable 4.An instructor is correcting a nursing student's clinical
4. The client tells her parents about feelings of worksheet. Which instructor statement is the best
low-self esteem example of effective feedback?

47. A client with dysthymic disorder reports to a nurse A. "Why did you use the client's name on your clinical
that his life is hopeless and will never improve in the worksheet?"
future. How can the nurse best respond using a B. "You were very careless to refer to your client by
cognitive approach? name on your clinical worksheet."
C. "Surely you didn't do this deliberately, but you
1. Agree with the client’s painful feelings breeched confidentiality by using the client's name."
2. Challenge the accuracy of the client’s belief D. "It is disappointing that after being told, you're still
3. Deny that the situation is hopeless using client names on your worksheet."
4. Present a cheerful attitude
5.After assertiveness training, a formerly passive client
48. A client with major depression has not verbalized appropriately confronts a peer in group therapy. The
problem areas to staff or peers since admission to a group leader states, "I'm so proud of you for being
psychiatric unit. Which activity should the nurse assertive. You are so good!" Which communication
recommend to help this client express himself? technique has the leader employed?

1. Art therapy in a small group A. The nontherapeutic technique of giving approval


2. Basketball game with peers on the unit B. The nontherapeutic technique of interpreting
3. Reading a self-help book on depression C. The therapeutic technique of presenting reality
4. Watching movie with the peer group D. The therapeutic technique of making observations
49. The home health psychiatric nurse visits a client
with chronic schizophrenia who was recently discharged 6.A client is diagnosed with dependent personality
after a prolong stay in a state hospital. The client lives in disorder states, "Do you think I should move from a
a boarding home, reports no family involvement, and parent's house and get a job?" Which nursing response
has little social interaction. The nurse plan to refer the is most appropriate?
client to a day treatment program in order to help him "Let's discuss and explore all of your options"
with:
7.When Interviewing a client, which nonverbal behavior
1. Managing his hallucinations should a nurse employ?
2. Medication teaching Sitting squarely, facing the client
3. Social skills training
4. Vocational training 8.A mother rescues two of her four children from a
house fire. In the emergency department, she cries, "I
50. Which activity would be most appropriate for a should have gone back in to get them. I should have
severely withdrawn client? died, not them." What is the nurses best response?
The smoke was too thick, you couldn't have gone back
1. Art activity with a staff member in.
2. Board game with a small group of clients
3. Team sport in the gym 9.A newly admitted client diagnosed with obsessive
4. Watching TV in the dayroom compulsive disorder washes hands continually. This
behavior presents unit activity attendance. Which
nursing statement best addresses the situation.
NSG 232 "Let's figure out a way for you to attend unit activities
1.The nurse is interviewing a newly admitted client. and still wash your hands"
Which nursing statement is an example of offering a
general lead?
10.Which example of therapeutic communication 6.The nurse is assigned to care for a client who is
technique would be effective in the planning phase of agitated. On entering the room, the client screams,
the nursing process? "Why don't you just leave me alone?" The nurse should
"We've discussed past coping skills. Let's see if these make which therapeutic response to the client?
comping skills can be effective now." "I can see that you are upset. I'll be back in a few
minutes to see how you are doing."
11.A student nurse tells the instructor, "I am concerned
that when a client asks me for advice I won't have a 7.The nurse is providing care to a client admitted to the
good solution." Which should be the nursing instructor's hospital with a diagnosis of anxiety disorder. The nurse
best response? is talking with the client, and the client says, "I have a
"Remember, clients, not nurses are responsible for their secret that I want to tell you. You won't tell anyone
own choices and decisions about it, will you?" Which is the appropriate nursing
response?
12.Which nursing statement is a good example of the "I can see that you are upset. I'll be back in a few
therapeutic communication technique of focusing? minutes to see how you are doing."
Your counseling session is in 30 minutes. I'll stay with
you until then. 8.A client is admitted to a psychiatric unit for treatment
of a psychotic disorder. The client is at the locked exit
13.During a nurse-client interaction, which nursing door and is shouting, "Let me out! There's nothing
statement may belittle the client's feelings and wrong with me! I don't belong here!" The nurse
concerns? identifies this behavior as which?
You are jumping to conclusions denial

14.Which nursing response is an example of the 9.The nurse is assigned to care for a client experiencing
nontherapeutic communication black of requesting an disturbed thought processes. The nurse is told that the
explanation client believes that the food is being poisoned. Which
"Can you tell me why you said that?" communication technique should the nurse plan to use
to encourage the client to eat?
open-ended questions and silence

Mental Health Nursing evolve 10.The nurse is assisting with the data collection on a
1.A client comes to the clinic after losing all of his client admitted to the psychiatric unit. After review of
personal belongings in a hurricane. The nurse notes that the data obtained, the nurse should identify which as a
the client is coping ineffectively. Which is the least priority concern?
realistic goal for this client? The client's report of self-destructive thoughts
The client will stop blaming himself for the lack of
insurance 11.A client says to the nurse, "I'm going to die, and I
wish my family would stop hoping for a 'cure'! I get so
2.The nurse is caring for a client diagnosed with angry when they carry on like this! After all, I'm the one
catatonic stupor. The client is lying on the bed, with the who's dying." Which therapeutic response should the
body pulled into a fetal position. The appropriate nurse make to the client?
nursing intervention is which? You're feeling angry that your family continues to hope
Sit beside the client in silence and verbalize occasional for you to be 'cured'?"
open-ended questions.
12.A long-term care resident with a history of paranoid
3.The nurse is reviewing the health care record of a schizophrenia refuses to eat and tells the nurse that she
client admitted to the psychiatric unit. The nurse notes believes that someone is poisoning the food. The nurse
that the admission nurse has documented that the should make which appropriate response to the client?
client is experiencing anxiety as a result of a situational "It must be frightening to you. Has something made you
crisis. The nurse should determine that this type of crisis feel that your food is poisoned?"
could be caused by which?
The death of a loved one 13.Laboratory work is prescribed for a client who has
been experiencing delusions. When the laboratory
4.Which data indicates to the nurse that a client may be technician approaches the client to obtain a specimen
experiencing ineffective coping following the loss of her of the client's blood, the client begins to shout, "You're
spouse? all vampires. Let me out of here!" The nurse present at
Constantly neglects personal grooming the time should respond by stating which?
"Are you fearful and think that others may want to hurt
5.Which client is most likely at risk to become a victim you?"
of elder abuse?
A 90-year-old woman with advanced Parkinson's 14.The nurse is assigned to care for a client admitted to
disease the hospital after sustaining an injury from a house fire.
The client attempted to save a neighbor involved in the
fire, but despite the client's efforts, the neighbor died.
Which action should the nurse take to enable the client dance at a long-term care facility. The nurse should use
to work through the meaning of the crisis? which piece of information when approaching the client
Inquiring about the client's feelings that may affect about this behavior?
coping Individuals with Alzheimer's disease have difficulty
tolerating excess stimulation and changes in routine.
15.The nurse receives a telephone call from a male
client who states that he wants to kill himself and has a 24.The nurse notices a "paranoid stare" during a
bottle of sleeping pills in front of him. Which would be conversation with the client diagnosed with
the best response by the nurse? posttraumatic stress disorder (PTSD). The client then
Keep the client talking and signal to another staff begins to fidget and gets up to pace around the room.
member to send help to the client. Which action by the nurse would be most beneficial?
Share the observation with the client and help the client
17.SELECT ALL THAT APPLY recognize his or her feelings.
The nurse is working with an older client who has a
diagnosis of depression. To work most effectively with 25.The nurse working in a mental health unit hears that
this client, the nurse recalls that which information is a client has been experiencing "flashbacks." The nurse
accurate regarding depression and the older client? interprets that this client is exhibiting a sign of which
Select all that apply. condition?
Suicide is a frequent cause of death among the older Posttraumatic stress disorder (PTSD)
population.
Some indications of dementia may actually originate as 26.A confused and disoriented client is admitted to the
depression. psychiatric unit diagnosed with posttraumatic stress
Depression in an older person is likely to have physical disorder (PTSD). The nurse initially plans to take which
manifestations. action with this client?
Accept the client as a person and make the client feel
18.The nurse is caring for a client with schizophrenia safe.
who states, "I decided not to take my medication
because I realize that it really can't help me. Only I can 27.A client with delirium becomes agitated and
help me." Which nursing response should be confused at night. The best initial intervention by the
therapeutic? nurse is which?
Do you recall needing to be hospitalized because you Use a night light and turn off the television.
stopped your medication?"
28.The licensed practical nurse is assisting the
19.The nurse enters a client's room, and the client registered nurse in admitting a client with an
immediately demands to be released from the hospital. exacerbation of schizophrenia and knows that which
On review of the client's record, the nurse notes that signs/symptoms displayed by the client are considered
the client was admitted 2 days ago for treatment of an positive symptoms? Select all that apply.
anxiety disorder and that the admission was a voluntary Hallucinations
admission. The nurse reports the findings to the Delusions
registered nurse (RN) and expects that the RN will take Neologisms
which action?
Contact the health care provider (HCP).

20.A client is admitted to the psychiatric nursing unit.


When collecting data from the client, the nurse notes
that the client was admitted on an involuntary status.
Based on this type of admission, the nurse expects
which?
The client presents a harm to self.

21.The nurse in a psychiatric unit is assigned to care for


a client admitted to the unit 2 days ago. On review of
the client's record, the nurse notes that the admission
was a voluntary admission. Based on this type of
admission, the nurse should expect which?
The client will participate in the treatment plan

22.The nurse is preparing a client for the termination


phase of the nurse-client relationship. Which task
should the nurse appropriately plan for this phase?
Assist in making appropriate referrals

23.A client with Alzheimer's disease became very


agitated when a group of children came to sing and
should the nurse expect in this client's plan of care to
Exam for Psychiatric Nursing promote independence?
1.A client begins experiencing physical symptoms
believed to be caused by psychological distress. This 1. Spending long periods of unscheduled time with the
client is most likely experiencing which disorder? client
2. Scheduling competitive activities so the client can test
1. Pain disorder skills
2. Somatization 3. Helping the client identify preferences, such as
3. Conversion disorder choosing which clothing item to wear
4. Body dysmorphic disorder 4. Avoiding discussing the client's feelings of
helplessness
2.An unemployed woman, age 24, seeks help because
she feels depressed and abandoned and doesn't know 8.A 75-year-old client who was admitted to the hospital
what to do with her life. Last week, her boyfriend broke with a stroke informs the nurse that he doesn't want to
up with her after she drove his car into a tree after an be kept alive with machines. He wants to make sure
argument. The client's initial diagnosis is borderline that everyone knows his wishes. Which action should
personality disorder. Which nursing observations the nurse take?
support this diagnosis?
1. Contact the social services department to make
1. Flat affect, social withdrawal, and unusual dress arrangements for the client to complete a living will.
2. Suspiciousness, hypervigilance, and emotional 2. Notify the physician so that he can place a do-not-
coldness resuscitate order on the client's medical record.
3. Lack of self-esteem, strong dependency needs, and 3. Make arrangements for the client to receive
impulsive behavior information about advance directives.
4. Insensitivity to others, sexual acting out, and violence 4. Explain that his condition is stable, so he doesn't
need to be concerned at this time.
3.Which of the following indications is the appropriate
use for electroconvulsive therapy (ECT)?
9.On admission to the inpatient psychiatric unit, a
1. Severe agitation client's facial expression indicates severe panic. The
2. Antisocial behavior client repeatedly states, "I know the police are going to
3. Noncompliance with treatment shoot me. They found out that I'm the child of the
4. Major depression with psychotic features devil." What should the nurse say to initiate a
therapeutic relationship with the client?
5.The nurse is teaching a client about the
antidepressant amitriptyline (Elavil). Which points 1. "You certainly look stressed. Can you tell me about
should she include in her teaching plan? the upsetting events that have occurred in your life
Select all that apply: recently?"
2. "Hello, my name is ___. I'm a nurse, and I'll care for
1. Smoking may lower the drug level. you when I'm on duty. Should I call you ___, or do you
2. Avoid prolonged exposure to the sun. prefer something else?"
3. Avoid drinking grapefruit juice because it interferes
with the drug's metabolism. 3. "You're having very frightening thoughts. I'll help you
4. The drug's effects may not be seen for 4 weeks. find ways to cope with this scary thinking."
5. Increase fluid and fiber intake to prevent 4. "Hello, ___. I'm going to be caring for you while I'm
constipation. on duty. You look very frightened, but I'm sure you'll
feel better by tomorrow."
6.A client with delusional thinking shows a lack of
interest in eating at meal times. She states that she is
unworthy of eating and that her children will die if she 10.A client is transferred to the locked psychiatric unit
eats. Which nursing action would be appropriate for from the emergency department after attempting
this client? suicide by taking 200 acetaminophen (Tylenol) tablets.
Now the client is awake and alert but refuses to speak
1. Telling the client that she may become sick and die with the nurse. In this situation, the nurse's first priority
unless she eats is to:
2. Paying special attention to the client's rituals and
emotions associated with meals 1. establish a rapport to foster trust.
3. Restricting the client's access to food except at 2. place the client in full leather restraints.
specified meal and snack times 3. try to communicate with the client in writing.
4. Encouraging the client to express her feelings at meal 4. ensure safety by initiating suicide precautions.
times

7.The nurse is assigned to care for a client with 11.A client with paranoid personality disorder is
dependent personality disorder. Which intervention admitted to a psychiatric facility. Which remark by the
nurse would best establish trust and rapport and 17.Which action should a nurse recommend be included
encourage the client to confide in the nurse? in a plan of care for clients diagnosed with
schizophrenia and are on antipsychotic medication?
1. "I get upset once in a while, too."
2. "I know just how you feel. I'd feel the same way in 1) limit fluid intake to prevent peripheral edema
your situation." 2) frequent tooth brushing to prevent gingival
3. "I worry, too, when I think people are talking about hyperplasia
me." 3) document visual and auditory hallucinations
4. "At times, it's normal not to trust anyone." 4) limit caloric and fat intake to minimize weight gain

12.The nurse receives a physician's order to administer Psychiatric Nursing Practice Test 8
1,000 ml of normal saline solution I.V. over 8 hours to a 1.A woman is admitted to the psychiatric emergency
client who recently had a stroke. What should the drip department. Her significant other reports that she has
rate be if the drop factor of the tubing is 15 gtt/ml? difficulty sleeping, has poor judgment, and is incoherent
31. 1000ml / 480 minutes x 15 drops/ml = 31 gtt/min. at times. The client's speech is rapid and loose. She
reports being a special messenger from the Messiah.
13.A child, age 5, is diagnosed with mycoplasmal She has a history of depressed mood for which she has
pneumonia and has a persistent productive cough. been taking an antidepressant. The nurse suspects
When monitoring the child's respirations, the nurse which diagnosis?
should keep in mind that children normally use which Bipolar illness
muscles to breathe?
2. A 26-year-old client is admitted to the psychiatric unit
1. Accessory with acute onset of schizophrenia. His physician
2. Thoracic prescribes the phenothiazine chlorpromazine
3. Abdominal (Thorazine), 100 mg by mouth four times per day.
4. Intercostal Before administering the drug, the nurse reviews the
client's medication history. Concomitant use of which
14.A client with herpes zoster is prescribed acyclovir drug is likely to increase the risk of extrapyramidal
(Zovirax), 200 mg by mouth every 4 hours while awake. effects?
The nurse should inform the client that this drug may Droperidol (Inapsine)
cause:
3. Propranolol (Inderal) is used in the mental health
1. palpitations. setting to manage which of the following conditions?
2. dizziness. Antipsychotic-induced akathisia and anxiety
3. diarrhea.
4. metallic taste. 4. A client with schizophrenia who receives
fluphenazine (Prolixin) develops pseudoparkinsonism
15.An 8-year-old client has tested positive for West Nile and akinesia. What drug would the nurse administer to
virus infection. The nurse suspects the client has the minimize extrapyramidal symptoms?
severe form of the disease when she recognizes which Benztropine (Cogentin)
signs and symptoms?
5.The etiology of schizophrenia is best described by:
1. Fever, rash, and malaise A combination of biological, psychological, and
2. Anorexia, nausea, and vomiting environmental factors.
3. Fever, muscle weakness, and change in mental status
4. Fever, lymphadenopathy, and rash 6. Which of the following is one of the advantages of
the newer antipsychotic medication risperidone
(Risperdal)?
16.A client is recovering in the labor and delivery area A lower incidence of extrapyramidal effects
after delivering a 6-lb, 3-oz boy. On assessment, the
nurse finds that the client's fundus is firm and located 7. A client with a diagnosis of paranoid schizophrenia
two fingerbreadths below the umbilicus. Although she comments tothe nurse, "How do I know what is really in
didn't have an episiotomy, her perineal pad reveals a those pills?" Which of the following is the best
steady trickle of blood. What is the probable cause of response?
these assessment findings? Allow him to open the individual wrappers of the
medication.
1. A boggy uterus
2. Normal involution 8.A client's medication order reads, "Thioridazine
3. A vaginal laceration (Mellaril) 200 mg P.O. q.i.d. and 100 mg P.O. p.r.n." The
4. A clotting problem nurse should:
Question the physician about the order.
9The nurse is assigned to a client with catatonic the nurse would be most helpful in dealing with the
schizophrenia. Which intervention should the nurse client's anger?
include in the client's plan of care?
Meeting all of the client's physical needs "You had to wait. Can we talk about how this is making
you feel right now?"
10Since admission 4 days ago, a client has refused to 19A client is unable to get out of bed and get dressed
take a shower, stating, "There are poison crystals unless the nurse prompts every step. This is an example
hidden in the showerhead. They'll kill me if I take a of which behavior?
shower." Which nursing action is most appropriate? Avolition
Accepting these fears and allowing the client to take a
sponge bath 20. Positive symptoms of schizophrenia include which of
the following?
11. Which information is most important for the nurse Hallucinations, delusions, and disorganized thinking
to include in a teaching plan for a schizophrenic client
taking clozapine (Clozaril)? 21A client tells the nurse that people from Mars are
Report a sore throat or fever to the physician going to invade the earth. Which response by the nurse
immediately. would be most therapeutic?

12The nurse is providing care to a client with a catatonic "That must be frightening to you. Can you tell me how
type of schizophrenia who exhibits extreme negativism. you feel about it?"
To help the client meet his basic needs, the nurse
should: 22. How soon after chlorpromazine (Thorazine)
Tell the client specifically and concisely what needs to administration should the nurse expect to see a client's
be done. delusional thoughts and hallucinations eliminated?
Several weeks
13A client with paranoid schizophrenia is admitted to
the psychiatric unit of a hospital. Nursing assessment 23.Important teaching for clients receiving antipsychotic
should include careful observation of the client's: medication such as haloperidol (Haldol) includes which
Thinking, perceiving, and decision-making skills. of the following instructions?

14A client is admitted to the psychiatric unit with a Use sunscreen because of photosensitivity.
tentative diagnosis of psychosis. Her physician
prescribes the phenothiazine thioridazine (Mellaril) 50 24. Every day for the past 2 weeks, a client with
mg by mouth three times per day. Phenothiazines differ schizophrenia stands up during group therapy and
from central nervous system (CNS) depressants in their screams, "Get out of here right now! The elevator
sedative effects by producing: bombs are going to explode in 3 minutes!" The next
A calming effect from which the client is easily aroused. time this happens, how should the nurse respond?

15The nurse is caring for a client who experiences false "I know you think there are bombs in the elevator, but
sensory perceptions with no basis in reality. These there aren't."
perceptions are known as:
Hallucinations 25. A client with schizophrenia is receiving antipsychotic
medication. Which nursing diagnosis may be
16An agitated and incoherent client, age 29, comes to appropriate for this client?
the emergency department with complaints of visual Ineffective protection related to blood dyscrasias
and auditory hallucinations. The history reveals that the
client was hospitalized for paranoid schizophrenia from 26. A client with paranoid personality disorder is
ages 20 to 21. The physician prescribes haloperidol admitted to a psychiatric facility. Which remark by the
(Haldol), 5 mg I.M. The nurse understands that this drug nurse would best establish rapport and encourage the
is used in this client to treat: client to confide in the nurse?
Psychosis “I get upset once in a while, too."

17The nurse is caring for a client with schizophrenia. 27Important teaching for a client receiving risperidone
Which of the following outcomes is the least desirable? (Risperdal) would include advising the client to:
Notify the physician if the client notices an increase in
The client spends more time by himself. bruising.
28A client with schizophrenia tells the nurse he hears
the voices of his dead parents. To help the client ignore
18A client with borderline personality disorder becomes
the voices, the nurse should recommend that he:
angry when he is told that today's psychotherapy
session with the nurse will be delayed 30 minutes
Listen to a personal stereo through headphones and
because of an emergency. When the session finally
sing along with the music.
begins, the client expresses anger. Which response by
29Yesterday, a client with schizophrenia began
treatment with haloperidol (Haldol). Today, the nurse 39The nurse is teaching a psychiatric client about her
notices that the client is holding his head to one side prescribed drugs, chlorpromazine and benztropine.
and complaining of neck and jaw spasms. What should Why is benztropine administered?
the nurse do? To reduce extrapyramidal symptoms
Evaluate the client for adverse reactions to haloperidol.
40A client with paranoid schizophrenia repeatedly uses
30. A client diagnosed with schizoaffective disorder is profanity during an activity therapy session. Which
suffering from schizophrenia with elements of which of response by the nurse would be most appropriate?
the following disorders?
Mood disorder "Your behavior won't be tolerated. Go to your room
immediately."
31During a group therapy session in the psychiatric unit,
a client constantly interrupts with impulsive behavior 41. Hormonal effects of the antipsychotic medications
and exaggerated stories that cast her as a hero or include which of the following?
princess. She also manipulates the group with attention-
seeking behaviors, such as sexual comments and angry Retrograde ejaculation and gynecomastia
outbursts. The nurse realizes that these behaviors are
typical of: 42. The nurse formulates a nursing diagnosis of
Histrionic personality disorder. Impaired verbal communication for a client with
schizotypal personality disorder. Based on this nursing
32The nurse is aware that antipsychotic medications diagnosis, which nursing intervention is most
may cause which of the following adverse effects? appropriate?
Lower seizure threshold Establishing a one-on-one relationship with the client

33A client is admitted to the psychiatric hospital with a 43. Which nonantipsychotic medication is used to treat
diagnosis of catatonic schizophrenia. During the some clients with schizoaffective disorder?
physical examination, the client's arm remains
outstretched after the nurse obtains the pulse and Lithium carbonate (Lithane)
blood pressure, and the nurse must reposition the arm.
This client is exhibiting: 44. When teaching the family of a client with
Waxy flexibility. schizophrenia, the nurse should provide which
information?
34A client with chronic schizophrenia receives 20 mg of
fluphenazine decanoate (Prolixin Decanoate) by I.M. Support is available to help family members meet their
injection. Three days later, the client has muscle own needs.
contractions that contort the neck. This client is
exhibiting which extrapyramidal reaction? 45. A client is admitted with a diagnosis of delusions of
Dystonia grandeur. This diagnosis reflects a belief that one is:
Highly important or famous
35A client is about to be discharged with a prescription
for the antipsychotic agent haloperidol (Haldol), 10 mg 46. A client, age 36, with paranoid schizophrenia
by mouth twice per day. During a discharge teaching believes the room is bugged by the Central Intelligence
session, the nurse should provide which instruction to Agency and that his roommate is a foreign spy. The
the client? client has never had a romantic relationship, has no
Apply a sunscreen before being exposed to the sun. contact with family members, and hasn't been
employed in the last 14 years. Based on Erikson's
36A client receiving fluphenazine decanoate (Prolixin theories, the nurse should recognize that this client is in
Decanoate) therapy develops pseudoparkinsonism. The which stage of psychosocial development?
physician is likely to prescribe which drug to control this Trust versus mistrust
extrapyramidal effect?
Amantadine (Symmetrel)
47. A client with a history of medication noncompliance
37Drug therapy with thioridazine (Mellaril) shouldn't is receiving outpatient treatment for chronic
exceed a daily dose of 800 mg to prevent which adverse undifferentiated schizophrenia. The physician is most
reaction? likely to prescribe which medication for this client?
Retinal pigmentation
Fluphenazine decanoate (Prolixin Decanoate)
38A client with persistent, severe schizophrenia has
been treated with phenothiazines for the past 17 years. 48. A client begins clozapine (Clozaril) therapy after
Now the client's speech is garbled as a result of drug- several other antipsychotic agents fail to relieve her
induced rhythmic tongue protrusion. What is another psychotic symptoms. The nurse instructs her to return
name for this extrapyramidal symptom? for weekly white blood cell (WBC) counts to assess for
Tardive dyskinesia which adverse reaction?
Granulocytopenia

49. A man with a 5-year history of multiple psychiatric


admissions is brought to the emergency department by
the police. He was found wandering the streets
disheveled, shoeless, and confused. Based on his
previous medical records and current behavior, he is
diagnosed with chronic undifferentiated schizophrenia.
The nurse should assign highest priority to which
nursing diagnosis?
Anxiety

50. A client is admitted to the psychiatric unit with


active psychosis. The physician diagnoses schizophrenia
after ruling out several other conditions. Schizophrenia
is characterized by:
Disturbances in affect, perception, and thought content
and form.

You might also like